You are on page 1of 106

ANLISIS DIMENSIONAL

FSICA MEDIR
Ciencia que estudia los fenmenos de la naturaleza
(fenmenos fsicos), para lo cual se vale de la Es averiguar cuantas veces est contenida la unidad de
observacin y experimentacin. una magnitud.
Todo lo que nos rodea y lo que hacemos diariamente,
tiene que ver con algn fenmeno fsico.
TIPOS DE MAGNITUDES DEBIDO A SU ORIGEN
MAGNITUD
Es todo aquello que est sujeto a un aumento o 1. Magnitudes Fundamentales
disminucin y sirve para caracterizar alguna propiedad
fsica de la naturaleza. Son aquellas elegidas como base para fijar las
unidades y en funcin de las cuales se expresan las
UNIDAD dems magnitudes. Segn el Sistema Internacional
Es una porcin de magnitud que se toma como referencia de Unidades (S.I.); son 7 :
para comparar magnitudes de la misma especie.

MAGNITUD UNIDAD SMBOLO DIMENSIN

1. Longitud Metro m L

2. Masa Kilogramo kg M

3. Tiempo Segundo s T

4. Temperatura Kelvin K

5. Intensidad de corriente Ampere A I

6. Intensidad luminosa Candela cd J

7. Cantidad de sustancia Mol mol N

MAGNITUDES DERIVADAS EJERCICIOS

Son aquellas que se expresan en funcin de las Hallar la ecuacin dimensional de las siguientes
magnitudes fundamentales. magnitudes :

ECUACIONES DIMENSIONALES 1. [a] a : Aceleracin

Son aquellas que expresan la relacin existente entre la 2. [F] F : Fuerza


magnitud derivada y las magnitudes fundamentales.
3. [P] P : Presin
Notacin : [N] N = Magnitud
4. [d] d : Densidad
Se lee : Ecuacin dimensional de N o dimensiones de N
5. [P] P : Peso
Ejemplo :
[V] V : Velocidad (Calcularemos la ecuacin 6. [W] W : Trabajo
de la velocidad)
7. [P] P : Potencia
V= (Expresamos cada trmino de esta
frmula en funcin de las 8. [EK ] EK : Energa cintica
magnitudes fundamentales)
9. [EP ] EP : Energa potencial
-1
V= [V] = LT

-1-
ASOCIACIN EDUCATIVA PITGORAS Fsica

PROPIEDADES ORIGEN DEL S.I


Para descubrir las leyes que gobiernan los fenmenos
1. Los nmeros reales en todas sus formas tienen naturales, los cientficos deben llevar a cabo mediciones
ecuacin dimensional igual a la unidad. de magnitudes relacionadas con dichos fenmenos. La
fsica, en particular, suele ser denominada ciencia de la
[N] = 1 medicin. Lord Kelvin, destacada fsica ingls del siglo
9 pasado, destac la importancia de las mediciones en el
Nmero estudio de la ciencias, por medio de las siguientes
palabras :
Ejemplo :
[7] = 1 [ ]=1 [] = 1 Siempre digo que s es posible medir aquello de lo
[Senx] = 1 [Log4] = 1 [0,5] = 1 que se habla y se consigue expresarlo en nmeros,
2 2 entonces puede saberse algo al respecto; pero
[4L ] = L
cuando no puede expresarse as, el conocimiento es
2. Las magnitudes no cumplen con las leyes de la suma deficiente e insatisfactorio.
y resta
Como sabemos, para efectuar una medicin es necesario
L+L=L M-M=M 2F + F = F escoger una unidad para cada magnitud. El
e s t a b l e cim iento de unidades, r e c o n o c i d a s
3. Principio de la homogeneidad internacionalmente, tambin es imprescindible en el
Si es una ecuacin dimensionalmente correcta se comercio y en el intercambio entre los pases.
suman, se restan y/o se igualan trminos, estos tienen Antes de que el Sistema Mtrico Decimal fuese instituido
la misma ecuacin dimensional. (a fines del siglo XVIII) las unidades de medida se
definan muy arbitrariamente y variaban de un pas a otro,
A + B = C - D [A; B; C; D : magnitudes] dificultando las transacciones comerciales y el
[A] = [B] = [C] = [D] intercambio cientfico entre las naciones. Las unidades de
longitud, por ejemplo, casi siempre se derivan de las
OBSERVACIN : Si nos dicen que un nmero o dimensiones de ciertas partes del cuerpo del monarca de
constante es adimensional, entonces su ecuacin un pas; por ejemplo, la yarda, el pie, la pulgada, etc. An
dimensional es igual a la unidad. en la actualidad, en los pases de habla inglesa se utilizan
todava unidades como stas, pero se definen
modernamente con bases en patrones menos arbitrarios.
MAGNITUD FRMULA UNIDAD Tambin podemos destacar otras inconveniencias de las
DERIVADA DIMENSIONAL unidades antiguas; sus mltiplos y submltiplos no eran
decimales, lo cual dificultaba enormemente la realizacin
rea de las operaciones matemticas con dichas medidas.
Hasta hace poco tiempo, los extranjeros en Inglaterra
Volumen tenan muchos problemas para efectuar operaciones con
las monedas inglesas, pues el sistema monetario britnico
Densidad
no era decimal (1 libra esterlina vala 12 chelines y 1
Velocidad cheln, 20 peniques).
Las inconvenciencias que acabamos de sealar llevaron
Aceleracin a algunos cientficos de los siglos XVII y XVIII a proponer
unidades de medida definidas con mayor rigor y que se
Fuerza adoptaran en forma universal. Las diversas propuestas,
aunque no tuvieron aceptacin inmediata, acabaron por
Presin dar lugar al establecimiento del llamado Sistema Mtrico
Decimal, en Francia. La firma del decret del 7 de abril de
Trabajo 1795, que instaur este sistema, constituy una de las
contribuciones ms significativas de la Revolucin
Potencia Francesa.
Por esa misma poca, el Sistema Mtrica Decimal ya se
Energa
empezaba a conocer en otros pases y en 1875 se
Impulso efectu en Pars la clebre Convencin de Metro, en la
que 18 de las naciones ms importantes del mundo se
Velocidad comprometieron a adoptarlo. Inglaterra no asisti a dicha
angular reunin, negndose a emplear las unidades de este
sistema.
Aceleracin Desde entonces, el uso del sistema mtrico se fue
angular extendiendo poco a poco en todo el mundo. Nuevas
unidades para medir otras magnitudes, conservando las
Frecuencia mismas caractersticas que se emplearon en la definicin
del metro, fueron incorporndose al sistema. Por otra
Capacidad parte, la precisin de los patrones establecidos en el siglo
calorfica pasado no bastaba en el gran avance cientfico del siglo
XX. As que los cientficos advirtieron la necesidad de
Carga elctrica una reestructuracin del sistema mtrico, y en 1960,
ava
durante la 11 Conferencia Genera de Pesas y Medidas,
Fuerza tambin llevada a cabo en Pars, se elabor un nuevo
electromotriz sistema denominado Sistema Internacional de Unidades
(S.I.).

-2-
ASOCIACIN EDUCATIVA PITGORAS Fsica

PROBLEMAS PROPUESTOS
01. Hallar la dimensin de E en el Sistema Internacional 07. Hallar la dimensin de [ab], si:

D = Densidad; V = Velocidad lineal


g = Aceleracin
S y Q : Fuerzas
-2 -1 -2
A) ML B) ML C) LMT R y d : Longitudes
-3
D) LM E) ML
-2 -2 -3 -1
A) ML T B) ML T C) MLT
2 -3 -3
02. En la siguiente frmula fsica: D) ML T E) MLT
2
E = AV + BP
donde: 08. En la siguiente expresin:
E = Energa; V = Velocidad; P = Presin, hallar [A/B]

A) ML-3 B) ML2 C) ML2T-3


-3 -4
D) ML T E) ML

03. Sabiendo que el impulso es I = Ft, encontrar las donde : F = Fuerza


dimensiones de Z para que la siguiente ecuacin V = Velocidad
sea dimensionalmente correcta : hallar la dimensin de b
-1 -1
A) M T B) MT C) MT
2
D) LT E) MT
W = Trabajo; F = Fuerza; m = Masa; t: Tiempo
2 -1 -2 09. Se ha experimentado que la velocidad del sonido V
A) LT B) LT C) LT en un gas es slo funcin de la densidad d del gas
-3 2 -1
D) LT E) L T y de su coeficiente de comprensibilidad B. Cul es
la frmula que expresa la velocidad del sonido en
04. Hallar x + y para que la siguiente ecuacin sea funcin de las caractersticas del gas, si el mdulo de
dimensionalmente correcta: compresibilidad tiene dimensiones de presin? ([K]
= 1)

Donde: H = Altura; b = Radio; a = Veloc idad


A) V = K B) C)
c = Aceleracin

A) 1 B) 2 C) 3 D) E)
D) 4 E) 5

05. Determinar las dimensiones que debe tener Q para 10. El valor de la velocidad tangencial (V) de un satlite
que la expresin propuesta esa dimensionalmente artificial terrestre est dado por la siguiente expresin:
a b
correcta: V = AR g

W = 0,5mV + AgH + BP donde:
A = Es un nmero
Q=A
R = Radio de curvatura
V = Velocidad; W = Trabajo; m = Masa g = Aceleracin de la gravedad
h = Altura; P = Potencia Hallar el valor de a y b
g = Aceleracin de la gravedad
= Exponente desconocido A) -1/2; -1/2 B) 1/2; 1/2 C) 1; 1
A y B son magnitudes desconocidas D) -1; -1 E) 2; 2
2 1/2 3 2/3 2/3 2/3
A) M T B) M T C) LM T 11. Se sabe que el periodo (P) de revolucin de un
3/2 1/2
D) M T E) 1 satlite alrededor de un planeta depende del radio de
la rbita (R), de la constante de gravitacin universal
06. Se da la siguiente ecuacin dimensionalmente (G) y de la masa del planeta alrededor del cual orbita.
correcta: Hallar una expresin para la masa del planeta si la
constante de gravitacin universal :

siendo:
V = Volumen; t = Tiempo; h = Altura (K: Constante de proporcionalidad)
Determinar la expresin dimensional de:
E=
A) B) C)

A) T-3 B) T-2 C) T-1 D) E)


2 3 -3
D) M L E) MT

-3-
ASOCIACIN EDUCATIVA PITGORAS Fsica

12. La fuerza de resistencia (R) que se crea a causa de donde:


la diferencia de presiones en los bordes delantero y m = Masa; a = Aceleracin; R = Longitud
posterior de un cuerpo en movimiento en el interior de b = Constante numrica; F = Fuerza
2
un fluido, est dada por la siguiente expresin: A) 1 B) MT C) MT
-1 -2
D) MT E) M

donde: 17. En un experimento de Fsica se comprob que la


UNA
C : Coeficiente adimensional relacin: QPF = (FAV)
: Densidad del fluido es dimensionalmente correcta
V : Velocidad relativa del cuerpo respecto al fluido siendo:
S : Superficie transversal del cuerpo P = Presin; F = Fuerza; A = rea
Hallar: , , V = Volumen; U = Energa.
A) 1; 1; 1 B) 2; 1; 1 C) 1; 2; 1 Cules son las dimensiones de N?
-4 -1 -2 -2 -2 -1
D) 2; 2; 1 E) 1; 2; 2 A) L M T B) LMT C) L M T
2 -1 -3 3 -1
D) L M T E) L MT
13. Un chorro de agua con densidad (D) y velocidad (V)
choca contra un rea (A). La fuerza que ejerce el 18. Hallar las dimensiones de y para que la expresin:
chorro de agua contra la superficie tiene la siguiente
forma:
x y z sea dimensionalmente correcta siendo :
F= VAD P = Presin; m = Masa; V = Velocidad
Hallar la frmula fsica correcta e = 2,73
2 3 -3 -2 -1
A) F = V AD B) F = VAD C) F = V AD A) T B) T C) T
-2
D) F = AVD
3
E) F =
5
VAD
2 3 D) MT E) MT

14. En la expresin: 19. La siguiente ecuacin nos define la velocidad en


ABX = 3CSen(2B/C Y) funcin del tiempo (t) de un cuerpo que se desplaza
hallar [X]/[Y], sabiendo que: sobre una superficie horizontal: V = AWCos(Wt). De
A = Potencia; B = Velocidad; C = Trabajo las siguientes proposiciones, podemos afirmar que es
A) MT-1 B) MT C) M-1T (son) verdadera(s) :
-1
-1 -1
D) M T E) MT2 I. [W] = T
II. [A] = L
-1
15. La siguiente ecuacin es dimensionalmente correcta. III. [V] = LT
3
Hallar [x/k ] A) Slo I B) Slo II C) Slo III
D) I y II E) Todas
20. La fuerza de rozamiento que sufre una esfera dentro
donde: de un lquido est dado por la frmula emprica:
x y z
E = Fuerza; d = Densidad; M = Masa F = Kn r V
P = Potencia; = Aceleracin angular siendo:
3 4
A) ML T
2 4
B) ML T C) MLT
2 K = Constante numrica
2 3 2 2 5
D) M L T E) M L n = Viscosidad =

16. Hallar la dimensin de V en la siguiente ecuacin r = Radio; V = Velocidad


dimensional y correcta: El valor de (x + y + z) es:
A) 1 B) -1 C) 2
D) -2 E) 3

TAREA
01. En qu unidades puede expresarse x para que la 03. Hallar la ecuacin dimensional del potencial elctrico
siguiente ecuacin sea dimensionalmente correcta: (V) :

donde: 2 -3 -1 -1 -2 -2
P = Presin; g = Aceleracin de la gravedad A) ML T I B) MLTI C) ML TI
2 -3 2 -2
d = Densidad; V = Velocidad D) ML TI E) ML TI
2
A) kg B) g/cm C) Newton
3
D) Joule E) kq/m 04. Hallar las unidades de A en el S.I :
2 3
02. Si la siguiente ecuacin Ax + By = C es
dimensionalmente homognea. Hallar [x/y] si:
A es una velocidad; B es una fuerza :L donde:
C es una aceleracin L y b = Longitudes; t = Tiempo; a = rea
1/3 -1/2 2/3 2 -2/3 -2
A) M T B) M T C) M T A) m/s B) m/s
2
C) s
-2
1/2 2 1/3 2
D) M T E) M T 2 2
D) m /s E) Es adimensional

-4-
ASOCIACIN EDUCATIVA PITGORAS Fsica

05. En la siguiente ecuacin dimensionalmente correcta: 08. La ecuacin es homognea. Sabiendo que
E = fuerza; P = Presin y D = Densidad, hallar las
unidades de A en el S.I.
hallar la ecuacin dimensional de .
donde: -1
V = Velocidad; F = Fuerza; L = Longitud A) kg.m B) kg.m C) kg.m.s
2 -2
A) 1 B) L C) LM D) kg.m E) kg.m
2
D) L E) M
09. La velocidad de propagacin de una onda en una
06. En la siguiente ecuacin dimensional y correcta: cuerda depende de la tensin de la cuerda (F), su
masa (m) y longitud (L). Hallar la frmula emprica
para la velocidad.
donde: A) B) C)
= Velocidad angular; a = Aceleracin
t = Tiempo
hallar [x. y. z] D) E) V = KFLm
-3 2 -2 2 -1
A) L B) L T C) L T
-2 3
D) LMT E) L M
10. Si se cumple que la ecuacin es dimensionalmente
07. Determinar la presin (P) dinmica ejercida por un correcta:
lquido que fluye sobre un objeto sumergido, UNA + UNI = IPEN
asumiendo que la presin depende de la velocidad U = Energa y R = Radio
del lquido (D) Y de su velocidad (V). calcular: [PERU]
(K : Constante adimensional) A) M2L5T-4 B) ML5T-6 C) ML-3T-6
-2 2
A) P = K B) P = KdV
1/2 2
C) P = Kd V D) MLT E) MLT
2 2
D) P = KdV E) P = Kd V-2

ANLISIS VECTORIAL

VECTOR Ejemplos :
* Sean vectores Y
Es un ente matemtico que grficamente se representa
por un segmento de recta orientado
* Sean vectores Y
* La fsica utiliza los vectores para representar las
magnitudes vectoriales II. RESTA DE VECTORES
Es una operacin que tiene por finalidad, hallar un
vector denominado vector diferencia , el cual es
igual a la resta de vectores

Ejemplo :

* Sean vectores Y

MTODOS PARA CALCULAR LA RESULTANTE

A. MTODO DEL PARALELOGRAMO

Se utiliza para calcular la resultante de dos vectores


* En general un vector se representa de la siguiente concurrentes y coplanares que tienen un mismo
forma : punto de origen.
Grficamente se construye un paralelogramo
trazando paralelas a los vectores. El vector resultante
se traza uniendo el origen de los vectores con la
intercepcin de las paralelas

OPERACIONES VECTORIALES

I. SUMA DE VECTORES O COMPOSICIN


VECTORIAL

Es una operacin que tiene por finalidad hallar un


nico vector denominado vector resultante , el
cual es igual a la suma de todos los vectores.
Vector resultante :

-5-
ASOCIACIN EDUCATIVA PITGORAS Fsica

Mdulo de R :

B. MTODO DEL POLGONO


Casos particulares :
Se utiliza para calcular la resultante de un conjunto de
A. Si = 0 (A88B) vectores concurrentes y coplanares.

* Se obtiene el mximo valor del mdulo de la Es un mtodo grfico que utiliza escalas apropiadas
resultante y consiste en trazar los vectores uno a continuacin
del otro manteniendo sus caractersticas. El vector
R = A + B = Rmx resultante se traza uniendo el origen del primer
vector con el extremo del ltimo vector.

Ejemplo :
B. Si = 180 (A98B)
Sean vectores
* Se obtiene el menor valor posible de la
resultante

R = A - B = Rmn

C. Si = 90 (A z B) Construimos el polgono vectorial :

* Se obtiene aplicando el Teorema de Pitgoras

Propiedad :

Cuando los dos vectores A y B son iguales en mdulo


C. M T O D O DE LAS COMPONENTES
RECTANGULARES

COMPONENTES RECTANGULARES DE UN
VECTOR

Son aquellos vectores que resultan de proyectar un


vector sobre dos (o tres) ejes perpendiculares entre
D. Si = 60 s :

E. Si = 120

Se cumple que :

Ax = ACos

Ay = ASen

-6-
ASOCIACIN EDUCATIVA PITGORAS Fsica

El mtodo de las componentes rectangulares permite


calcular el mdulo y la direccin de la resultante de Si la direccin de es 0
un conjunto de vectores. Pasos a seguir: -
1 Se halla las componentes rectangulares
2 Se calcula la resultante en cada uno de los ejes Si la direccin de es 90
coordenados (Rx; Ry) -
3 Se calcula el mdulo de la resultante aplicando
Pitgoras y su direccin aplicando la funcin
tangente Si la direccin de -

PROBLEMAS PROPUESTOS
01. Hallar el mdulo de la resultante de los vectores 05. Se tiene los vectores , los cuales forman un
, si se sabe que: A = ;B=4 ngulo de 120. Hallar la relac in , si:

A) 1/5 B) 1/7 C) 2/9


D) 2/5 E) 2/7

06. La resultante mxima que se puede obtener con


es 12 y la mnima es 3. Qu mdulo tendr la
A) B) 5 C) 2 resultante de si formarn 60 entre s?
D) E)
A) 6 B) 7,5 C) 10,5
02. Determinar el mdulo de la resultante si M es punto D) 12 E) 5
medio de los vectores (A = 2; B = 10)
07. Determinar el mdulo del vector resultante.

A) 3 B) 5 C) 8
D) 2 E) 7
A) 30 B) 30 C) 50
03. Dos vectores A y B forman 60 entre s siendo el
D) 50 E) 30
mdulo de la resultante y el mdulo del vector
diferencia . Si A y B formarn 90 entre s, el 08. Hallar el mdulo de la resultante de los vectores
mdulo de la resultante sera mostrados, sabiendo que M, N y O son puntos
A) 1 B) C) 2 medios.
D) 4 E) 2

04. Determinar la magnitud del vector resultante de los


vectores que se muestran en la figura. Si A = 2 u;
B = 3 u, C = uyD=1u

A) 8 B) 4 C) 2
D) Cero E) 6
A) 5 B) 6 C) 4
D) 7 E) 3

-7-
ASOCIACIN EDUCATIVA PITGORAS Fsica

09. Determinar el mdulo del vector resultante si: A) B) C)


D) E)

14. Si ABCD es un cuadrado, hallar en funcin de


.

A) 10 cm B) 20 cm C) 4 cm
D) 7 cm E) 15 cm
10. Dados los siguientes vectores, hallar el mdulo de la
resultante de los vectores mostrados = 3 y = 4,
A) B) C)
siendo perpendiculares.
D) E)

15. Si: , el mdulo del vector es:


Datos:

A) 6 B) 8 C) 10
D) 15 E) 5
11. Hallar el mdulo del vector resultante, si se sabe que:
B = 3; D = 4; E = 5

A) 4 B) 12 C) 4
D) 8 E) 0

16. Si la resultante del grupo de vectores es nula, hallar


A) 34 B) 29 C) 19
D) 14 E) 8
12. Si el exgono regular es de lado L, hallar el mdulo
del vector resultante.

A) 30 B) 37 C) 45
D) 53 E) 60

17. Si la resultante en el eje X es 10, hallar

A) L B) L C) L
D) L E) L

13. Hallar en funcin de

A) 30 B) 37 C) 45
D) 53 E) 60

-8-
ASOCIACIN EDUCATIVA PITGORAS Fsica

18. El sistema mostrado tiene una resultante nula. Hallar A) 37; 80 N B) 60; 40 N C) 65; 30 N
el ngulo D) 83; 75 N E) 53; 20 N

20. Hallar la resultante:

A) ArcCos(1/ ) B) ArcCos(1/ )
C) 30 D) 53
E) ArcCos(1/ ) A) B) C)
D) E)
19. Hallar el ngulo y el mdulo de la resultante de las
fuerzas mostradas, sabiendo que la resultante se
encuentra sobre la lnea de accin de la fuerza
de 90 N.

TAREA
01. Determinar el vector que debe adicionar al sistema de 04. Se tiene los vectores , los cuales forman un
vectores mostrados de tal manera que la resultante
ngulo de 120. Hallar la relac in , si:
sea (-6; 2)

A) 1/5 B) 1/7 C) 2/9


D) 2/5 E) 2/7
05. Hallar el vector resultante de los vectores mostrados.
La figura es un paralelogramo

A) B) (-2; 0) C) (-5; 2)
D) E)

02. Hallar el mdulo del vector resultante:

A) 2 B) - C) 2
D) -2 E) -
06. Calcular el mdulo de la resultante de los vectores
mostrados, si

A) 4 B) 5 C) 6
D) 7 E) 8
03. Los vectores poseen como resultante mxima y
mnima 10 cm y 2 cm respectivamente. Si cuando
forman un ngulo su resultante es 8 cm, hallar el
ngulo que forma esta resultante con el vector de
mayor mdulo.
A) ArcCos(7/8) B) ArcCos(2/3)
C) ArcCos(1/5) D) ArcCos(6/7) A) 7 B) 14 C) 21
E) ArcCos(2/9) D) 28 E) 30

-9-
ASOCIACIN EDUCATIVA PITGORAS Fsica

07. Sean los vectores en un rombo de lado L 09. Hallar en funcin de

que mide . Hallar el mdulo del vector , si la

resultante pasa por la diagonal mayor (M y N son


puntos medios)

A) B) C)

D) E)
A) L /2 B) L C) L
D) L /4 E) L /3 10. Se tiene dos vectores para los cuales se
cumplen que . Hallar en qu
08. En la figura M es punto medio de . Si relacin se encuentran los mdulos de los vectores
, hallar y el ngulo que forman dichos vectores.
A) K = J ; 60 B) J = K ; 135
C) K = J ; 150 D) K = 2J; 127
E) J = 2K; 106

D) 3 B) 2 C)
D) 2 E) 3

MOVIMIENTO RECTILNEO UNIFORME


MOVIMIENTO RECTILNEO UNIFORMEMENTE VARIADO
CINEMTICA t Movimiento segn la trayectoria:

CONCEPTO: a. Rectilneo
Es la parte de la ciencia fsica llamada mecnica, que se
encarga del estudio de la geometra del movimiento pero
sin analizar sus causas. b. Parablico
MOVIMIENTO:
Es el fenmeno fsico en donde un cuerpo, llamado mvil,
cambia de posicin con respecto a un sistema de
referencia al cual se considera fijo. El movimiento es
relativo al sistema de referencia y generalmente se
c. Circular
consideran sistemas de referencia fijos a la tierra.

O: Origen de coordenadas
d. Elptico

ELEMENTOS DEL MOVIMIENTO:

1. TRAYECTORIA:
Lnea descrita por el mvil (recta o curva)

-10-
ASOCIACIN EDUCATIVA PITGORAS Fsica

2. VECTOR POSICIN : IMPORTANTE:


Vector dirigido desde el origen de coordenadas hasta
la posicin que ocupa el mvil en cierto instante. a) El desplazamiento no depende de la trayectoria
descrita.
3. DESPLAZAMIENTO ( ): b) La distancia recorrida es llamada tambin espacio
recorrido.
Vector dirigido desde la posicin inicial hasta la final,
indica el cambio de posicin.
RAPIDEZ PROMEDIO (Vp ):
Magnitud fsica escalar que indica la rapidez de la
4. DISTANCIA RECORRIDA (d):
distancia recorrida durante un cierto intervalo de tiempo.
Longitud de la trayectoria descrita por el mvil.

VELOCIDAD INSTANTNEA ( ):
Magnitud fsica vectorial que se determina mediante el
= xf - xo = x cociente del desplazamiento efectuado y el intervalo de
tiempo empleado cuando este es muy pequeo (tiende a
cero). Indica la rapidez y direccin del movimiento en un
instante cualquiera, en el movimiento curvilneo es
tangente a la trayectoria.

(a) Movimiento Rectilneo:

= =

APLICACIN (b) Movimiento Circulneo:

a.

= x = 20 - 2 = +18

b.
t Movimiento rectilneo:

| m| = Vp

= (4; 8) t IMPORTANTE:
= (12; 3) a. Si V = Constante, el movimiento es uniforme.
=(12; 3) - (4; 8) b. Si V Constante, el movimiento es variado.

ACELERACIN MEDIA ( m ):
VELOCIDAD MEDIA ( m ): Magnitud fsica vectorial que expresa la rapidez del
Magnitud fsica vectorial que indica la rapidez y direccin cambio de la velocidad durante un intervalo de tiempo.
del desplazamiento efectuado durante cierto intervalo de
tiempo.

t : Cambio vectorial de la velocidad


t Unidades: m/s2

-11-
ASOCIACIN EDUCATIVA PITGORAS Fsica

ACELERACIN INSTANTNEA ( ): t IMPORTANTE:


Magnitud fsica vectorial que se determina mediante el
cociente del cambio de la velocidad instantnea y el
intervalo de tiempo transcurrido cuando este es muy
pequeo. Indica la rapidez y direccin del cambio de la
velocidad en un instante cualquiera, en el movimiento 0
curvilneo est dirigida al interior de la trayectoria.
TIEMPO DE ENCUENTRO (tE ):
Si dos mviles parten simultneamente al encuentro,
efectuando ambos un MRU, entonces el tiempo que
emplearn en encontrarse estar dado por la siguiente
expresin:
a. Movimiento Rectilneo

t d1 + d 2 = D
pero: d = V.t
0 V1. t = V2 .t = D
b. Movimiento Curvilneo 0 t(V1 + V2) = D

t Sentido fsico de la velocidad:


Si V = 10 m/s=Cte.
t Componentes de la aceleracin 0 Se recorren 10 m por cada segundo

TIEMPO DE ALCANCE (tA):


Si un mvil va al alcance de otro que se aleja
simultneamente en la misma direccin, efectuando
ambos un MRU, entonces el tiempo que emplea en
alcanzarlo estar dado por la siguiente expresin:

: Aceleracin tangencial
: Aceleracin normal

0a=

MOVIMIENTO RECTILNEO UNIFORME (MRU)

CONCEPTO: Movimiento efectuado en lnea con


velocidad constante, el mvil recorre distancias iguales en
intervalos de tiempo tambin iguales. OBSERVACIN:
Ubiquemos al mvil en un eje coordenado.

LEYES DEL MRU Sabemos que: d = V.t


t = Constante Pero:
t d es directamente proporcional al t

ECUACIONES DEL MRU 0


Donde: o = Posicin inicial
= Posicin final
t = Intervalo de tiempo transcurrido

-12-
ASOCIACIN EDUCATIVA PITGORAS Fsica

MOVIMIENTO RECTILNEO UNIFORMEMENTE PARA RECORDAR :


VARIADO (MRUV)
Si la aceleracin es constante, entonces no cambia de
CONCEPTO valor ni direccin.
Es el movimiento efectuado en lnea recta con aceleracin
constante, en tiempos iguales el mvil efecta cambios de L Sentido fsico de la aceleracin:
velocidad tambin iguales pero recorre distancias Si : a = 3 m/s2 = constante
diferentes.
! La velocidad cambia en 3 m/s por cada segundo.

IMPORTANTE :

a)

LEYES DEL MRUV

L = Constante
Acelerado por que aumenta la velocidad
L El cambio de velocidad V= Vf - Vi es directamente
proporcional al tiempo transcurrido t b)
! = Constante

L Si la velocidad inicial es nula, la distancia recorrida es


directamente proporcional al tiempo transcurrido al
cuadrado
! = constante
Retardado por que disminuye la velocidad

OBSERVACIN : Distancia recorrida en el ensimo


ACELERACIN EN EL MRUV ( )
segundo

LOS NMEROS DE GALILEO


2
* Unidad : m/s
Si el mvil parte del reposo (velocidad inicial nula) y viaja
con MRUV, se cumple que las distancias que recorre en
intervalos de tiempos iguales son directamente
CASOS : proporcionales a los nmeros impares, los cuales son
(a) Movimiento acelerado : V f > Vi denominados los nmeros de Galileo.

(b) Movimiento retardado : V f < Vi

Ejemplo

Suponga que un mvil parte del reposo y acelera, si en


los 2 s iniciales recorre 4 m, cunto recorrer en los 6 s
siguientes?
ECUACIONES DEL MRUV
Solucin :
1. d = Vi . t En este caso consideraremos intervalos de tiempo de 2 s
cada uno
2. Vf = Vi a . t
3. 2a.d

4.

Donde :

(+) : Movimiento acelerado

(-) Movimiento retardado ! k = 4 m y x = 3k + 5k + 7k = 15k ! x = 60 m

-13-
ASOCIACIN EDUCATIVA PITGORAS Fsica

OBSERVACIN : Ejemplo :
Ubiquemos al mvil en un sistema de coordenadas
Un mvil con MRUV parte desde la posicin +2 m con una
velocidad de +4 m/s y acelera a +3 m/s. Determine su
velocidad final y su posicin final al cabo de 2 s

Solucin :

2
En este caso entonces las ecuaciones se L = 2 + 4(2) + (2) = + 16 m
expresan de la siguiente manera :

L = 4 + 3(2) = +10 m/s


1.

2.
Donde :
xo = Posicin inicial
xf = Posicin final
t = Intervalo de tiempo transcurrido

Regla de signos :

PROBLEMAS PROPUESTOS
01. Un auto se desplaza con una determinada velocidad 07. Si en el instante mostrado se enciende la vela, qu
constante V durante 4 s, recorriendo una rapidez posee el extremo de la sombra en la pared si
determinada distancia. Luego aumenta su velocidad la vela se consume a razn constante de 2 cm/s?
en 4 m/s, recorriendo la misma distancia en 3,5 s.
Hallar V en m/s
A) 28 m/s B) 14 m/s C) 7 m/s
D) 20 m/s E) 21 m/s

02. Una persona sale todos los das de su casa a la


misma hora y llega a su trabajo a las 09:00 h. Un da
se traslada al doble de la velocidad acostumbrada y
llega a su trabajo a las 08:00 h. A qu hora sale A) 2 cm/s B) 3 cm/s C) 4 cm/s
siempre de su casa? D) 5 cm/s E) 6 cm/s
A) 06:00 h B) 07:00 h C) 08:00 h
D) 05:00 h E) 04:00 h 08. Un auto recorre rectilneamente la primera mitad del
camino con una velocidad de 60 km/h y la segunda
03. Roco ha estado viajando durante 4 h. Si hubiera mitad en una direccin perpendicular a la primera
viajado 1 h menos con una velocidad mayor en con una velocidad de 120 km/h. Hallar la velocidad
5 km/h, habra recorrido 5 km menos. Cul es su media y promedio en km/h
velocidad en km/h? A) 40 ; 80 B) 20 ; 40 C) 60; 60

A) 4 km/h B) 5 km/h C) 10 km/h D) 50; 80 E) 10 ; 20


D) 8 km/h E) 20 km/h
09. Un tren de 100 m demora 10 s en pasar a una
04. Abelito observa que caminando a razn de 0,8 m/s persona que se mueve en su misma direccin
de su casa a la academia Pitgoras tarda 4 min ms con 2 m/s, y en pasar un tnel 40 s. Determinar la
que caminando a 0,9 m/s. Cul es la distancia de su longitud del tnel
casa a la academia? A) 100 m B) 200 m C) 280 m
D) 380 m E) 400 m
A) 1 000 m B) 1 500 m C) 2 200 m
D) 1 750 m E) 1 728 m 10. Dos mviles situados en una misma lnea recta
separados 1,5 km parten simultneamente con
05. Una persona est situada a 510 m de un gran cerro. velocidades constantes de 42 m/s y 28 m/s
Si emite un grito, despus de qu tiempo escuchar alejndose cada vez ms. Al cabo de qu tiempo
el eco? estarn separados 5 km?
(Vsonido=340 m/s) A) 20 s B) 50 s C) 100 s
A) 1,5 s B) 2,0 s C) 2,5 s D) 80 s E) 60 s
D) 3,0 s E) 4,0 s
11. Un ciclista se desplaza con una rapidez constante
06. Cunto tiempo demora un tren de 400 m de de 4 m/s, de pronto observa un bache en su
longitud que viaja a una velocidad de 72 km/h en camino y gira 74 su timn. Si la maniobra dur 0,4
pasar por un tnel de 80 m de largo? segundos, qu aceleracin media experiment el
ciclista si mantuvo constante su rapidez en el giro?
2 2 2
A) 36 s B) 42 s C) 18 s A) 4,8 m/s B) 8,6 m/s C) 10 m/s
2 2
D) 24 s E) 50 s D) 6 m/s E) 12 m/s

-14-
ASOCIACIN EDUCATIVA PITGORAS Fsica

12. En el diagrama para que el mvil vaya de A hacia A) 25 m/s B) 35 m/s C) 45 m/s
B emplea 2 segundos, observndose que en A D) 55 m/s E) 65 m/s
su rapidez es 8 m/s y que en B es 4 m/s. Qu
aceleracin media tendr el mvil en este trayecto? 16. Un mvil con MRUV parte con cierta velocidad inicial.
Si al recorrer 180 m en 12 s logra duplicar su
velocidad, el mdulo de su velocidad al transcurrir los
6 primeros segundos es :
A) 5 m/s B) 10 m/s C) 15 m/s
D) 20 m/s E) 25 m/s

17. Un cuerpo que se mueve con MRUV recorre 36 m en


3 segundos. Cuntos metros recorri en el 2do.
segundo de los 3 segundos?
2 2 2
A) 2 m/s B) 4 m/s C) 10 m/s A) 18 m B) 16 m C) 14 m
2 2 D) 12 m E) 6 m
D) 6 m/s E) 5 m/s

13. Una pelota de tenis que viaja con una velocidad 18. Un leopardo africano cuya aceleracin es 8 m/s2,
2
horizontal de 10 m/s, recibe un golpe de raqueta que persigue a una gacela cuya aceleracin es 5 m/s
lo devuelve con una velocidad horizontal de 15 m/s y est ubicada a 150 m de l. Si ambos parten del
en sentido opuesto a su velocidad inicial. Si el golpe reposo simultneamente, calcular el tiempo que
de la raqueta dur 0,5 s, cul es el valor de la tarda el leopardo en atrapar la gac ela
aceleracin media que experimenta la pelota durante A) 2 s B) 3 s C) 4 s
el golpe? D) 9 s E) 10 s
2 2 2
A) 10 m/s B) 5 m/s C) 15 m/s
D) 25 m/s
2
E) 50 m/s
2 19. Un mvil con MRUV recorre 50 m en los dos primeros
segundos, 75 m en los siguientes 2 segundos.
14. Se muestra la incidencia y rebote elstico de una Cuntos metros recorrer en los siguientes
partcula. Si el choque dura un tiempo t y la 4 segundos?
velocidad de incidencia es V, hallar el mdulo de la A) 90 m B) 100 m C) 115 m
aceleracin media en el tiempo t D) 125 m E) 225 m

20. Los mviles A y B parten del reposo con aceleracin


2 2
de 1 m/s y 2 m/s respectivamente. Si B parte 1 s,
despus que A,qu distancia los separa cuando
sus velocidades tengan igual valor?
A) B) C)

D) E) Cero

15. Un auto parte del reposo y viaja durante 4 s, en ese


instante su velocidad es de 20 m/s. Si contina con la A) Cero B) 2 m C) 15 m
misma aceleracin, la velocidad que tendr a los 9 s D) 8 m E) 12 m
de iniciado su recorrido es :

TAREA
01. Un mvil parte del reposo con una aceleracin A) 5 s B) 7 s C) 10 s
2
de 3 m/s y en un determinado instante desacelera D) 13 s E) 12 s
hasta detenerse. Cul es la aceleracin en este
04. Dos mviles parten de un mismo punto en
segundo tramo, si el mvil estuvo en movimiento
direcciones opuestas, dirigiendose respectivamente
12 s y recorri una distancia total de 144 m?
2 2 2 a P y a Q. Luego de llegar a su destino emprenden
A) +3 m/s B) +4 m/s C) +5 m/s
2 2 el retorno. A qu distancia de Q se vuelven a
D) +6 m/s E) +8 m/s
encontrar?
02. Dos mviles estn ubicados en un mismo lugar de
una pista rectilnea a una distancia de 80 m de un
poste. Si estos empiezan a acercarse
simultneamente al poste con rapidez constante de
35 m/s y 45 m/s, cunto tiempo debe transcurrir
para que los autos equidisten del poste? A) 20 m B) 30 m C) 15 m
A) 1 s B) 2 s C) 3 s D) 10 m E) 25 m
D) 4 s E) 5 s
05. Dos autos parten de dos puntos A y B distantes
03. Un auto viaja con una velocidad de 10 m/s y divisa 1 200 m con velocidades de 50 m/s y 60 m/s
13 m delante de l un camin que viaja a una respectivamente uno al encuentro del otro. El
velocidad constante de 36 km/h en el mismo sentido segundo parte 2 s despus que el primero. Qu
que el auto. Si el auto para rebasar el camin acelera distancia los separa cuando el segundo llegue al
2
con a=2 m/s , calcular el tiempo necesario para punto A?
lograrlo (longitud del auto y camin: 3 m y 9 m A) 800 m B) 900m C) 1 000 m
respectivamente) D) 1 100 m E) 1 200 m

-15-
ASOCIACIN EDUCATIVA PITGORAS Fsica

06. Dos partculas A y B se encuentran separadas 200 m. 09. Un mvil parte del reposo con aceleracin
Si parten una hacia la otra con velocidades constante recorriendo 18 m en los 3 primeros
constantes de 20 m/s y 50 m/s, qu distancia separa segundos. Calcular la distancia que recorrer el
a las partculas cuando B pasa por el punto de mvil en los 7 segundos siguientes.
partida de A? A) 182 m B) 32 m C) 64 m
A) 200 m B) 180 m C) 120 m D) 96 m E) 152 m
D) 80 m E) 40 m
10. Un tren parte de Lima a las 08:00 h y llega a
07. Dos trenes de 100 m y 200 m de longitud se acercan Huancayo a las 16:00 h, otro tren parte de Huancayo
mutuamente con velocidades constantes de 70 m/s y a las 07:00 h y llega a Lima a las 17:00 h. A qu
80 m/s respectivamente. Calcular el tiempo que distancia de Huancayo se cruzan los trenes, si la
tardan en cruzarse totalmente. distancia entre Lima y Huancayo es de 405 km y los
A) 0,2 s B) 0,5 s C) 1 s trenes llevan velocidades constantes?
D) 2 s E) 2,5 s A) 100 km B) 160 km C) 202,5 km
D) 200 km E) 300 km
08. Un mvil con MRUV, que posee una aceleracin de
4 m/s2 y parti del reposo. Determinar la distancia
que recorri en los 3 primeros segundos y la distancia
recorrida en el tercer segundo de su movimiento.
A) 18 m y 10 m B) 10 m y 10 m
C) 18 m y 18 m D) 9 m y 10 m
E) 5 m y 10 m

MOVIMIENTO VERTICAL DE CADA LIBRE


(MVCL)
Un cuerpo se encuentra en cada libre cuando se mueve OBSERVACIN :
nicamente por efecto de su propio peso. Esto implica
que toda otra fuerza (tal como la friccin del aire), es
despreciada.

OBSERVACIN : Valores de la aceleracin de la gravedad (g) en


Cada cuerpo celeste tiene su propio valor para su mdulo :
aceleracin de la gravedad.
2
Debe tenerse en cuenta que el cuerpo en cada libre En el polo : 9,83 m/s
experimenta una aceleracin especial, la cual es conocida 2
como aceleracin de la gravedad y su valor, para En el ecuador : 9,78 m/s
2
nuestros problemas, ser cerca a la Tierra de 9,8 m/s . 2
Para facilidad de los problemas, a veces se toma el valor En Lima - Per : 9,8 m/s
2
de la aceleracin de la gravedad como 10 m/s , pero es
necesario que el problema lo especifique. t ta + tb : Tiempo de subida (tsubida)

TIPOS DE MVCL t tc + td : Tiempo de bajada (t bajada)

- Hacia abajo : El movimiento es acelerado t ta + tb + tc + td : Tiempo de vuelo (tv ; tvuelo)

- Hacia arriba : El movimiento es desacelerado

-16-
ASOCIACIN EDUCATIVA PITGORAS Fsica

De la figura se deduce lo siguiente : FRMULAS ADICIONALES


1. tsubida = tbajada =
1. | |=| |; | |=| |
2. tvuelo =
2. | | = 0 (altura mxima)
3. ta = t d ; t b = t c ; t a + t b = t c + td 3. Altura mxima (hmx) : hmx =

ECUACIONES DEL MVCL


4. hn = V o g(2n - 1)
1. Vf = Vo gt
2. = 2gh hn : Altura recorrida en el nmero de segundos n
2
3. h = Vo t gt FRMULAS VECTORIALES

4. h=

REGLA DE SIGNOS

(+) : Cuando el movimiento es descendente


= Vector desplazamiento
(-) : Cuando el movimiento es ascendente Convencin : 8 (+)
9(-)
OBSERVACIN :
Por lo tanto : g(-) 9
Existen otros movimientos de cada libre que no son
verticales; por ejemplo : El movimiento parablico, el
movimiento de un satlite alrededor de la Tierra.

PROBLEMAS PROPUESTOS

01. Desde el piso se lanza un proyectil hacia arriba y 06. Un paquete ubicado a 70 m del piso es lanzado
retorna al punto de lanzamiento, al cabo de 8 verticalmente hacia arriba con Vo=20 m/s. Determinar
2
segundos. Con qu velocidad retorna al punto de a qu altura se encontrar luego de 6 s (g = 10 m/s )
2
lanzamiento? (g = 10 m/s ) A) 5 m B) 10 m C) 15 m
A) 10 m/s B) 20 m/s C) 30 m/s D) 25 m E) 35 m
D) 40 m/s E) 50 m/s
07. Un cuerpo es disparado verticalmente hacia arriba
02. Un cuerpo se suelta de una altura de 100 m. Con con una velocidad de 98 m/s. Si la altura alcanzada
qu velocidad llegar al piso y en qu tiempo? por el cuerpo coincide con la del edificio, cuntos
2
(g = 10 m/s ) pisos tiene el edificio, si cada piso tiene 5 m de altura
A) 20 m/s; s B) 20 m/s; 2 s y qu tiempo demorar en volver al piso?
2
(g = 9,8 m/s )
C) 10 m/s; s D) 5 m/s; 2 s A) 49; 10 s B) 98; 20 s C) 49; 5 s
E) 2 m/s; 3 s D) 98; 10 s E) 98; 15 s

03. Una pelota cae verticalmente al piso y rebota en l. 08. Desde el piso se lanza una pelota verticalmente
La velocidad justo antes del choque es V y justo hacia arriba a razn de 20 m/s. Al cabo de qu
despus del choque es 0,9V. Si la pelota se deja caer tiempo como mximo se encontrar a 15 m de altura?
2
desde un metro de altura, a qu altura llegar (g = 10 m/s )
despus del primer bote? (g = 9,8 m/s2) A) 1 s B) 2 s C) 3 s
A) 0,90 m B) 1,00 m C) 0,95 m D) 4 s E) 5 s
D) 0,85 m E) 0,81 m
09. Un cuerpo se lanza verticalmente hacia arriba con
04. Un cuerpo cae en el ltimo segundo un cuarto de la Vo = 30 m/s. Al cabo de qu tiempo asciende la
altura total. Qu tiempo estuvo en movimiento? ltima tercera parte de su altura mxima?
2
A) 2(2+ ) s B) (1+ ) s C) 2(1- ) s (g = 10 m/s )
A) 1,21 s B) 1,41 s C) 1,53 s
D) 2 s E) 2(1+ )s D) 1,73 s E) 1,81 s

05. Se lanza un cuerpo verticalmente hacia arriba desde 10. Una pelota se lanza verticalmente hacia arriba con
tierra con una velocidad de 40 m/s. Hallar despus de una velocidad de 10 m/s. Al cabo de qu tiempo la
qu tiempo y a qu altura se encuentra cuando su pelota poseer una velocidad de 40 m/s?
2
velocidad sea de 10 m/s hacia abajo. (g = 10 m/s )
2 (g = 10 m/s )
A) 10 s; 20 m B) 5 s; 75 m C) 5 s; 200 m A) 3 s B) 4 s C) 5 s
D) 10 s; 75 m E) 8 s; 120 m D) 6 s E) Absurdo

-17-
ASOCIACIN EDUCATIVA PITGORAS Fsica

11. Una piedra se lanza verticalmente desde un punto A A) 450 m B) 360 m C) 620 m
con una velocidad de 80 m/s. A qu distancia de A D) 210 m E) 870 m
se encontrar un punto B, en el cual la velocidad de
la piedra ser 20 m/s hacia abajo? 17. En el pozo de la figura caen sin friccin gotas a razn
2
(g = 10 m/s ) de una gota/segundo. Un objeto asciende a una
A) 100 m B) 200 m C) 300 m velocidad constante de 10 m/s y es alcanzado por
D) 320 m E) 360 m una gota cuando est a una profundidad h = 500 m.
Cunto subir, aproximadamente, el objeto hasta
12. Desde la azotea de un edificio de 80 m, una persona ser alcanzado por la segunda gota?
2
suelta una pelota de ftbol con la intencin de agarrar (g=10 m/s )
dicha pelota, otra persona distante 20 m de la base
del edificio, presentando un MRU, se dirige hacia el
lugar del impacto. Cul debe ser la velocidad V
2
para que logre su propsito? (g=10 m/s ). Desprecie
los efectos del aire

A) 3 m B) 5 m C) 7 m
D) 9 m E) 11 m

18. Un pequeo cohete comienza a moverse con una


2
aceleracin constante de 5 m/s . Si al cabo de 6 s se
A) 6 m/s B) 8 m/s C) 4 m/s termina el combustible, qu altura como mximo
D) 5 m/s E) 10 m/s logra elevarse? Desprecie los efectos del aire y
2
considere. (g=10 m/s )
13. Desde la azotea de un edificio de 80 m, se suelta un
cuerpo, luego de 2 s se lanza verticalmente hacia
abajo otro cuerpo. Si ambos llegan simultneamente
a la base del edificio, calcular con qu rapidez fue
lanzado el segundo cuerpo. Despreciar los efectos
del aire (g=10 m/s2)
A) 35 m/s B) 10 m/s C) 30 m/s
D) 60 m/s E) 20 m/s

14. Una esfera se deja en libertad desde una altura de A) 130 m B) 125 m C) 110 m
80 m y al rebotar en el piso se eleva slo hasta la D) 90 m E) 135 m
cuarta parte de la altura anterior. Qu tiempo ha
transcurrido hasta que se produce el 3er. 19. Desde una altura de 20 m, respecto a la superficie de
impacto?(g=10 m/s )
2 un lago se suelta un objeto, llegando hasta el fondo
A) 4 s B) 6 s C) 8 s del lago en 7 s. Si consideramos que el objeto se
D) 9 S E) 10 s mueve con velocidad constante estando dentro del
lago, qu profundidad tiene el lago?
2
15. De la llave de un cao malogrado que est a 7,2 m de (g=10 m/s )
altura cae una gota de agua cada 0,1 s. Cuando est A) 100 m B) 80 m C) 60 m
por caer la tercera gota, se termina de malograr el D) 120 m E) 75 m
cao y sale un chorro grande de agua. Cul deber
ser la velocidad con que sale el chorro para que 20. Desde un globo aerosttico que sube describiendo un
alcance a la primera gota, en el momento preciso que MRU con una velocidad de 10 m/s, una persona
sta choca con el piso? suelta una moneda y observa que demora 6 s en
(Considerar : g = 10 m/s )
2 llegar al suelo. Calcular desde que altura h fue
A) 1,8 m/s B) 2,0 m/s C) 2,2 m/s soltada la moneda. Desprecie los efectos del aire
2
D) 2,4 m/s E) 2,6 m/s sobre la moneda (g=10 m/s )

16. Dos cuerpos P y Q se colocan en la misma vertical


tal como se indica en la figura. El cuerpo P se lanza
hacia arriba con una velocidad de 60 m/s y en el
mismo instante Q se deja caer. Desde qu altura
x se tendr que dejar Q para que ambos se
encuentren en la mxima altura recorrida por P?

A) 120 m B) 80 m C) 100 m
D) 125 m E) 140 m

-18-
ASOCIACIN EDUCATIVA PITGORAS Fsica

TAREA
01. Desde lo alto de un edificio, una piedra es lanzada 05. Una bola empieza a deslizarse sin friccin desde el
hacia arriba con una velocidad inicial V y 5 punto superior de una circunferencia por una canaleta
segundos despus otra piedra es lanzada hacia abajo inclinada bajo un ngulo respecto a la vertical.
del mismo lugar y con la misma velocidad inicial, Dentro de cunto tiempo la bola alcanzar la
llegando ambas a tierra al mismo instante. Hallar V circunferencia si su dimetro es D?
2
(g = 10 m/s ) A) B) C)
A) 25 m/s B) 50 m/s C) 75 m/s D) E)
D) 100 m/s E) 20 m/s
06. Un observador situado a 35 m de altura ve pasar un
02. Un globo aerosttico sube verticalmente con una objeto hacia arriba y 6 s despus lo ve regresar.
velocidad de 30 m/s. El piloto del globo al encontrarse Con qu velocidad fue lanzado el objeto desde el
2
a una altura de 240 m con respecto al suelo, lanza piso? (g = 10 m/s )
verticalmente hacia abajo un tomate con una A) 10 m/s B) 20 m/s C) 30 m/s
velocidad respecto a su mano de 20 m/s. Al cabo de D) 40 m/s E) 50 m/s
qu tiempo el tomate tocar el suelo?
07. En un planeta X se deja caer una piedra desde
A) 6 s B) 7 s C) 8 s cierta altura y se observa que en 1 s determinado
D) 10 s E) 12 s recorre 26 m y en el siguiente segundo 32 m. Hallar
el valor de la aceleracin de la gravedad de dicho
03. Desde los puntos A y B se lanzan en el mismo planeta
2 2 2
instante dos objetos verticalmente hacia arriba con A) 6 m/s B) 12 m/s C) 10 m/s
2 2
velocidades V y 2V respectivamente. Si el que se D) 8 m/s E) 4 m/s
lanz de A llega slo hasta el nivel donde se ubica
el punto B, cul es la distancia que separa a los 08. Un cuerpo es lanzado verticalmente hacia arriba
objetos cuando el que se lanz de B comienza a desde la Tierra. A los 5 segundos de ser lanzado
descender? alcanza la altura h de manera que al ascender
25 m ms, slo le falta 2 segundos para alcanzar su
2
altura mxima. Hallar h.(g = 10 m/s )
A) 275 m B) 125 m C) 175 m
D) 375 m E) 385 m

09. El marco superior de una ventana de 8,25 m de


altura, se ubica a 9 m del borde de una azotea de un
edificio. Desde la azotea es lanzada verticalmente
hacia abajo una moneda con una velocidad de 4 m/s.
A) 2H B) 3H C) 4H En cunto tiempo la moneda pasar por toda la
2
D) 5H E) 6H ventana? (g=10 m/s )
A) 1 s B) 3 s C) 0,5 s
04. Un globo aerosttico se mueve verticalmente hacia D) 0,75 s E) 1,5 s
abajo con una velocidad de 20 m/s. En un instante
dado el piloto lanza una manzana con una velocidad 10. Desde el penltimo piso de un edificio se deja caer
de 35 m/s hacia arriba (respecto a su mano). Qu (V = 0) una piedra, al mismo tiempo (t = 0) que del
aceleracin retardatriz deber imprimir al globo para ltimo piso se lanza hacia abajo otra piedra con una
detenerse justo cuando la manzana vuelve a pasar velocidad inicial de 4 m/s. La distancia entre cada
2
frente a l? (g=10 m/s ) piso es 7 m. Calcular al cabo de qu tiempo estarn
separadas las piedras 3 m. Dar como respuesta el
2 2 2 2
A) 1 m/s B) 2 m/s C) 3 m/s tiempo mnimo. (g = 10 m/s )
2 2
D) 4 m/s E) 5 m/s A) 1 s B) 2 s C) 3 s
D) 4 s E) 5 s

GRFICAS EN CINEMTICA
En el estudio de las magnitudes cinemticas es comn aceleracin y la posicin del mvil se pueden expresar en
encontrar una relacin entre dos o ms magnitudes, de tal funcin del tiempo y por lo tanto se pueden construir las
manera que si aumenta el valor de una de ellas, entonces grficas correspondientes.
cambia el valor de la otra (aumentando o disminuyendo),
por lo tanto se afirma que entre ellas existe una I. En el MRU
proporcin (directa o inversa) o una variacin lineal, 1. Grfica V - t (Velocidad versus tiempo)
cuadrtica, cbica, etc), en general se dice que una de En este caso la grfica es una lnea horizontal
ellas est en funcin de la otra. paralela al eje del tiempo, esto se debe a que la
Cuando una magnitud es funcin de otra, entonces se velocidad es constante y no depende del tiempo
puede construir una grfica que relacione a dichas transcurrido
magnitudes y para ello se emplean los ejes rectangulares
x - y, en cinemtica encontramos que la velocidad, la

-19-
ASOCIACIN EDUCATIVA PITGORAS Fsica

* Propiedad

II. En el MRUV
= rea
1. Grfica a - t (Aceleracin versus tiempo)
D = Desplazamiento En este caso la grfica es una lnea horizontal
paralela al eje del tiempo, esto se debe a que la
OBSERVACIN : aceleracin es constante y no depende del
(a) Primer cuadrante ! rea (+) tiempo transcurrido.
! desplazamiento hacia la derecha
(b) Cuarto cuadrante ! rea (-)
! desplazamiento hacia la izquierda

1. Grfica x - t (Posicin versus tiempo)


En este caso la grfica es una lnea recta
inclinada la cual no necesariamente pasa por el
origen de coordenadas, esto se debe a que el
mvil va cambiando de posicin durante el
transcurso del tiempo. * Propiedad

V = Vf - Vi = rea

2. Grfica V - t (Velocidad versus tiempo)


En este caso la grfica es una lnea recta
inclinada cuya pendiente puede ser positiva o
negativa, esto se debe a que la velocidad del
mvil va cambiando continuamente ya sea
aumentando o disminuyendo as como tambin
cambiando su direccin.
* Propiedad

V = Tg

Ejemplo :
Se presenta un mvil con MRU represente sus grficas
V-tyx-t

* Propiedad

a = Tg

Solucin :
D = rea
L Hallando la velocidad :
D = Desplazamiento
L Construyendo las grficas tenemos :
OBSERVACIONES :
L Si el mvil parte del reposo la grfica es :

-20-
ASOCIACIN EDUCATIVA PITGORAS Fsica

L Si el mvil desacelera la grfica es :

a = Tg Resolucin :

Pero: Tg = -Tg ! a = -Tg

3. Grfica x - t (Posicin versus tiempo)

En este caso la grfica es un arco de parbola


cuyo eje es vertical paralelo al eje de
coordenadas (x), si el mvil parte del reposo la
grfica es una semiparbola, cumplindose que
en cada punto de la grfica la pendiente nos da
la velocidad instantnea del mvil

L d = rea = A1 + |A2| + A3
d = 10(40) + |10(-5)| + 10(10)
d = 400 + 50 + 100 = 550 m
L D = rea = A1 + A2 + A3
D = 10(40) + 10(-5) + 10(10) = 450 m
02. Un mvil efecta dos MRUV consecutivos tal como se
indica en la grfica. Si la velocidad inicial es 10 m/s,
determine la velocidad final al cabo de los 20 s
iniciales

* Propiedad

V = Tg

OBSERVACIN : Resolucin :
L V = rea = A1 + A2
L Si el mvil parte del reposo la grfica es : ! Vf - 10 = 15(6) + (20 -15)(4) = 110 ! Vf = 120 m/s

03. La grfica mostrada corresponde a un MRUV,


determine el valor de la aceleracin del mvil

PROBLEMAS RESUELTOS
Resolucin :
01. Un mvil realiza tres MRU consecutivos tal como se L Datos : xo = 4 m; x = 29 m; t = 5 s; Vi = 0 m/s
representan en la grfica MRU mostrada, hallar la L Distancia recorrida : d = x - xo = 29 - 4 = 25 m
distancia recorrida en todo el movimiento y tambin el L Hallando la aceleracin :
desplazamiento 2
! a = 2 m/s

-21-
ASOCIACIN EDUCATIVA PITGORAS Fsica

PROBLEMAS PROPUESTOS
01. Dada la grfica x vs t, en qu relacin estn las 05. Hallar la velocidad media en el intervalo de t=4 s
velocidades en los tramos ? hasta t = 12 s

A) 1; 2 B) 2; 3 C) 3; 4
D) 1; 3 E) 4; 1

02. Se muestra la grfica V - t de dos partculas que se A) 2 m/s B) 5 m/s C) 7,5 m/s
mueven en el eje X. Despus de qu tiempo de D) 10 m/s E) 12,5 m/s
haber salido el mvil A sus velocidades se igualan?
06. Hallar el mdulo de la velocidad media y promedio
desde t = 0 hasta t = 10 s

A) 10 s B) 12 s C) 20 s
D) 24 s E) 30 s

03. El movimiento rectilneo de un mvil est


representado por la siguiente grfica x - t :

A) 8 m/s; 5 m/s B) 8 m/s; 4 m/s


C) 6 m/s; 5 m/s D) 5 m/s; 8 m/s
E) 10 m/s; 14 m/s

07. Hallar la posicin del mvil para t = 15 s, si para t = 0


su xo = -4 m, segn la grfica V - t

Se puede afirmar que :


I. El mvil se detiene dos veces en el trayecto
mostrado
II. El mvil siempre tiene velocidad positiva o nula
III. La velocidad en t = 4,5 s es 1 m/s

A) Slo I es verdadera B) Slo II es verdadera


C) I y II son verdaderas D) I y III son verdaderas A) 61 m B) 57 m C) 36 m
E) Todas son verdaderas D) 28 m E) 48 m

04. Del grfico mostrado hallar el mdulo de la 08. Un automvil a las 6 horas se encuentra en el
velocidad media y la velocidad promedio en el kilometro 600 de la Panamericana Norte, llegando a
intervalo [2; 8] segundo. Dar la respuesta en m/s. Lima a las 10 horas. A las 9 horas, cunto dista de
Lima?

A) B) 25; 50 C) 60; 80
A) 150 km B) 120 km C) 300 km
D) 100; 50 E) 30; 60 D) 450 km E) 480 km

-22-
ASOCIACIN EDUCATIVA PITGORAS Fsica

09. Un mvil se mueve en lnea recta con una velocidad A) 10 s B) 12 s C) 14 s


cuya dependencia del tiempo se muestra en la figura. D) 16 s E) 18 s
Qu distancia, en km, recorre en las primeras cuatro
horas? 14. Un mvil que se mueve en lnea recta describe un
MRUV, cuya grfica X - t se muestra. Hallar el rea
del tringulo sombreado

A) 40 B) 80 C) 160
D) 240 E) 320
10. En el grfico se muestran las velocidades en funcin
del tiempo de 3 mviles. Determinar la relacin entre A) 5 B) 4 C) 3
la aceleracin menor y la mayor D) 2 E) 1

15. Se muestra la grfica V - t de una partcula que se


mueve sobre el eje x. Hallar el mdulo del vector
desplazamiento

A) 4 B) 5 C) 2
D) 1/5 E) 1/4
11. Se muestra el grfico x-t de un automvil que se
desplaza en lnea recta. Hallar la velocidad A) 40 m B) 30 m C) 10 m
instantnea para t=3 D) 70 m E) 36 m

16. Dos mviles A y B parten simultneamente sus


posiciones iniciales son xA =-2 m y xB = +4 m.
Calcular en qu posicin se encuentran, si ambos se
mueven en la misma recta

A) 12 m/s B) 24 m/s C) 20 m/s


D) 14 m/s E) 10 m/s
12. Dos mviles parten de un mismo punto en la misma
direccin, describiendo las grficas mostradas en la
figura. Determinar el instante en que A alcanza a B

A) 2 s B) 4 s C) 6 s
D) 8 s E) 10 s

17. En el grfico a -t de un coche que se mueve en


el eje x. Hallar la velocidad para t=4, si para t=2 el
coche se encontraba moviendo en la direccin del eje
A) 10 s B) 12 s C) 15 s negativo con una rapidez de 8 m/s
D) 18 s E) 20 s
13. Dos mviles parten de un mismo punto. En qu
instante se encuentran?

A) 20 m/s B) -20 m/s C) 4 m/s


D) 8 m/s E) 24 m/s

-23-
ASOCIACIN EDUCATIVA PITGORAS Fsica

18. La grfica representa el MRUV de dos mviles. Si A) La velocidad media es nula


ambos llegan a encontrarse cuando sus velocidades B) El tiempo de recorrido es 4 s
son iguales, qu distancia los separaba C) No se puede conocer el tiempo de recorrido
inicialmente? D) La velocidad media es 8 m/s
2
E) La aceleracin es de 2 m/s

20. El grfico representa el movimiento de dos mviles.


Qu distancia los separa al cabo de 12 s?

A) 2 m B) 4 m C) 16 m
D) 8 m E) 24 m

19. La grfica corresponde al movimiento uniformemente


variado que realiza un cuerpo. En el intervalo desde
x = 0 hasta x=4 podemos afirmar :

A) 20 m B) 15 m C) 30 m
D) 10 m E) 25 m

TAREA
01. Calcular la distancia que recorre un mvil, que se
desplaza segn la grfica mostrada, durante el
dcimo segundo

A) De t = 0,2 h a t = 0,4 h el auto permanece parado


B) La distancia total recorrida por el vehculo fue
8 km
C) En el instante t = 0,6 (h) el auto estaba de regreso
A) 1 m B) 0,6 m C) 1,2 m en la posicin inicial
D) 2,4 m E) 4,8 m D) El auto recorri 4 km en un sentido y 4km, en
sentido contrario
02. En el movimiento rectilneo de un mvil se muestra la E) En el instante t = 0 el automvil se hallaba en el
grfica a - t. Si parte con -2 m/s en t = 0 s, qu kilmetro 20 y en el instante t = 0,6 h en el
velocidad tiene el mvil en t = 8 s? kilmetro -20

04. El grfico x - t representa el movimiento rectilneo de


dos mviles en lnea recta. Hallar el tiempo de
encuentro

A) -6 m/s B) 10 m/s C) -8 m/s


D) 12 m/s E) 14 m/s

03. El movimiento de un auto en una carretera se


representa en la figura de este problema. Entre las
afirmaciones siguientes, relativas al movimiento,
seale la que est equivocada : A) 2,5 s B) 2,8 s C) 2 s
D) 4 s E) 3 s

-24-
ASOCIACIN EDUCATIVA PITGORAS Fsica

05. En la figura se muestra el grfico posicin vs tiempo 08. Se tiene la grfica V vs t de un mvil que para
perteneciente a un mvil que se desplaza en lnea x = 2 el tiempo t = 0. Cul es su posicin para t = 2
recta. Determine si los enunciados son verdaderos y t = 5?
(V) o falsos (F) :
( ) El mvil se acerca al origen
( ) El mvil posee velocidad decreciente
( ) El mvil posee velocidad constante negativa

A) 18; 52 B) 22; 52 C) 18; 50


D) 22; 48 E) 12; 50

09. La grfica representa la velocidad de una partcula en


funcin del tiempo. Qu fraccin de su recorrido
A) VVF B) VFV C) FVV total efecta en los ltimos 4 segundos de su
D) FVF E) VFF movimiento?

06. Considerando el movimiento rectilneo de ida y vuelta


representado por la grfica de la figura, indicar
verdadero (V) y falso (F):

A) 56% B) 45% C) 32%


D) 28,1% E) 16%

( ) En la ida tiene una velocidad +4 m/s 10. A partir de la grfica V - t construir la grfica x - t, si
( ) En la vuelta su velocidad es -2 m/s se sabe que el coche se mueve sobre el eje X y
( ) La longitud recorrida en los 6 s es de 16 m desde el origen
A) VVV B) VVF C) VFV
D) VFF E) FFV

07. Una partcula que describe un movimiento rectilneo


a lo largo del eje X vara su posicin en funcin del
tiempo segn muestra la figura. Cul de las grficas
representar su velocidad en funcin del tiempo?

-25-
ASOCIACIN EDUCATIVA PITGORAS Fsica

MOVIMIENTO RELATIVO - MOVIMIENTO PARABLICO


MOVIMIENTO RELATIVO

Un cuerpo se encuentra en movimiento relativo respecto


a otro cuando su posicin respecto a este segundo cuerpo
cambia en el transcurso del tiempo. Por el contrario, si
dicha posicin permanece invariable se dice que los
cuerpos se encuentran en reposo relativo. As, por
ejemplo, la posicin de un pjaro en vuelo, o de un
hombre corriendo o de un automvil en marcha est
variando continuamente con respecto a la superficie
terrestre, y decimos entonces que dichos cuerpos se
mueven con relacin a la Tierra. Por el contrario un rbol
ACELERACIN RELATIVA ( ):
o una casa son cuerpos que mantienen una posicin
invariable respecto a la superficie terrestre y, por tanto, se
encuentran en reposo con relacin a la Tierra. La aceleracin relativa de un mvil A con respecto a un
mvil B se obtiene mediante la diferencia vectorial de
sus aceleraciones medidas con respecto a un mismo
sistema de referencia

NOTAS

Tanto el reposo como el movimiento tienen carcter 1. ;


relativo; es decir son estados que dependen de las
condiciones mutuas entre el cuerpo supuesto en reposo
o en movimiento y el cuerpo respecto al cual se refieren 2. Si : Y
estas propiedades.
Un asiento de un automvil se encuentra en reposo
3. Si : Y
respecto a ste pero en movimiento respecto a la
superficie terrestre. Por el contrario, un rbol y una casa
estn en reposo respecto a la Tierra, pero en movimiento
respecto al automvil 4. Si las posiciones de 2 cuerpos A y B son
conocidas en un cierto instante al igual que su
Concluimos, pues, que un mismo cuerpo puede velocidad relativa, siendo esta ltima constante,
encontrarse en reposo respecto a otro y, a la vez, en entonces la trayectoria relativa de A con respecto a
movimiento respecto a un tercero. Por consiguiente, al B es la lnea trazada desde su posicin inicial en la
analizar el movimiento de un cuerpo es necesario direccin de su velocidad relativa.
especificar con relacin a que otro cuerpo se refiere el
movimiento.
Convencionalmente se denomina movimiento absoluto al
movimiento efectuado por un mvil con respecto a un
sistema considerado fijo. El sistema que consideramos fijo
ser la Tierra a no ser que se diga la contrario.

VELOCIDAD RELATIVA
La velocidad relativa de un mvil A con respecto a un
mvil B se obtiene haciendo la diferencia vectorial de
sus velocidades medidas con respecto a un mismo
sistema de referencia

MOVIMIENTO COMPUESTO

Es aquel movimiento realizado por un mvil que est


formado por dos o ms movimientos elementales que
La velocidad nos representa la velocidad con la cual pueden ser MRU, MRUV, etc.
se desplaza el mvil A al tomar como sistema de Si los movimientos que forman el compuesto son
referencia al mvil B, es decir considerando que el mvil perpendiculares se realizan al mismo tiempo y en forma
B estuviera en reposo independiente.

-26-
ASOCIACIN EDUCATIVA PITGORAS Fsica

Ejemplo: Un barco cruza un ro MOVIMIENTO VERTICAL (CADA LIBRE)

(-): Movimiento subida


(+): Movimiento bajada

FRMULAS VECTORIALES
= Velocidad del barco respecto al ro
= Velocidad del ro
= Velocidad resultante del barco
Se cumple que los movimientos elementales y
son MRU y es el compuesto.
Luego: FRMULAS ADICIONALES
y = VB t
x = VR t
dPC = VT t
MOVIMIENTO PARABLICO
Es aquel movimiento cuya trayectoria es una curva
llamada parbola en donde la aceleracin es constante.
Si un cuerpo se lanza en forma inclinada o se lanza en
forma horizontal y se mueve cerca a la tierra
despreciando la resistencia del aire realiza un movimiento
parablico de cada libre en donde su aceleracin de la
gravedad es constante.
ste movimiento parablico es un movimiento compuesto 1. ALTURA MXIMA
formado por dos movimientos perpendiculares que se
realizan al mismo tiempo y en forma independiente, uno
de ellos en un MRU horizontal y el otro vertical de cada
libre.

2. TIEMPO DE VUELO (Tabc)

3. ALCANCE HORIZONTAL (R)

tAB = tBC

FRMULAS

Como Sen2 = 2SenCos

4.

MOVIMIENTO HORIZONTAL (MRU)

-27-
ASOCIACIN EDUCATIVA PITGORAS Fsica

5. ECUACIN DE LA TRAYECTORIA Sabiendo:

Reemplazando:

x = Vxt = Vo Cost

Luego:

PROBLEMAS PROPUESTOS
01. Dos coches P y Q se mueven con velocidades de 04. Dos argollas se desplazan por dos carriles que se
10 m/s y 14 m/s segn la figura. Hallar las cortan en ngulo recto con velocidades constantes
velocidades de cada uno respecto de R que viaja a VA = 12 m/s y VB = 9 cm/s. A partir de las posiciones
razn de 8 m/s mostradas se pide calcular :
A. La distancia mnima a la que se encontrarn los
cuerpos durante sus movimientos
B. El tiempo necesario para que se presente la
distancia mnima

A) 90 cm; 8 s B) 90 cm; 4 s C) 120 cm; 4 s


A) 6 m/s (6); 16 m/s (7) D) 120 cm; 8 s E) 90 cm; 12 s
B) 2 m/s (6); 22 m/s (7)
C) 1 m/s (6); 10 m/s (7) 05. Un tren de longitud 169 m se mueve con velocidad
D) 18 m/s (6); 6 m/s (7) constante de 18 m/s y en direccin opuesta un auto
E) 2 m/s (6); 6 m/s (7) de longitud 3 m con velocidad constante de 25 m/s.
Hallar la velocidad relativa del auto respecto al tren y
02. Dos coches P y Q se desplazan uniformemente en la al tiempo que emplean en cruzarse.
misma direccin (P detrs de Q) con velocidades VP
= 10 m/s y VQ = 30 m/s, separados inicialmente una
distancia d = 40 m. Se pide calcular :
A. La velocidad de Q respecto a P
B. El tiempo que emplean para estar separados una
distancia 3d
A) 20 m/s; 4 s B) 20 m/s; 6 s C) 10 m/s; 2 s
D) 20 m/s; 8 s E) 5 m/s; 8 s

03. Dado los coches A y B con velocidades constantes A) 27 m/s (7); 3 s B) 19 m/s (6); 1 s
de 80 m/s y 70 m/s respectivamente, hallar el tiempo C) 7 m/s (7); 2 s D) 27 m/s (6); 4 s
que emplean los mviles para que su distancia de E) 43 m/s (7); 4 s
separacin sea de 290 m, si la inicial era de 160 m
06. Un automvil sube por una pendiente inclinada 30
con la horizontal con una aceleracin constante de
2
6 m/s . Si el conductor ve caer una gota de lluvia
delante de l, cul es la aceleracin relativa de la
2
gota para el conductor? g = 10 m/s
2 2 2
A) 14 m/s B) 7 m/s C) 8 m/s
A) 5 s B) 4 s C) 3 s D) 9 m/s
2
E) 16 m/s
2
D) 2 s E) 1 s

-28-
ASOCIACIN EDUCATIVA PITGORAS Fsica

07. Dos mviles A y B estn sobre la misma recta y 12. Desde una boya, que se encuentra en medio de un
parten del reposo simultneamente yendo al ancho ro, partieron los botes A y B. Los botes
2
encuentro con aceleraciones constantes de 1 m/s y tomaron direcciones perpendiculares entre s; el bote
2
2 m/s desde una distancia de 150 m. Halle la A, a lo largo del ro, y el bote B, a lo ancho.
aceleracin relativa de A respecto a B y el tiempo que Habindose separado a una misma distancia de la
tardan en encontrarse boya, los botes emprendieron el regreso. Hallar la
relacin entre el tiempo consumido por cada bote
2
A) 1 m/s y 17,3 s TA /TB , si la velocidad de cada uno de ellos supera
2
B) 3 m/s y 10 s 1,25 veces a la del ro
2
C) 2 m/s ; 15 s
2
D) 1 m/s y 10 s A) 5/3 B) 5/7 C) 4/3
2
E) 1 m/s y 20 s D) 7/4 E) 8/7

08. Dos mviles parten en direcciones perpendiculares 13. La velocidad de un bote respecto a tierra es de 5 m/s
de un mismo punto con velocidades de 14 m/s y y la velocidad de la corriente del ro es de 4 m/s.
48 m/s. Despus de cunto tiempo se encontrarn Hallar la distancia aguas abajo a la cual llegar a la
separados una distancia de 400 m? orilla opuesta cuando el bote atraviesa el ro de 54 m
de ancho. La velocidad del bote respecto al ro es
perpendicular a las orillas

A) 43,2 m B) 90 m C) 72 m
D) 80 m E) N.A
A) 4 s B) 10 s C) 8 s 14. Un avin caza, que vuela horizontalmente, con una
D) 16 s E) 2 s rapidez de 250 m/s, tal como se muestra, suelta una
2 bomba en A y simultneamente se dispara desde B
09. Un leopardo africano cuya aceleracin es 8 m/s un proyectil con direccin horizontal, impactando a la
2
persigue a una gacela cuya aceleracin es 5 m/s y bomba a 500 m de altura. Si las velocidades de
est ubicada a 150 m de l. Si ambos parten del impacto son perpendiculares, con qu rapidez se
reposo simultneamente, calcular el tiempo que tarda dispar el proyectil?. Desprecie los efectos del aire (g
el leopardo en atrapar la gacela 2
= 10 m/s )
A) 2 s B) 3 s C) 4 s
D) 9 s E) 10 s

10. Dos bolitas A y B pasan simultneamente por los


puntos P y Q con velocidades VA = 6 m/s y
VB =6 m/s que lo mantienen constantes. Calcular el
mnimo acercamiento entre A y B

A) 55 m/s B) 40 m/s C) 60 m/s


D) 250 m/s E) 50 m/s

15. Se lanza un cuerpo desde el punto A con una


velocidad . Despreciando la
2
resistencia del aire, calcular d y H. (g = 10 m/s )

A) 10 m B) 12 m C) 18 m
D) 24 m E) 36 m

11. Una lancha navegando ro abajo, dej atrs una


balsa en el punto A. Transcurridos T=1 h, la lancha
dio la vuelta y volva a encontrar la balsa a una
distancia L=6 km ms abajo del punto A. Hallar la
velocidad de la corriente en ambas direcciones, si a
lo largo del recorrido de la lancha el motor trabaj por A) 200 m; 150 m
igual B) 120 m ; 60 m
C) 160 m; 80 m
A) 3 km/h B) 2 km/h C) 6 km/h D) 155 m; 60 m
D) 4,5 km/h E) 1,5 km/h E) 140 m; 80 m

16. En el grfico se muestra la trayectoria parablica de

-29-
ASOCIACIN EDUCATIVA PITGORAS Fsica

un proyectil en un plano vertical, calcular el tiempo A) 6,5 s B) 6 s C) 6,25 s


que transcurre desde A hasta B. La velocidad en D) 7 s E) 5,6 s
2
A es 15 m/s, considere (g = 10 m/s )
19. A qu distancia del punto de lanzamiento, la esfera
chocar con el piso por primera vez? Desprecie la
resistencia del aire y considere que el primer choque
2
es elstico. (g = 10 m/s )

A) 2,2 s B) 1,8 s C) 3 s
D) 2,8 s E) 2,5 s

17. Una piedra se lanza de un edificio a otro con la


velocidad de 10 m/s, logrando impactar, formando un A) 160 m B) 200 m C) 150 m
ngulo de 45 con la horizontal. Hallar la separacin D) 240 m E) 180 m
entre los edificios
2
(g = 10 m/s ) 20. En el grfico mostrado, si el cuerpo A se lanza 3 s
antes que B. Calcular desde qu altura fue lanzado A,
si colisiona con el cuerpo B cuando ste alcanza su
altura mxima. Desprecie la resistencia del aire y
2
considere (g = 10 m/s )

A) 8,4 m B) 11,2 m C) 14, 6 m


D) 16,1 m E) 6,4 m
A) 120 m B) 130 m C) 90 m
18. Una partcula es lanzada perpendicularmente a un D) 115 m E) 100 m
plano inclinado tal como se muestra. Determine el
tiempo que debe pasar para que impacte en el plano.
No considere la oposicin del aire.
2
(g = 10 m/s )

TAREA
01. Dos trenes de 200 y 400 metros de longitud avanzan
en vas paralelas en sentidos opuestos y cuando se
encuentran sus velocidades son 12 m/s y 18 m/s y
2
sus aceleraciones de 3 m/s cada una. Hallar el
tiempo que demoran en cruzarse completamente
(considere MRUV)
A) 5 s B) 10 s C) 15 s
D) 20 s E) 25 s

02. Un auto viaja a razn de 90 km/h y lo hace


paralelamente a un tren de pasajeros que viaja en el
mismo sentido a razn de 72 km/h. Cunto tiempo A) 22,5 segundos B) 20 segundos
emplear el auto en adelantar al tren? C) 18 segundos D) 25 segundos
E) 23 segundos

03. Dos mviles 1 y 2 salen simultneamente de un punto

-30-
ASOCIACIN EDUCATIVA PITGORAS Fsica

O con velocidades constantes V1 = 14 cm/s y 08. Si en el movimiento parablico mostrado, tenemos


V2=50 cm/s, cuyas direcciones forman un ngulo de que: tiempo (A 6 B) = 2 . Tiempo (B6C). Entonces es
74. Calcular : verdadero (V) o falso (F)
A. La velocidad relativa de 1 respecto a 2 ( ) El tiempo de vuelo es cuatro veces el tiempo
B. El tiempo despus del cual la distancia relativa desde C hasta D
entre ambos sea 960 cm ( ) La velocidad horizontal Vx es menor que V
A) 30 cm/s; 40 s B) 48 cm/s; 20 s ( ) La longitud a es el doble de la longitud b
C) 50 cm/s; 25 s D) 40 cm/s; 30 s
E) 24 cm/s; 40 s

04. Un barco en el ecuador navega hacia el este con una


velocidad Vo = 7 m/s. Desde el sureste hacia el
ecuador sopla un viento con una velocidad V=15 m/s
formando un ngulo = 53 con el ecuador. Hallar la
velocidad V' del viento respecto al barco y el ngulo
' entre el ecuador y la direccin del viento en el
sistema de referencia ligado con el barco
A) FVV B) VVV C) VVF
A) 20 m/s; 37 B) 20 m/s; 143
D) FFV E) FFF
C) 20 m/s; 53 D) 20 m/s; 45
E) 20 m/s; 127
09. Dos partculas A y B se lanzan desde las posiciones
mostradas. Considerando despreciable los efectos
05. En la figura se dan dos mviles A y B con
del aire y que 45 < < < 90, indique verdadero
velocidades relativas conocidas :
(V) o falso (F) :
VA/T = 6 m/s; VB/A = 3 m/s
( ) A llega ms lejos y B permanece ms tiempo en
Segn las direcciones indicadas, se pide encontrar la
vuelo
velocidad de B respecto de T (tierra)
( ) Las partculas alcanzan igual altura mxima
( ) A alcanza mayor altura que B

A) FVV B) VVV C) FFF


A) 3 m/s (6) B) 6 m/s (6) C) 3 m/s (7) D) VFV E) FFV
D) 6 m/s (7) E) Cero
10. Una pelota es lanzada con un velocidad inicial Vo,
06. Respecto al movimiento parablico de un cuerpo formando un ngulo con la horizontal. El tiempo que
sometido solamente a la accin de la gravedad. tarda la pelota en ir de la posicin A a la posicin C
Indicar verdadero (V) o falso (F): es (sin considerar los efectos del aire):
( ) Horizontalmente tenemos un movimiento uniforme
( ) La distancia que se desplaza horizontalmente es
proporcional al tiempo transcurrido
( ) No presenta aceleracin
A) FFF B) VVF C) FFV
D) VVV E) VFV

07. La trayectoria parablica mostrada es la de un cuerpo


que fue lanzado con una velocidad V (su mdulo).
Indicar verdadero (V) o falso (F):
( ) En A y en B las velocidades son iguales A) La mitad del tiempo entre O y B
( ) La rapidez en A es igual a la rapidez en B B) Igual al tiempo entre O y A
( ) En el punto ms alto (C) la velocidad es nula C) Igual al tiempo entre B y D
D) La mitad del tiempo entre B y D
E) No se puede afirmar nada

A) VVV B) FFV C) FVF


D) FFF E) VVF

-31-
ASOCIACIN EDUCATIVA PITGORAS Fsica

MOVIMIENTO CIRCULAR - MCU - MCUV


CONCEPTO PERIODO (T) : Tiempo empleado por el mvil con MCU
Es el movimiento de trayectoria circular en donde el valor en efectuar una vuelta o revolucin (describir 2 rad)
de la velocidad del mvil se mantiene constante en todo
instante, se recorren en la circunferencia distancias FRECUENCIA (f) : Magnitud fsica escalar que indica el
iguales en tiempos iguales y tambin se describen nmero de vueltas (revoluciones) efectuadas por el mvil
ngulos centrales iguales en tiempos iguales. con MCU en la unidad de tiempo. Se determina mediante
la inversa del periodo

donde:
N = # de revoluciones
t = Tiempo transcurrido
* Unidad : herzt (Hz) =

VELOCIDAD TANGENCIAL O LINEAL ( ): Es la


* Equivalencias :
velocidad instantnea del MCU, su valor nos indica la
longitud de circunferencia recorrida en la unidad de 1Hz < >
tiempo y es tangente a la circunferencia de trayectoria

VELOCIDAD ANGULAR ( ): Es La magnitud fsica


vectorial que nos indica la rapidez y direccin del ngulo
central descrito. Su direccin se determina mediante la ACELERACIN CENTRPETA ( ): Es la aceleracin
regla de la Mano Derecha que posee todo cuerpo con MCU, est relacionada con el
cambio de direccin de la velocidad tangencial y est
dirigida hacia el centro de la trayectoria circular.

pero: V = r

MOVIMIENTO CIRCULAR UNIFORMEMENTE VARIADO


(M.C.U.V)

CONCEPTOS PREVIOS

1. ACELERACIN TANGENCIAL O LINEAL ( )


Si un cuerpo se desplaza por una curva y el valor o
mdulo de su velocidad tangencial cambia, entonces
aparece la aceleracin tangencial cuya direccin ser
LEYES DEL MCU tangente a la circunferencia y su sentido coincidir con
el de la velocidad tangencial si el movimiento es
P | | = Constante acelerado y ser de sentido opuesto a ella, si el
P = Constante movimiento es desacelerado.
P d es directamente proporcional a t Unidades : ; etc
P es directamente proporcional a t

ECUACIONES DEL MCU

Movimiento acelerado

-32-
ASOCIACIN EDUCATIVA PITGORAS Fsica

Por el teorema de Pitgoras

Caractersticas del M.C.U.V.

1. = constante; constante
Movimiento desacelerado 2. = constante; = constante
3. constante; constante
2. ACELERACIN ANGULAR ( )
Si un cuerpo se desplaza por una curva y su velocidad 4. En tiempos iguales la rapidez tangencial V cambia
angular cambia, entonces aparece la aceleracin cantidades iguales
angular cuya direccin es perpendicular al plano de 5. En tiempos iguales la rapidez angular cambia
rotacin y su sentido coincidir con el de la velocidad cantidades iguales
angular si el movimiento es acelerado y ser de 6. En tiempos iguales recorre arcos diferentes y realiza
sentido opuesto a ella si el movimiento es desplazamientos angulares diferentes.
desacelerado
FRMULAS
Unidades :
I. TANGENCIALES
; etc

* Este grfico es de un M.C.U.V ........................


Movimiento acelerado
1. Vf = Vi aT . t 2.

2
3. S = Vi.t aT . t 4. Sn = Vi aT (2n - 1)

Sn = arco recorrido en el nmero de segundo n


(n - simo segundo)

(+) Movimiento acelerado


Movimiento desacelerado
(-) Movimiento desacelerado
3. ACELERACIN ( )
Se denomina as a la resultante de la aceleracin Adems :
tangencial con la aceleracin centrpeta, tambin se le
denomina aceleracin instantnea

II. ANGULARES

Movimiento acelerado

* Este grfico es de un M.C.U.V . ....................


Movimiento desacelerado

-33-
ASOCIACIN EDUCATIVA PITGORAS Fsica

1. f = i t 2. Disco o rueda
2.
2
3. = it t

4. n = i (2n - 1)

n : ngulo descrito en el nmero de segundo n


Para las ruedas :

(+) : Movimiento acelerado A = B

(-) : Movimiento desacelerado 3.

Adems :

Relacin entre la Aceleracin tangencial aT y la


aceleracin angular VBloque = R

aT =

VBloque = VRueda
aT = R
Adems : aBloque = aTRueda
PROPIEDADES DE LA TRANSMISIN DE
MOVIMIENTOS :
aBloque = R
1.
4.

Adems :

VA = VB Puntos perifricos

Puntos perifricos

-34-
ASOCIACIN EDUCATIVA PITGORAS Fsica

PROBLEMAS PROPUESTOS
01. Una piedra de amolar, rota con una velocidad angular
constante. Un punto ubicado a 2 cm de la periferia,
medidos en la direccin radial, posee una velocidad,
tangencial, 1/5 menor que la que posee un punto
ubicado en la periferia. Cul es el radio de la piedra?
A) 390 m/s B) 395 m/s C) 400 m/s
A) 2,5 cm B) 5,5 cm C) 10 cm D) 405 m/s E) 410 m/s
D) 12 cm E) 14 cm
06. Si el bloque tiene que bajar a velocidad constante de
02. El tronco de cono mostrado est girando en torno al 16 m/s, cul debe ser la velocidad angular con qu
eje y-y. Hallar la relacin en que se encuentran las debe girar la rueda C?
velocidades lineales de los puntos A y B, si el (RA=8 cm; RB =15 cm; RC =25 cm)
periodo es 5 segundos

A) 105 rad/s B) 108 rad/s C) 120 rad/s


D) 218 rad/s E) 311 rad/s

A) 0,5 B) 1 C) 2 07. Calcular la aceleracin angular que tiene un disco,


D) 3 E) 4 sabiendo que es capaz de triplicar su velocidad luego
de realizar 600 vueltas en 20 s
2
03. Se dispara una bala con una velocidad V=200 m/s A) rad/s B) 2 C) 3
contra un cascarn esfrico de papel que gira con D) 4 E) 5
movimiento uniforme respecto a un eje vertical.
Sabiendo que el radio del cascarn es 2 m, calcular 08. Un disco de 300 cm de radio, parte del reposo con
con qu velocidad angular mnima deber girar el M.C.U.V. y luego de 16 s su velocidad es 20 rad/s.
cascarn para que el proyectil haga un solo agujero. Qu arco habr recorrido en dicho tiempo?
La direccin del movimiento de la bala pasa por el A) 160 m B) 320 m C) 480 m
centro de la esfera. D) 600 m E) 80 m
2
09. Una rueda acelera a razn de 2 rad/s y gira un
ngulo de 75 rad en 5 s. Cunto tiempo ha estado
en movimiento antes de comenzar el intervalo de 5 s,
si parti del reposo?
A) 6 s B) 5 s C) 4 s
D) 3 s E) 2 s

10. Una volante empieza a girar desde el reposo. Si al


A) 2 rad/s B) 4 rad/s C) 6 rad/s cabo de 10 s tiene una velocidad de 180 RPM,
D) 8 rad/s E) 10 rad/s cuntas vueltas habr girado?
A) 50 B) 40 C) 30
04. Dos partculas parten simultneamente de los D) 25 E) 15
extremos de un dimetro AB y en los
sentidos indicados en la figura. Si giran con periodos 11. Una partcula se mueve en torno a una circunferencia
TA = 20 s y TB = 30 s respectivamente, calcular al con M.C.U.V. partiendo del reposo, si tarda 2 minutos
cabo de qu tiempo logran cruzarse por segunda vez en recorrer entre 2 puntos de la trayectoria un
desplazamiento angular de 24 rev. Si cuando pasa
por el segundo punto lo hace a razn de 18 R.P.M.
Hallar el nmero de vueltas entre el primer punto y el
punto de partida
A) 2 B) 3 C) 4
D) 5 E) 6

A) 12 s B) 14 s C) 16 s 12. Un ciclista corre por un veldromo con M.C.U.V. de


D) 18 s E) 20 s tal modo que al cabo de 5 s su velocidad lineal es de
15 m/s. Se observa tambin que durante dicho tiempo
05. En la figura el cilindro gira a razn de el ciclista logr girar un ngulo central de 2 rad/s,
180 RPM, el cilindro es hueco de 3 m de largo. Si se siendo el radio de la pista igual a 25 m. Calcular la
dispara un proyectil por una de las bases, perfora a la velocidad lineal que tena al inic iar su movimiento.
otra base luego de que el cilindro ha girado B, hallar A) 5 m/s B) 10 m/s C) 15 m/s
la velocidad de la bala. D) 20 m/s E) 25 m/s

-35-
ASOCIACIN EDUCATIVA PITGORAS Fsica

13. Desde el reposo se da la partida de una partcula con 18. Un cilindro presenta movimiento de rotacin y
M.C.U.V., hallar la velocidad lineal del mvil luego de traslacin, si la velocidad en A es 20 m/s, calcular la
2 s de movimiento, si en ese instante, la aceleracin velocidad en B.
2
normal es m/s y forma 30 con la aceleracin
lineal

A) 2 m/s B) 4 m/s C) 6 m/s


D) 8 m/s E) 10 m/s

14. En cierto instante la aceleracin de un mvil con


2
M.C.U.V. mide 5 m/s y forma 127 con la velocidad
lineal. Hallar la velocidad del mvil 2 s despus de
este momento. El radio del trayecto circular es 16 m
A) 2 m/s B) 3 m/s C) 4 m/s
D) 5 m/s E) 6 m/s

15. Dos mviles parten simultneamente y desde el A) 10 m/s B) 10 m/s C) 20 m/s


reposo con aceleraciones angulares constantes de D) 20 m/s E) 5 m/s
2 2
/9 rad/s y 2/9 rad/s . Cunto tardarn en
encontrarse? 19. En el instante mostrado, se observa la rueda
delantera de un automvil. La velocidad de A
es 4 m/s, calcular la velocidad del punto B.

A) 3 s B) 4 s C) 5 s
D) 6 s E) 7 s

16. Hallar la velocidad angular de la rueda B despus


de 5 s de iniciado el movimiento sabiendo que la
2
rueda A posee una aceleracin angular de 3 rad/s .
A) m/s B) m/s C) m/s
RA = 2 m; RB = 0,5 m
D) 2 m/s E) 5 m/s

20. Una rueda de radio (R = 1,6) m, rueda


uniformemente por una superficie horizontal. Del
punto A de la rueda se desprende una gota de
barro. Con qu rapidez se traslada la rueda, si la
gota despus de estar en el aire vuelve a caer sobre
el mismo punto, luego que la rueda dio 4 v ueltas?.
2
(g = 10 m/s )
A) 12 rad/s B) 24 rad/s C) 30 rad/s
D) 42 rad/s E) 60 rad/s

17. El disco A posee una aceleracin angular de


2
10 rad/s . Calcular la velocidad del bloque Q
despus de 15 s de iniciado el movimiento
RA = 30 cm.
RB = 50 cm; RC = 20 cm
A) 4 m/s B) 5 m/s C) 8 m/s
D) 10 m/s E) 16 m/s

A) 12 m/s B) 15 m/s C) 18 m/s


D) 20 m/s E) 24 m/s

-36-
ASOCIACIN EDUCATIVA PITGORAS Fsica

TAREA
01. Los radios de una polea compuesta son : r y 2r en 07. Desde qu altura se debe dejar caer la piedra para
el instante mostrado est girando con 0,4 rad/s. En que pase por el agujero cuando, el disco genere 3
cunto tiempo ms los bloques A y B estarn a la vueltas. Velocidad angular del disco. 6 rad/s
2
misma altura? (g=10 m/s )

A) 2 m B) 5 m C) 3 m
A) 3,3s B) 4,3s C) 5,3s D) 1,5 m E) 2,5 m
D) 6,3s E) 7,3s
08. Una partcula gira con MCUV, para el instante
02. La esferita mostrada gira uniformemente a razn de indicado, su velocidad y aceleracin son 20 m/s y
2
120 RPM, si la cuerda que lo sostiene tiene una 20 m/s . Determinar su velocidad y aceleracin
longitud de 1 m, qu velocidad lineal tiene la angular.
esferita?

2
A) 0,8 rad/s y 0,48 rad/s
A) 2,28 m/s B) 3,14 m/s C) 4,71 m/s B) 0,8 y 0,96 rad/s
2
D) 5,28 m/s E) 6,28 m/s C) 0,4 y 1,24 rad/s
2

D) 0,6 y 0,8 rad/s2


03. Un volante parte del reposo, gira con MCUV E) 1,2 y 2,4 rad/s 2
2
10 rad/s . Si en 2 s gira 180 rad, qu tiempo
transcurri del inicio hasta el lapso de los 2 s? 09. Si el eje que une a los discos A y B giran con
A) 2 s B) 4 s C) 6 s =12rad/s, calcular la velocidad del disco C.
D) 8 s E) 10 s RA = 2 m RB = 3 m RC = 4 m
04. Un cuerpo con MCUV, parti con una velocidad
angular de 4 rad/s y una aceleracin angular de
2
3 rad/s al transcurrir los primeros 6 s. Hallar su
velocidad angular

A) 660 RPM B) 330 RPM C) 220 RPM


D) 110 RPM E) 66 RPM
2
05. La aceleracin angular de una rueda es 2 rad/s .
Al cabo de 0,5 s de iniciado el movimiento su
2
aceleracin instantnea es de m/s . Si parti del
reposo, hallar el radio de la rueda A) 12 m/s B) 4 m/s C) 3 m/s
D) 36 m/s E) 24 m/s
A) 1 m B) 2 m C) 3 m
D) m E) 4 m 10. Una partcula realiza un MCUV con una aceleracin
angular constante de 3 rad/s2 y un radio de giro de
06. Desde una altura de 5 m se suelta una piedra sobre 5 m, hallar la rapidez del mvil en el instante que su
un punto P perteneciente a la periferie del disco; la velocidad y aceleracin formen 53
piedra es soltada cuando el disco comienza a girar y A) 6 m/s B) 12 m/s C) 8 m/s
cuando P tiene 900 R.P.M. Qu distancia separa D) 14 m/s E) 10 m/s
al punto P y la piedra cuando sta choca con el
disco?
2
Radio del disco = 20 cm; g = 10 m/s

A) 50 cm B) 40 cm C) 30 cm
D) 20 cm E) 10 cm

-37-
ASOCIACIN EDUCATIVA PITGORAS Fsica

ESTTICA
INTRODUCCIN

La mecnica clsica se basa en tres leyes fundamentales


que expres por primera vez Sir Isaac Newton en 1686,
en sus Philosaphiae Naturalis Principia Matemtica (Los
Fundamentos Matemticos de la Ciencia de la
Naturaleza). No debe creerse, sin embargo, que la
mecnica como ciencia, comenz con Newton. Muchos le
haban precedido en estos estudios, siendo quizs el ms
destacado Galileo Galilei, quien en sus trabajos sobre el
movimiento acelerado haba establecido una gran parte
de los fundamentos utilizados por Newton para la
formulacin de sus tres leyes.
Las leyes de Newton no pueden reducirse F = Kx
matemticamente de ninguna manera, son
generalizaciones de observaciones experimentales del F : Fuerza deformadora
movimiento real de los cuerpos materiales y de cmo las K : Constante de rigidez (depende del tipo de
fuerzas aplicadas afectan a esos movimientos. En material)
consecuencia, son leyes naturales que describen el x : elongacin
comportamiento del mundo externo. L : Longitud natural (sin deformar)
En este captulo slo utilizaremos la Primera y Tercera
Ley de Newton. La Segunda Ley se estudiar en NATURALEZA DE LAS FUERZAS
Dinmica.
Todas las interacciones se agrupan en tres tipos de
CONCEPTO : fuerzas :

Es la parte de la mecnica que se encarga de estudiar a 1. FUERZA GRAVITACIONAL


los cuerpos que se encuentran en equilibrio.
Es la fuerza de atraccin entre dos cuerpos debido a
EQUILIBRIO : sus respectivas masas, esta fuerza es muy dbil, y
para sentir su efecto es necesario que por lo menos
Un cuerpo se encuentra en equilibrio cuando no tiene uno de los cuerpos tenga una masa muy grande
aceleracin, por lo tanto slo hay 2 posibilidades: est en como la del Sol o de los planetas.
reposo o se mueve en lnea recta con velocidad constante

Equilibrio :

FUERZA

Cuando suspendemos un cuerpo, golpeamos un clavo,


estiramos o comprimimos un resorte, empujamos un
automvil o limpiamos una ventana de vidrio, decimos que
estamos interaccionando; la interaccin es pues jalar o
EL PESO : de los cuerpos es una fuerza
empujar los dems cuerpos, entonces :
gravitacional y se debe a que la masa de la Tierra (M)
atrae la masa (m) de los cuerpos.
La fuerza es la medida de la interaccin que se
manifiesta entre dos cuerpos
W = mg
Cabe recalcar que esta interaccin puede ser por contacto
o a distancia
W : Peso del cuerpo
Su unidad en el S.I. es : .................................... m : masa del cuerpo
g : aceleracin de la gravedad
MEDICIN ESTTICA DE LA FUERZA
* El peso es un vector que siempre apunta hacia el
Consideremos el resorte en espiral de longitud (L) que se centro de la Tierra y puede variar de un lugar a
muestra en la figura, en el extremo de este resorte otro ya que depende de la aceleracin de la
apliquemos una fuerza (F) vertical hacia abajo, gravedad (g)
observaremos un aumento (x) en la longitud directamente
proporcional a la fuerza aplicada. 2. FUERZA ELECTROMAGNTICA

Robert Hooke fue el primero que estableci esta relacin Se descompone en :


mediante el invento de un resorte compensador para un FUERZA ELCTRICA : Es la fuerza de atraccin o
reloj. repulsin entre dos cuerpos debido a que ambos
La ley de HOOKE se escribe como : poseen cargas elctricas,

-38-
ASOCIACIN EDUCATIVA PITGORAS Fsica

FUERZA MAGNTICA : Es una fuerza adicional a la 3. FUERZA NORMAL (N)


fuerza elctrica cuando las cargas elctricas estn en
Consideremos un cuerpo sobre una superficie plana
movimiento.

3. FUERZAS NUCLEARES
Son fuerzas que aparecen cuando la distancia entre
-15
los cuerpos es menor que 10 m y desaparecen
cuando esta distancia aumenta, luego son fuerzas de
corto rango.
Estas fuerzas explican porque las partculas dentro
del ncleo del tomo se mantienen unidas.

Todas las diferentes fuerzas que se manifiestan * Debido al contacto las molculas inferiores del
en la naturaleza son de origen gravitacional, cuerpo se comprimen (acercan).
electromagntico o nuclear.
* En el contacto aparece una fuerza NORMAL (N)
para contrarrestar el acercamiento molecular.
FUERZAS USUALES USADAS EN ESTTICA
* Separando imaginariamente el cuerpo de la
1. TENSIN (T) EN UNA CUERDA superficie plana representamos la fuerza normal
(N) la cual siempre ingresa al cuerpo en forma
Tomemos una cuerda fija en el punto B y jalada perpendicular al contacto.
desde el otro extremo A mediante una fuerza F
Las fuerzas de tensin (T), compresin (C) normal
(N) son moleculares y por tanto de naturaleza
electromagntica.

LEYES DE NEWTON

* Debido a la fuerza F las molculas de la cuerda 1RA. LEY (LEY DE LA INERCIA)


se separan
* Para contrarrestar esta separacin molecular La primera ley de Newton o ley de la inercia fue
aparece una fuerza de restitucin, llamada enunciada en el ao 1787 y establece que :
TENSIN (T) la cual se opone a la fuerza exterior
F.
* Separando imaginariamente la porcin MA de la
cuerda observamos que la tensin (T) se opone a Todo cuerpo contina en su estado de REPOSO o
la fuerza exterior F, ya que en el punto M las de movimiento a velocidad CONSTANTE mientras
molculas se separan. que sobre el cuerpo no acte una fuerza resultante
EXTERIOR que lo obligue a cambiar de velocidad
2. COMPRESIN (C) EN UNA BARRA :
La tendencia que tiene un cuerpo de mantener su
Tomemos una barra apoyada en el punto B y en el estado de reposo o de movimiento a velocidad
otro extremo A apliquemos una fuerza F que constante se llama INERCIA.
comprime la barra
INTERPRETACIONES DE LA LEY DE LA INERCIA
Ejemplo 01 :

* Debido a la fuerza F las molculas de la barra se


acercan
* Para contrarrestar este acercamiento molecular
aparece una fuerza de restitucin, llamada
COMPRESIN (C) la cual se opone a la fuerza
exterior F.
* Separando imaginariamente una porcin MA de la
barra observamos que la fuerza de compresin Cuando tiramos hbilmente el mantel de la mesa
(C) se opone a la fuerza exterior F, por que en el observamos que los utensilios (copas, botella, tazn)
punto M las molculas se acercan tienden a permanecer en reposo.

-39-
ASOCIACIN EDUCATIVA PITGORAS Fsica

Ejemplo 02 : Ejemplo 02 :

Si un caballo se detiene de golpe, el jinete sale En el lanzamiento de un cohete, ste ejerce una gran
expelido por encima, por que todo cuerpo en fuerza de accin sobre los gases, para
movimiento, por inercia, tiende a seguir en expulsarlos, y los gases ejercen una fuerza igual y
movimiento.
opuesta de reaccin sobre el cohete que lo
impulsa hacia arriba.

Ejemplo 03 :

Algunas veces si no disminuimos la velocidad del


auto, ste puede salirse de la carretera en la curva ya
que por la ley de la inercia el auto trata de conservar
su velocidad constante (en lnea recta)

Ejemplo 03 :

Al clavar con un martillo, ste impulsa al clavo, hacia


abajo (accin) y el clavo reacciona sobre el martillo
detenindolo e inclusive hasta hacerlo rebotar.

: Velocidad excesiva

3RA. LEY (LEY DE LA ACCIN Y REACCIN)


(Tercera Ley de Newton)

Descubierta por Isaac Newton y publicada en el


mismo ao que la ley anterior, establece que : La accin y reaccin actan en cuerpos diferentes. La
accin sobre el clavo y la reaccin sobre el martillo.
Siempre que un objeto ejerce una fuerza (ACCIN)
sobre otro objeto, el segundo ejerce una fuerza igual
(REACCIN) y opuesta sobre el primero. 1RA. CONDICIN DE EQUILIBRIO

Si un cuerpo se encuentra en equilibrio entonces la


La accin y la reaccin actan sobre objetos fuerza resultante que acta sobre l es igual a cero
diferentes. La accin sobre uno de los cuerpos y la
reaccin sobre el otro cuerpo, por esto nunca se
anulan.

INTERPRETACIONES DE LA LEY DE ACCIN Y


REACCIN Si sobre un cuerpo en equilibrio (m) actan 3 fuerzas,
stas deben ser concurrentes, coplanares o paralelas.
Ejemplo 01 Ejemplo :

Cuando un patinador empuja el pasamanos (accin


) ste reacciona y la fuerza de reaccin ( ) hace
que l se aparte.

-40-
ASOCIACIN EDUCATIVA PITGORAS Fsica

Para plantear la solucin a este problema, podemos D.C.L. del bloque


escoger cualquiera de las 3 formas que indicamos a
continuacin.
1. Por descomposicin rectangular : trazando un
sistema de coordenadas rectangulares.
Se debe cumplir que :

a. 3 Y

B. 3 Y

Primera solucin
Por descomposicin rectangular. Ubicando los ejes
adecuadamente :

2. Mediante el tringulo de fuerzas, ya que si la


resultante es cero, los vectores fuerza deben formar
un polgono cerrado.

T = 50 N

3. Aplicando el Teorema de Lamy

N = 50 N

Segunda solucin :
DIAGRAMA DEL CUERPO LIBRE (D.C.L)
Mediante el tringulo de fuerzas :
Consiste en aislar a un cuerpo y graficar sobre l, primero
su peso y luego todas las fuerzas externas que actan
sobre l (tensiones, compresiones, reacciones, etc)

Ejemplo 1 :

En el sistema mostrado hallar la tensin en la cuerda y la


reaccin normal del plano inclinado liso, si el bloque pesa
100 N, existe equilibrio

! T=K
T = 50 N
! N=K
N = 50 N

-41-
ASOCIACIN EDUCATIVA PITGORAS Fsica

Tercera solucin : Primera solucin :

Aplicando el Teorema de Lamy :

3Fx = 0
N1 - N2 Cos53 = 0
N1 = N2Cos53

3Fy = 0
N2 Sen53 - 100 = 0
N2 = 125 N

Reemplazando :
N1 = 75 N

Segunda solucin :

De (1) ! ,

T = 50 N

De (2) ! ,
Tercera solucin :

Por el Teorema de Lamy


N = 50 N

Ejemplo 2 :

El sistema mostrado est en equilibrio. Hallar las


reacciones normales si el peso de la esfera homognea
es de 100 N y no existe rozamiento

Resolviendo :
N1 = 75 N
N2 = 125 N

-42-
ASOCIACIN EDUCATIVA PITGORAS Fsica

PROBLEMAS PROPUESTOS
01. Calcular F para mantener el equilibrio
(W bloque = 300 N)

A) 3 N B) 6 N C) 9 N
D) 12 N E) 15 N

06. Se muestra un cuerpo de peso 2P en equilibro. Qu


proposicin es verdadera?

A) 75 N B) 150 N C) 300 N
D) 100 N E) 200 N

02. Si RA = 5 N, RB = 13 N, hallar el peso de la barra

A) La tensin en 1 es P
B) La tensin en 2 es 2P
C) La tensin en 3 es 2P
D) La tensin en 1 es igual que en 2
E) El cuerpo no puede estar en equilibrio

07. El conjunto de cuerpos mostrados estn en equilibrio.


A) 6 N B) 18 N C) 12 N Determinar el mdulo de la tensin en la cuerda 1.
D) 9 N E) 8 N 2
(M = 4 m). Considere g=10 m/s y desprecie el
rozamiento
03. El sistema mostrado se encuentra en equilibrio y la
barra es de peso despreciable, encontrar la reaccin
de la articulacin.

A) 40 N B) 20 N C) 35 N
D) 30 N E) 25 N

08. Cada uno de los cuerpos mostrados se encuentran


en equilibrio. Determinar el mdulo de la fuerza de
2
reaccin del piso sobre el cuerpo A. (g = 10 m/s )
A) 100 N B) 200 N C) 300 N
D) 400 N E) 500 N

04. Hallar la tensin de la cuerda AB, el peso del bloque


en equilibrio es 49 N

A) 20 N B) 40 N C) 30 N
D) 16 N E) 60 N

09. Una persona sostiene una esfera sobre un plano


inclinado. En cul de los casos se ejerce menos
fuerza sobre la esfera? Desprecie el rozamiento
I.
A) 140 N B) 135 N C) 165 N
D) 80 N E) 220 N

05. El sistema est en equilibrio. Hallar el peso del bloque


A
W B = 7 N; W C = 5 N

-43-
ASOCIACIN EDUCATIVA PITGORAS Fsica

II.

A) 90 N B) 90 N C) 80 N
A) En I
B) En II D) 80 N E) 50 N
C) En ambos casos
D) Falta conocer 14. Si la barra homognea de 100 N se encuentra en
E) Depende de la masa de la esfera equilibrio, determine el mdulo de la tensin en la
cuerda

10. Determinar la medida del ngulo , si el conjunto de


cuerpos permanece en equilibrio. Los bloques tienen
pesos iguales

A) 200 N B) 80 N C) 50 N
D) 100 N E) 100 N

15. Una barra homognea de 80 N se encuentra apoyada


en una pared lisa. Calcular el mdulo de la fuerza que
le ejerce la articulacin A a la barra mencionada.
A) 16 B) 53 C) 37
D) 74 E) 60

11. Las esferas mostradas son idnticas y de 60 N cada


una. Calcular el mdulo de la fuerza que ejerce el
taco de madera sobre la esfera inferior. Desprecie el
rozamiento
A) 80 N B) 60 N C) 200 N
D) 100 N E) 120 N

16. El delgado cable que pasa por un pequeo orificio


practicado en la barra de 40 N, sostiene a la esfera
de 60 N. Hallar el mdulo de la fuerza de reaccin
sobre el extremo, inferior de la barra

A) 60 N B) 100 N C) 160 N
D) 120 N E) 200 N

12. La esfera homognea permanece en reposo apoyada


sobre una superficie semiesfrica y atada con una
cuerda en donde la tensin es de 48 N. Calcular el
mdulo de la fuerza de reaccin por parte de la A) 100 N B) 80 N C) 60 N
superficie semiesfrica. Desprecie el rozamiento D) 120 N E) 110 N

17. Una cuerda de peso P est suspendida por los


extremos (tal como se muestra). Si dicha cuerda
presenta una tensin T en su punto inferior , calcular
la fuerza que ejerce el techo sobre el extremo B de la
cuerda

A) 56 N B) 60 N C) 80 N
D) 50 N E) 36 N

13. Si el cuerpo A de 90 N de peso se encuentra en


equilibrio, calcular el mdulo de la fuerza F que hace A) B) C)
posible esto. Desprecie el rozamiento
D) E)

-44-
ASOCIACIN EDUCATIVA PITGORAS Fsica

18. Una esfera homognea de 2 kg se mantiene en un A) 127 B) 180 C) 270


plano inclinado en la forma como se indica. Calcular D) 360 E) 217
el mdulo de la fuerza con que la esfera aplasta al
plano. Considerar la longitud natural del resorte 20. Una esfera de 40 N est en reposo sobre un plano
2
10 cm y K=5 N/cm. (g=10 m/s ). Despreciar el inclinado y sujetada por un cable ideal, Calcular el
rozamiento mdulo de la tensin en el cable

A) 16 N B) 15 N C) 25 N
A) 15 N B) 20 N C) 10 N
D) 30 N E) 32 N
D) 5 N E) 15 N
19. Los bloques suspendidos se encuentran en equilibrio.
Determinar +

TAREA
01. Si los 3 bloques tienen la misma masa, calcular 03. Si la barra de 80 N de peso se encuentra en
para el equilibrio (No existe rozamiento) equilibrio, hallar la lectura del dinammetro

A) 30 B) 60 C) 37
D) 53 E) 45 A) 60 N B) 90 N C) 160 N
D) 80 N E) 120 N
02. En el sistema mostrado en equilibrio, calcular el valor
de ( > 0) 04. El resorte de constante K = 10 N/cm sostiene a una
esfera de 24 N de peso. Determinar la deformacin
del resorte

A) 15 B) 45 C)- 55
D) 75 E) 90 A) 5 cm B) 4 cm C) 3 cm
D) 2 cm E) 1 cm

-45-
ASOCIACIN EDUCATIVA PITGORAS Fsica

05. Si los bloques pesan 80 y 20 N, hallar el valor de la 08. Hallar lo que marca el dinammetro en los casos
tensin T del cable para el equilibrio mostrados. El bloque pesa 360 N y las poleas no
pesan

A) 50 N B) 100 N C) 20 N
D) 25 N E) 30 N A) 120 N; 90 N B) 100 N; 70 N C) 250 N; 300 N
D) 120 N; 200 N E) 50 N; 90 N
06. La figura muestra una esfera de radio r y peso 6 N
apoyada en una superficie cilndrica de radio R. 09. Si la pequea esfera pesa 400 N, hallar la tensin en
Hallar la reaccin en el punto A la cuerda que lo sostiene
(R = 3r)

A) 250 N B) 200 N C) 150 N


D) 100 N E) 50 N

A) N B) 25 N C) 30 N 10. La esfera mostrada de radio 1 m y masa 3 kg se


D) 2 N E) 3 N encuentra apoyada en un hoyo semicilndrico de radio
igual a 80 cm. Calcular la reaccin en los puntos de
07. Determinar la reaccin de la pared sobre la esfera de contacto A y B
peso 80 N. No considere rozamiento

A) RA = RB = 30 N
B) RA = RB = 25 N
C) RA = 15; RB = 10 N
A) 40 N B) 50 N C) 60 N D) RA = RB = 20 N
D) 80 N E) 100 N E) RA = 10 N; RB = 15 N

SEGUNDA CONDICIN DE EQUILIBRIO

MOMENTO DE UNA FUERZA ( )


El momento producido por la
fuerza F con respecto al
Es una magnitud vectorial, donde su mdulo indica el punto O est dado por :
grado de giro que produce una fuerza a un cuerpo
alrededor de un punto denominado: centro de momentos
o centro de giro. La direccin del vector momento es
perpendicular al plano formado por el centro de giro y la
lnea de accin de la fuerza y su sentido se determina
mediante la regla de la mano derecha. d = OP = brazo de palanca
F = fuerza aplicada

-46-
ASOCIACIN EDUCATIVA PITGORAS Fsica

CONVENCIN DE SIGNOS = .........................................................................


v Si el cuerpo gira o intenta girar en sentido horario,
debido a una fuerza F, se dice que el momento = .........................................................................
producido por dicha fuerza es negativo

= .........................................................................

= .........................................................................
v Si el cuerpo o sistema gira o intenta girar en sentido
antihorario. Debido a una fuerza F, se dice que el = .........................................................................
momento producido por dicha fuerza es : positivo
TEOREMA DE VARIGNON
En un sistema de fuerzas, la suma de momentos
producidos por cada una de ellas, es igual al momento
producido por la fuerza resultante del sistema

APLICACIN
En el siguiente sistema de fuerzas paralelas, determinar
a qu distancia del extremo A acta la fuerza resultante
Alrededor de A : Movimiento positivo

Alrededor de A : Momento negativo


v Clculo del mdulo de la fuerza resultante : (R)
CASO PARTICULAR : R = +100 - 80 - 20 + 100 ! R = +100 N
Cuando una fuerza acta directamente en el centro de
momentos o su lnea de accin pasa por dicho punto, el v Clculo del momento resultante o suma de
momento producido por la fuerza es cero momentos: (3 MA )
= -(80)(1) - (20)(2) + (100)(3) ! =+180 N.m

v Aplicando el Teorema de Varignon :


=
Donde : R; es la fuerza resultante

+ 180 N.m = (R)(x)


EJEMPLO : Determinar el momento producido por cada + 180 N.m = 100 N (x)
una de las fuerzas que actan sobre la lmina cuadrada x = 1,8 m
y el momento resultante con respecto al punto O
F1 = 60 N; F2 = 20 N; F3 = 70 N; F4 = 141 N
P = Punto de aplicacin de la fuerza resultante (R)

SEGUNDA CONDICIN DE EQUILIBRIO

Si un cuerpo se encuentra en equilibrio, se cumple, que


la suma de momentos de las fuerzas que actan sobre l,
con respecto a un mismo punto es igual a cero

-47-
ASOCIACIN EDUCATIVA PITGORAS Fsica

NOTA : Para que un cuerpo se encuentre en equilibrio es v Aplicando la segunda condicin de equilibrio con
necesario que cumpla con las 2 condiciones de equilibrio
respecto al punto O : =0
APLICACIN : = + (540 N)(AO) - (W B)(OB) = 0
Determinar el peso que debe tener la persona sentada en
el extremo derecho, para que el sistema pueda estar en
equilibrio. Adems la persona sentada en el extremo (540 N)(1,2 m) = W B (1,8 m) ! W B = 360 N
izquierdo pesa 540 N
(No considere el peso de la barra AB)
AO = 1,2 m; OB = 1,8 m CUPLA O PAR DE FUERZAS

Es un sistema de 2 fuerzas paralelas; iguales en mdulo


y dirigidas en sentido contrario, cuando una cupla acta
sobre un cuerpo trata de proporcionarle cierto movimiento
giratorio

RESOLUCIN :
v Grafiquemos el diagrama de cuerpo libre de AB

Mcupla = F . d

PROBLEMAS PROPUESTOS
01. Determine a qu distancia del apoyo articulado A se 03. La viga ABC es de seccin uniforme. Su peso propio
encontrar la fuerza resultante de las cuatro fuerzas es de 40 newtons y se apoya en una articulacin
paralelas que se muestran. (punto B). En el extremo C se halla sometida a la
tensin de un cable. Considerando el sistema en
equilibrio. Cunto valdr la tensin del cable en
newtons? (Considere: g = 10 m/s2)

A) 1 m B) 2 m C) 3 m
D) 4 m E) 5 m A) 10 B) 20 C) 30
D) 40 E) 50
02. Un peso P est colocado sobre una viga horizontal
apoyada en A y B. La distancia entre los soportes: es 04. Una barra uniforme, de peso 100 N, est sujeta
de 3 m y el peso P est situado de tal manera que mediante tres cuerdas, como se indica en la figura. Si
la reaccin en el soporte A es el doble de la una pesa W de 200 N, se coloca en la posicin
reaccin en el soporte B. Sin considerar el peso de indicada, cules sern las tensiones en newtons, en
la viga, la distancia x en metros es: cada cuerda T1, T2, T3 respectivamente.

A) 0,5 B) 2,5 C) 2,0


D) 1,5 E) 1,0 A) 100; 200; 300 B) 100; 250; 400
C) 100; 150; 250 D) 150; 250; 350
E) 150; 300; 450

-48-
ASOCIACIN EDUCATIVA PITGORAS Fsica

05. La barra homognea se encuentra en equilibrio tal 10. La barra de la figura de 1 m de longitud es
como se indica, determine en qu relacin se homognea y descansa inicialmente sobre el piso y
encuentran la tensin en la cuerda horizontal y el la pared vertical, ambos lisos. El resorte unido a la
peso de la barra. barra en su extremo inferior tiene una constante
elstica de 50 N/m. Cuando la barra est vertical el
resorte no est estirado. Calcular el peso de la barra
si en la posicin indicada, sta se encuentra en
equilibrio.

A) 3/2 B) 3/4 C) 4/3


D) 2/3 E) 1/3

06. El sistema mostrado se encuentra en equilibrio, la


placa triangular homognea pesa 60 N. Determine la
tensin en la cuerda vertical.
A) 20 N B) 30 N C) 40 N
D) 50 N E) 60 N
11. Un espejo uniforme de 13 N cuelga de dos cuerdas
como se muestra. Encuentre la magnitud de la fuerza
necesaria para mantenerlo en su posicin.
A) 20 N B) 40 N C) 60 N
D) 80 N E) 90 N

07. Un alambre rgido homogneo de 25 cm de longitud


es doblado tal como se indica, con a = 5 cm. Para
que el alambre apoyado se mantenga en equilibrio la
longitud x deber ser:

A) 9 N B) 7 N C) 5 N
D) 3 N E) 1 N

12. En la figura, se muestra a una barra homognea de


peso W y longitud L si no existe rozamiento se
A) 5 cm B) 7 cm C) 12 cm pide determinar la tensin en la cuerda horizontal.
D) 15 cm E) 18 cm (L = 5a)
08. El sistema que se muestra est en equilibrio, la barra
homognea tiene 5 m de longitud y cada bloque pesa
50 N. Qu peso tiene la barra? ( = 1 m)

A) 5W/4 B) 5W/8 C) 5W/12


D) 5W/24 E) 4W/32

13. A partir del equilibrio existente en el sistema


mostrado, determine la tensin en la cuerda
perpendicular a la barra homognea, en su punto
medio. Se sabe que la barra y el bloque pesan 60 N
cada uno.
A) 5 N B) 10 N C) 15 N
D) 20 N E) 25 N

09. La barra quebrada est en equilibrio y se pide


determinar el valor del ngulo ; la barra pesada es
homognea.

A) 37 B) 53 C) 60 A) 25 N B) 30 N C) 35 N
D) 74 E) 75 D) 40 N E) 45 N

-49-
ASOCIACIN EDUCATIVA PITGORAS Fsica

14. En la figura se muestra un sistema en equilibrio 18. Una persona de peso W camina sobre una tabla
conformado por una barra homognea de 400 N de homognea como se muestra en la figura. Qu
peso y una esfera de 700 N de peso, dispuestas tal distancia mxima X avanzar a partir del punto O
como se indican. Si la barra se apoya en su punto para que la tabla contine en equilibrio? Peso de la
medio sobre la esfera, determine la reaccin del piso barra 3W
sobre la esfera.

A) L/7 B) 7L/24 C) 3L/4


A) 1 400 N B) 1 200 N C) 1 000 N D) L/4 E) L/3
D) 800 N E) 600 N
19. Una placa homognea descansa sobre dos muelles
15. Cul es el valor de la tensin que soporta la cuerda elsticos, encuentre K1/K2 conocindose que los
horizontal en el sistema en equilibrio que se muestra; muelles estn igualmente deformados
la barra homognea pesa 100 N y el bloque pesa
400 N.

A) 1/2 B) 1/3 C) 1/4


D) 2/3 E) 3/4
A) 200 N B) 400 N C) 600 N 20. La figura muestra una barra AB uniforme y
D) 800 N E) 900 N homognea de 5 newtons de peso y 4 m de longitud.
Si la esfera de 10 newtons de peso se encuentra
16. En la figura el peso del bloque es 15 N y la barra es apoyada sobre la barra, hallar la fuerza de reaccin
de peso despreciable. Hallar la reaccin en el apoyo entre la barra y la esfera
C

A) 2 N B) 3 N C) 4 N
D) 12 N E) 6 N
A) 18,8 N B) 28,8 N C) 38,8 N
D) 48,8 N E) 58,8 N

17. En el sistema que se muestra existe equilibrio y se


sabe que la barra es de peso despreciable, la
esfera pesa 80 N y su radio es la octava parte de la
longitud de . Cul es la tensin en la cuerda?

A) 15 N B) 20 N C) 25 N
D) 30 N E) 35 N

-50-
ASOCIACIN EDUCATIVA PITGORAS Fsica

TAREA

01. Una barra horizontal de peso P y de longitud


5a, puede rotar alrededor de un eje fijado en el gozne
A. Un peso de valor tambin P est suspendido a
una distancia a del extremo A; para que el sistema
est en equilibrio la fuerza vertical F cuya direccin
dista del extremo B es :

A) 10 N B) 15 N C) 25 N
D) 20 N E) 25 N

06. Si el sistema mostrado en la figura se encuentra en


equilibrio, los pesos de la barra AB y el bloque Q de
A) P/9 B) 2P/9 C) 4P/9 60 N y 30 N respectivamente, hallar la tensin del
D) 5P/9 E) 7P/9 cable que sostiene a la barra
02. Las dos barras son idnticas y se encuentran en
equilibrio; si cada barra pesa 50 N. Cul es el valor
de la reaccin en la articulacin?

A) 25N B) 25 N C) 25 N A) 60 N B) 120 N C) 100 N


D) 40 N E) 80 N
D) 25 N E) 50 N
07. Si la barra horizontal AB, uniforme y homognea pesa
03. Encuentre F para mantener horizontalmente una 40 newtons, determinar la fuerza de tensin en la
barra homognea de 20 N de peso cuerda 1". El peso de la polea mvil es despreciable

A) 14 N B) 15 N C) 16 N
D) 17 N E) 18 N

04. Una placa cuadrada de poco peso tiene 10 m en


cada lado, sobre ella actan 4 fuerzas como se puede A) 10 N B) 8 N C) 6 N
ver en el diagrama, halle el momento de fuerza (en D) 12 N E) 5 N
N x m) en el instante mostrado, alrededor de la
articulacin. 08. Si la barra doblada en forma de T es de peso
despreciable y en sus extremos estn soldadas dos
esferillas de pesos W y 6W, hallar el ngulo que
define la posicin de equilibrio del sistema

A) -68 B) +68 C) -88


D) +88 E) 0

05. El sistema fsico mostrado en la figura se encuentra


en equilibrio. Si la estructura es de peso
despreciable y la esfera A pesa 50 N, hallar la
A) 37 B) 53 C) 60
tensin en la cuerda horizontal
D) 30 E) 45

-51-
ASOCIACIN EDUCATIVA PITGORAS Fsica

09. Una barra homognea de 140 N se encuentra en 10. Si la barra uniforme mostrada pesa 5 N y mide 15 m,
equilibrio. Determinar la suma de las deformaciones hallar la tensin en la cuerda horizontal, sabiendo
que experimentan los resortes de rigideces que el bloque pesa 10 N.
K1=2 N/cm, K2=3 N/cm. Los resortes se encuentran
sin deformar cuando la barra se encuentra horizontal.

A) 15 N B) 10 N C) 5 N
D) 20 N E) 25 N
A) 15 cm B) 30 cm C) 45 cm
D) 40 cm E) 55 cm

DINMICA
DINMICA

Estudia la dependencia entre el movimiento de los


cuerpos materiales y las fuerzas que actan sobre ellos.
$
El movimiento de un cuerpo dado queda determinado por
la naturaleza y disposicin de los otros cuerpos que
forman su medio ambiente as como por las condiciones
iniciales del movimiento. FUERZA DE GRAVEDAD

INERCIA Es la fuerza de atraccin gravitatoria ejercida sobre un


cuerpo por la Tierra.
La comparacin de los resultados de la accin de una La fuerza de atraccin gravitatoria ejercida sobre un
misma fuerza sobre cuerpos diferentes, conduce a la cuerpo por la Tierra es un aspecto de la accin mutua
nocin de la inercia de los cuerpos. La experiencia entre la Tierra y el cuerpo. Esto es, la Tierra atrae al
muestra, que en general, si se aplica una misma fuerza a cuerpo y al mismo tiempo el cuerpo atrae a la Tierra. La
2 cuerpos distintos en reposo, libres de otras influencias, fuerza ejercida sobre la Tierra por el cuerpo tiene igual
estos cuerpos durante un mismo intervalo de tiempo magnitud y direccin opuesta a la fuerza ejercida sobre el
recorrern distancias diferentes y adquirirn distintas cuerpo por la Tierra.
velocidades. La fuerza de gravedad (peso) no es una propiedad del
cuerpo, si no que depende de su masa y de las
La inercia caracteriza la propiedad de los cuerpos caractersticas del planeta o del elemento material que
materiales de cambiar ms rpido o ms lentamente la ejerce atraccin sobre el cuerpo.
velocidad de su movimiento bajo la accin de las fuerzas
aplicadas. APLICACIONES DE LA SEGUNDA LEY DE NEWTON

La medida cuantitativa de la inercia de un cuerpo dado, I. Al movimiento rectilneo


es una magnitud fsica escalar denominada masa del
cuerpo. En mecnica se considera que la masa es una En este caso se debe tener en cuenta que la
cantidad escalar positiva, y constante para cada cuerpo aceleracin es paralela a la trayectoria rectilnea, por
dado, es decir no depende de la velocidad del cuerpo lo que en este caso es recomendable descomponer
considerado. las fuerzas en una componente paralela y
perpendicular a la trayectoria rectilnea, tenindose :
SEGUNDA LEY DE NEWTON
3Fx = ma 3Fy = 0
Toda fuerza resultante no nula al actuar sobre un cuerpo
de masa m constante produce una aceleracin que
posee la misma direccin de la fuerza resultante, siendo
su valor directamente proporcional al valor de la fuerza
resultante e inversamente proporcional a la masa del
cuerpo.

II. Al movimiento circular

-52-
ASOCIACIN EDUCATIVA PITGORAS Fsica

Para este caso la fuerza resultante se analiza en B. Componente tangencial ( ) : Esta


trminos de las siguientes componentes :
componente se obtiene sumando las componentes
tangenciales de las diferentes fuerzas actuantes,
A. Componente radial ( ) : Llamada tambin produciendo la aceleracin tangencial.
fuerza centrpeta, se obtiene mediante la suma de
las componentes radicales de las diferentes
fuerzas actuantes y genera a la aceleracin
centrpeta.

El papel de esta componente tangencial es la de


modificar la velocidad , es decir, acelera o retarda
el movimiento.

ROZAMIENTO

La resistencia que se opone al resbalamiento, o a su


tendencia a resbalar, de un cuerpo sobre otro es una
fuerza tangente a la superficie de contacto, que recibe el
nombre de rozamiento.

El papel de la fuerza centrpeta es desviar


continuamente el cuerpo del camino rectilneo que
recorrera por inercia en ausencia de la fuerza
actuante

PROBLEMAS PROPUESTOS
01. Una fuerza al actuar sobre un cuerpo le comunica aplica una fuerza de 8 N durante 10 s, luego se deja
2
una aceleracin de 2 m/s y cuando acta sobre otro de aplicar dicha fuerza durante 20 s y finalmente se
2
cuerpo, le comunica una aceleracin de 3 m/s . Si la le aplica una fuerza de 16 N en sentido opuesto al de
misma fuerza acta sobre los dos cuerpos juntos, su velocidad, hasta que se detiene. Calcular la
cul ser la aceleracin del sistema formado por los distancia total recorrida por el bloque
dos cuerpos? A) 1 200 m B) 1 000 m C) 750 m
D) 830 m E) 1 100 m
2 2 2
A) 5 m/s B) 0,5 m/s C) 0,2 m/s
2 2
D) 1,2 m/s E) 0,8 m/s 05. El sistema mostrado carece de friccin. Determine la
deformacin del resorte de K=500 N/m
02. Encontrar la compresin que experimentan los
bloques, si se sabe que no existe rozamiento.

A) 20 cm B) 40 cm C) 60 cm
D) 120 cm E) 1,2 cm
2
06. Un ascensor tiene una aceleracin de 1 m/s hacia
abajo. Cul ser el estiramiento del resorte adherido
al techo del ascensor?
2
Si : m = 1 kg; K = 36 N/m y g = 10 m/s
A) 40 N B) 20 N C) 46 N
D) 45 N E) 10 N

03. En un automvil de 1 000 kg de masa que viaja con


una velocidad de 20 m/s se malogran los frenos y
para detenerlo se coloca una ruma de heno. Si
atraviesa el heno en 4 s y sale con una velocidad
de 8 m/s, hallar la fuerza promedio que actu sobre
l
A) 0,30 m B) 0,25 m C) 0,27 m
A) 1 kN B) 3 kN C) 10 kN D) 0,15 m E) 0,35 m
D) 5 kN E) 30 kN 07. Un bloque es abandonado desde lo alto de un plano
inclinado liso. Si recorre 15 m en su tercer segundo
04. Un bloque de 2 kg de masa se encuentra en reposo de movimiento, determinar el ngulo de inclinacin
sobre una superficie horizontal sin rozamiento. Se le del plano. (g=10 m/s )
2

-53-
ASOCIACIN EDUCATIVA PITGORAS Fsica

A) 30 B) 37 C) 45 un dinmetro de masa despreciable que a su vez est


D) 53 E) 60 suspendido del techo de un vagn que acelera con
2
a=24 m/s . Cunto marca el dinmetro?
2
08. Si el sistema mostrado parte del reposo, g = 10 m/s
despreciando el rozamiento, con qu velocidad
llegar el bloque a tierra?
2
(g = 10 m/s )

A) 130 N B) 260 N C) 65 N
D) 100 N E) 75 N

13. Dos partculas de masas iguales de 200 g cada una


gira con velocidad angular constante de 2 rad/s sobre
A) 1 m/s B) 2 m/s C) 3 m/s un plano horizontal liso. Hallar la tensin en el hilo (1)
D) 4 m/s E) 5 m/s

09. Calcular la tensin en la cuerda que une los bloques


A y B. Despreciar el rozamiento y considerar
2
g = 10 m/s

A) 2,4 N B) 1,6 N C) 0,8 N


D) 3,6 N E) 2,8 N

14. Una piedra gira en un plano vertical, de modo que


describe una circunferencia. Si est atada a una
cuerda de longitud 2,5 m, cul es la mnima
velocidad angular que debe mantener la piedra para
2
A) 6,25 N B) 7,25 N C) 7,50 N continuar con su movimiento circular? (g = 10 m/s )
D) 8,75 N E) 9,25 N A) 1 rad/s B) rad/s C) 3 rad/s
D) 4 rad/s E) 5 rad/s
10. Dos bloques de 100 N y 200 N estn unidos por una
cuerda unida a un resorte de masa despreciable y de 15. Una esfera de 5 kg asciende por un rizo de modo
constante K = 300 N/m, como se indica en la figura. que en el instante mostrado presenta una velocidad
Considerando lisas las superficies, hallar el V= 62 m/s. Si el rizo tiene un radio R = 4,5 m y no
estiramiento del resorte una vez establecido el existe rozamiento, cul es la reaccin del rizo en la
2
movimiento. (g = 10 m/s ) posicin mostrada?

A) 0 N B) 40 N C) 80 N
A) 10 cm B) 20 cm C) 30 cm D) 30 N E) 20 N
D) 40 cm E) 50 cm
16. Una piedra gira en un plano vertical describiendo una
11. Hallar la aceleracin del carrito para que el bloque circunferencia. Si la cuerda que lo mantiene en
m no resbale (no hay friccin) movimiento puede soportar como mximo 10 veces
su peso, cul es la mxima velocidad que puede
experimentar dicho cuerpo sin llegar a romper la
cuerda de longitud 2,5 m?
2
(g = 10 m/s )
A) 5 m/s B) 10 m/s C) 15 m/s
D) 20 m/s E) 25 m/s

A) gSen B) gCtg C) gCos 17. Una bolita se encuentra atada a una cuerda y gira en
D) gCsc E) gTg un plano vertical. Si en el instante mostrado su
velocidad lineal es de 5 m/s, cul es la tensin de la
12. Un cuerpo de masa 5 kg est colgado del gancho de cuerda?

-54-
ASOCIACIN EDUCATIVA PITGORAS Fsica

2
L = 2 m; = 53; m = 6 kg; g = 10 m/s 19. El sistema mostrado rota uniformemente con respecto
a un eje vertical, la cuerda que une a los dos bloques
es de longitud L y se pide determinar la longitud x

A) 111 N B) 36 N C) 75 N
D) 85 N E) N.A.

18. La figura muestra un pndulo cnico de masa m y


altura h, que gira con velocidad angular constante.
Hallar la velocidad angular.
A) B)

C) D)

E)

20. Cmo estn relacionadas entre s las fuerzas con


las cuales un carro hace presin en el centro de un
A) = B) = puente cncavo y de un puente convexo el radio de
C) = D) = curvatura del puente en ambos casos es igual a 45 m
y la velocidad del carro es la misma e igual a
E) = 54 km/h.
A) 7 B) 8 C) 6
D) 5 E) 3

TAREA
01. Se muestra un bloque de 6 kg de masa sujeto por dos elstica que puede estirarse 0,5 m como mximo, si
cuerdas, tal como se muestra, que gira a una la plataforma de 4 M de masa, posee una aceleracin
velocidad angular constante de 2 rad/s. Calcular la de 4 m/s2 (L > 0,5 m; g = 10 m/s2)
tensin en la cuerda inferior. (g = 10 m/s2)

A) 1 N B) 2 N C) 3 N
D) 4 N E) 5 N
A) 17 N B) 42,5 N C) 45 N
D) 22,5 N E) 21,25 N 04. Con qu aceleracin debe bajar el carro para que el
hilo que sostiene a la esfera, se mantenga horizontal
02. Si el sistema se desplaza con una aceleracin de respecto a tierra?
2
6 m/s , hallar la fuerza de tensin en la cuerda que
sostiene al bloque: m=5 kg

A) gSen B) gSec C) gCsc


A) Cero B) 10 N C) 20 N D) gTg E) gCos
D) 30 N E) 50 N
2
05. Si el ascensor sube con una aceleracin de 2 m/s .
03. Hallar la mxima tensin soportada por la cuerda Determinar la aceleracin de los bloques con

-55-
ASOCIACIN EDUCATIVA PITGORAS Fsica

respecto del ascensor 08. El sistema resorte - bloque gira con una velocidad
mA = 4mB = 8 kg (g = 10 m/s2) angular de 8 rad/s. Si no existe rozamiento. Hallar la
deformacin producida en el resorte. (K = 256 N/cm;
m = 5 kg)

2 2 2 A) 1 cm B) 2 cm C) 2,5 cm
A) 7,2 m/s B) 6,8 m/s C) 6,4 m/s D) 4 cm E) 5 cm
2 2
D) 6 m/s E) 3,6 m/s
09. Una bolita de masa m descansa inicialmente en la
06. Con respecto al sistema mostrado, indicar verdadero parte ms baja del casquete semiesfrico cuyo
(V) o falso (F) : interior es liso y tiene 2 m de radio. Determinar el
valor de , cuando el casquete gire a razn de
rad/s

( ) La fuerza de reaccin entre m y M aumenta


cuando F, aumenta A) 30 B) 37 C) 45
( ) Al aumentar el valor de m la fuerza de reaccin D) 53 E) 60
entre las masas aumenta
( ) Si : m=M, la reaccin entre ellas es cero 10. Una esfera de 5 kg asciende por un rizo de modo que
A) VFV B) VFF C) VVV en el instante mostrado, presenta una velocidad de
D) FFF E) FVV 6 m/s. Si el rizo tiene un radio R = 4,5 m y no
existe rozamiento, cul es la reaccin del rizo en la
07. La persona de 60 kg se encuentra sobre una
posicin mostrada?
plataforma de 4 kg y sostiene una piedra de 1 kg
mediante una cuerda. Hallar la tensin en dicha
cuerda.
2
(g = 10 m/s ; M = 35 kg)

A) Cero B) 10 N C) 20 N
D) 40 N E) 80 N

A) 3 N B) 5 N C) 6 N
D) 7 N E) 10 N

ROZAMIENTO
CONCEPTO diagramas del cuerpo libre para problemas donde
La resistencia que se opone al resbalamiento, o a su interviene el rozamiento son los mismos que para
tendencia a resbalar, de un cuerpo sobre otro en una aquellos en que intervienen superficies lisas, salvo que ha
fuerza tangente a la superficie de contacto, que recibe el de incluirse una fuerza de rozamiento tangente a la
nombre de rozamiento. Las superficies en realidad no son superficie de contacto.
lisas por lo que la reaccin de un cuerpo sobre otro no es
normal a dicha superficie de contacto. Si se descompone
la reaccin (F) en dos componentes, una perpendicular
(N) y otra tangente a la superficie de contacto, la
componente tangencial (f) a dicha superficie se denomina
fuerza de friccin o rozamiento. En consecuencia, los

-56-
ASOCIACIN EDUCATIVA PITGORAS Fsica

dos materiales cualquiera determinados, cuando las


superficies de contacto estn en una condicin fijada. No
obstante, vara mucho para diferentes condiciones de las
superficies y con la naturaleza de los cuerpos en contacto.

Leyes de Rozamiento: Los resultados de un gran nmero


de experiencias sobre el rozamiento en superficies secas,
publicadas por C.A de Coulomb en 1781, proporcionaron
las primeras informaciones sobre las leyes del
rozamiento, obteniendose las siguientes leyes:

1. La fuerza mxima de rozamiento que puede


Se suele hablar de dos tipos de rozamiento: producirse es proporcional a la fuerza normal entre
las superficies en contacto.
A. Rozamiento Esttico (fs): Cuando no hay
movimiento relativo entre los cuerpos en contacto; es 2. Esta fuerza mxima es independiente del tamao de
decir, cuando ninguno se mueve, o ambos se la superficie de contacto.
desplazan como si fueran uno solo, oponindose a
cualquier intento de movimiento relativo. En este caso 3. La fuerza lmite de rozamiento esttico es mayor que
la fuerza de rozamiento desarrollada es exactamente la fuerza de rozamiento cintico, siempre que acte
suficiente para mantener el reposo relativo con las la misma fuerza normal.
dems fuerzas que actan sobre el cuerpo.
Esto implica que la fuerza de rozamiento esttico es una 4. El coeficiente de rozamiento cintico es menor que el
fuerza regulable o variable alcanzando un valor mximo coeficiente de rozamiento esttico.
o limite, el cual depende de la normal y de la aspereza de
la superficie en contacto. Por lo tanto la fuerza de 5. La fuerza de rozamiento cintico es independiente de
rozamiento esttico cumple con : la velocidad relativa de los cuerpos en contacto.

Ejemplos :
0 # fs #
1. Hallar la reaccin total de la superficie spera sobre
el bloque de peso 80 N, si est a punto de resbalar.
B. Rozamiento Cintico (fk) : Se genera cuando los s = 0,8
cuerpos en contacto se encuentran en movimiento
relativo. La fuerza de rozamiento es constante y
prcticamente independiente del valor de la velocidad
relativa.

Coeficiente de Rozamiento : Constante experimental


que permite comparar las propiedades de rozamiento de
pares distintos o iguales de materiales en diferentes
condiciones de sus superficies en contacto, y con objeto
de calcular la fuerza de rozamiento mxima Resolucin:
correspondiente a una fuerza normal cualquiera.
El coeficiente de rozamiento esttico de 2 superficies
cualquiera se define como la razn del rozamiento
mximo o lmite a la fuerza normal correspondiente :

Donde el rozamiento lmite es el rozamiento que existe 3 Fx = 0


cuando las superficies estn a punto de empezar a
moverse la una con respecto a la otra (estado de F = s N
movimiento inminente)
En general, cuando las superficies en contacto se mueven F= N ....... (1)
una respecto a la otra, el rozamiento disminuye. En este
caso, la razn de la fuerza de rozamiento a la fuerza F=N
normal se define como coeficiente de rozamiento cintico.
3Fy = 0
F + N = 80 ...... (2)

(1) en (2)

N = 50 newton
El valor del coeficiente de rozamiento tiene que Fsmax = sN = 0,8.50 = 40 newton
determinarse experimentalmente, y es una constante para

-57-
ASOCIACIN EDUCATIVA PITGORAS Fsica

Luego : 3. Halla la aceleracin del bloque sobre el plano


inclinado spero; (k = 0,5, m = 5 kg, g = 10 m/s2)

F=
F=
F = 10 newton

2. Hallar mximo para que el cuerpo no resbale Resolucin:


sobre el plano inclinado spero ( s = 0,75)

FR = ma
3Fx = 0 Y sN = mgSen
80 - 30 - k N = ma ..... (1)
3Fy = 0 Y N = mgCos
3Fy = 0
Dividiendo :
N = 40
s = Tg
En (1)
= Tg Y = 37
80 - 30 - 0,5.40 = 5a
2
a = 6 m/s
Tambin :

Tg =

Tg = s

PROBLEMAS PROPUESTOS

01. Si la masa de 5 kg es jalada por la fuerza F de 50 N, 02. El bloque A de la figura pesa 40 N y sus coeficientes
con qu aceleracin avanza la masa si = 0,5 de rozamiento con el plano horizontal son 0,15 y 0,25.
Hallar el mximo peso del bloque B para mantener el
2
(g = 10 m/s ) equilibrio

A) 2 m/s2 B) 3 m/s2 C) 4 m/s2


2 2
D) 5 m/s E) 6 m/s A) 40 N B) 30 N C) 20 N
D) 10 N E) 5 N

-58-
ASOCIACIN EDUCATIVA PITGORAS Fsica

03. Hallar la mnima aceleracin de m2 para que la masa 10. Calcular el coeficiente de rozamiento si el ngulo de
m1 no resbale sobre m2. (g=9,8 m/s2) inclinacin es de 45

2 2 2
A) 9,8 m/s B) 81 m/s C) 49 m/s
2 2
D) 1,96 m/s E) 11 m/s A) 0,75 B) 1 C) 0,5
04. Cunto tardar un bloque en recorrer un plano D) 0,25 E) 0,45
inclinado de 13 m de longitud y 5 m de altura (= 0,4),
si parti del reposo? (g=10m/s )
2 11. Si el bloque A pesa 4 kg y el bloque B, 6 kg adems
A) 1 s B) 6,5 s C) 13 s el coeficiente de rozamiento cintico es 0,3, cul
D) 19,5 s E) 26 s debe ser el valor de F para arrastrar el bloque B hacia
2
la izquierda a velocidad constante. (g=10 m/s )
05. Entre que valores debe variar F para mantener el
bloque de 200 N en reposo:

A) 42 N B) 30 N C) 34 N
D) 54 N E) 14 N
A) 100N #F#160N B) 120N#F#160N 12. Hallar la aceleracin con la cual se mueve el bloque
C) 140N#F#180N D) 160N#F#200N 2
mostrado sobre el plano inclinado. (g = 10 m/s )
E) 80N#F#160N
06. La escalera homognea se encuentra en reposo
apoyada en una pared lisa y un suelo spero. Hallar
s del piso si la escalera se encuentra a punto de
resbalar

2 2 2
A) 3,5 m/s B) 5 m/s C) 2 m/s
2 2
D) 4 m/s E) 7 m/s
A) 3/4 B) 4/3 C) 3/8
13. Hallar la mxima aceleracin de M para que m no
D) 1/2 E) 5/8
resbale sobre M, (g = 10 m/s2)
07. Un bloque de madera se lanza con una velocidad de
4 m/s sobre una superficie horizontal spera. Qu
distancia recorre hasta detenerse?
2
(k = 0,2; g=10 m/s )
A) 4 m B) 6 m C) 8 m
D) 10 m E) 2 m

08. Si se sostiene un libro empujando horizontalmente A) 6 m/s


2
B) 4 m/s
2
C) 8 m/s
2

contra la pared vertical, si el libro pesa 0,5 N y el D) 2 m/s


2
E) 5 m/s
2

coeficiente de rozamiento esttico es 0,4 qu fuerza


se debe aplicar, para que el libro no caiga? 14. Si la masa del bloque es de 20 kg, hallar F, para
A) 3,2 N B) 2 N C) 2,8 N que el bloque est a punto de resbalar hacia arriba. (g
D) 4,5 N E) 1,25 N = 10 m/s2)
09. Determinar el coeficiente de rozamiento si el bloque
est a punto de resbalar.

A) 100 N B) 200 N C) 300 N


A) 0,7 B) 0,8 C) 0,75 D) 400 N E) 500 N
D) 0,56 E) 0,25

-59-
ASOCIACIN EDUCATIVA PITGORAS Fsica

15. Si slo existe rozamiento entre el bloque pequeo y 18. La quinta parte de la fuerza aplicada a un bloque es
el coche de masa M, hallar la mxima aceleracin igual a la fuerza de rozamiento cuyo coeficiente es
con la cual puede viajar el sistema para que el bloque 0,1. Determinar la aceleracin del bloque.
no deslice sobre el coche. A) 10% g B) 20% g C) 30% g
D) 40% g E) 50% g

19. Un cuerpo de 5 N de peso es transportado con


velocidad constante por F sobre una superficie
horizontal.
Si se quiere transportarlo con una aceleracin de
2
0,4 m/s , en cunto se debe incrementar a F?
2
(g=10 m/s )
A) F/m B) sg C) smg/M A) 0 B) 2 N C) 20 N
D) mg/M + m E) sMg/m D) 0,2 N E) 0,02 N

16. Un bloque de 100 N descansa sobre una superficie 20. Un elevador asciende tal como se muestra en la
horizontal de 0,1 y 0,2. Al mencionado bloque le figura. Hallar para que el bloque se mantenga en
aplicamos una fuerza horizantal F(Q) = t2 + t. Al cabo reposo respecto al elevador.
de qu tiempo empezar el movimiento?
(F en newton y t en segundos)
A) 2 s B) 3 s C) 4 s
D) 5 s E) 10 s

17. Hallar si debido a la fuerza F el cubo homogneo


est a punto de deslizar y volcar. Dar Tg

A) /(1 - 2) B) /3 C) /(2+ )
D) /4 E) (1 + 2)/ A) 1 B) 0,5 C) 0,25
D) 0,75 E) 0,45

TAREA
01. Un bloque de 10 N se encuentra apoyado sobre una A) 180 N B) 90 N C) 20 N
superficie horizontal rugosa de k=0,5 y s =0,6. Si D) 50 N E) 80 N
sobre el bloque acta una fuerza horizontal de 4 N,
hallar la fuerza de rozamiento esttico entre el bloque 04. Hallar que ngulo debe formar el plano inclinado
y la superficie horizontal. con la horizontal para que el bloque est a punto de
deslizar hacia abajo (s=0,75)

A) 4 N B) 6 N C) 8 N
D) 10 N E) 12 N
A) 30 B) 37 C) 53
02. Si al bloque mostrado se le aplica una fuerza
D) 60 E) 45
horizontal de F = 20 Newton, hallar la fuerza de
rozamiento sobre el bloque.
05. Hallar con qu aceleracin se mueve el bloque
(s = 0,8; k = 0,6. W = 100 N) 2
mostrado (k = 0,5; m = 10 kg; g = 10 m/s )

A) 10 N B) 20 N C) 30 N
D) 40 N E) 50 N 2 2 2
A) 1 m/s B) 2 m/s C) 3 m/s
2 2
03. Hallar el valor de F, si el bloque de 9 kg est a punto D) 4 m/s E) 5 m/s
de resbalar hacia abajo.
2 06. Un bloque se lanza con una velocidad de 20 m/s
(g = 10 m/s )
sobre un plano horizontal spero (k = 0,5). Hallar
la distancia que recorre sobre el plano hasta
detenerse. (g= 10 m/s2)
A) 20 m B) 30 m C) 40 m
D) 50 m E) 60 m

-60-
ASOCIACIN EDUCATIVA PITGORAS Fsica

07. Un bloque parte del reposo en B y tarda 2 s en llegar 09. Un bloque A de 5 kg de masa descansa sobre otro
al punto A. Determinar el coeficiente de bloque B de 15 kg. El coeficiente de rozamiento
rozamiento esttico para las superficies en contacto. esttico entre los bloques es 0,5 y se desprecia el
2
(g=10 m/s ) rozamiento entre el bloque B y el piso. Calcular el
mximo valor de F para que los bloques se muevan
2
juntos. (g=10 m/s )

A) 25 N B) 50 N C) 100 N
A) 0,125 B) 0,150 C) 0,625 D) 150 N E) 200 N
D) 0,75 E) 0,525
10. Una losa de 40 kg est sobre un suelo sin friccin.
08. Un bloque resbala por un plano inclinado 45 con la Un bloque de 10 kg se coloca encima de la losa. Los
horizontal en un tiempo doble del que le tomara el coeficientes de rozamiento entre la losa y el bloque
resbalar sobre un plano inclinado del mismo ngulo son 0,4 y 0,6. Si al bloque de 10 kg se le aplica una
pero sin friccin. Determinar el coeficiente de friccin. fuerza horizontal de 100 N, cul es la aceleracin de
2
A) 0,90 B) 0,25 C) 0,50 la losa? (g=10 m/s )
2 2 2
D) 0,75 E) 0,85 A) 2 m/s B) 6 m/s C) 4 m/s
2 2
D) 1 m/s E) 5 m/s

TRABAJO - POTENCIA

TRABAJO (W) Casos que se pueden presentar :

Magnitud escalar que caracteriza la accin que ejerce la


fuerza sobre el cuerpo al comunicarle cierto
desplazamiento. El trabajo caracteriza la accin de las
fuerzas capaces de modificar el mdulo de la velocidad
del cuerpo, es decir, que pueden acelerar o retardar el
movimiento del cuerpo considerado. Esto implica que slo
pueden realizar trabajo aquellas fuerzas que tengan un
componente en la direccin del movimiento, es decir una
componente tangente a la trayectoria en cada uno de sus
puntos.
El valor del trabajo se calcula conociendo la fuerza y la
trayectoria que recorre el cuerpo, tenindose los
siguientes casos bsicos :

1. Fuerza constante :

2. Fuerza de mdulo constante la cual es tangente a la


trayectoria en cada uno de sus puntos (espacio
recorrido)
W F = FT d

W F = FM d = FdCos

Unidad : joule (J) = N . m

-61-
ASOCIACIN EDUCATIVA PITGORAS Fsica

3. Para una fuerza de direccin constante cuyo mdulo TRABAJO DE LA FUERZA DE GRAVEDAD
vara con su posicin o distancia (x). En este caso se La fuerza de gravedad (peso) realiza un trabajo que posee
efecta la grfica de la fuerza con respecto a la las siguientes caractersticas:
posicin (x), el trabajo est representado por el rea
encerrada por la grfica con el eje de la posicin
entre la posicin inicial (xo) y la posicin final (xf)

1. El trabajo no depende de la trayectoria recorrida


4. Trabajo neto o total: (Wtotal): En general sobre un 2. El trabajo es igual al producto del peso con el
cuerpo actan 2 o ms fuerzas (sistema de fuerzas) desplazamiento vertical diferencia de alturas :
en este caso se define el trabajo total o neto como la
suma algebraica de los trabajos realizados por cada
una de las fuerzas que actan sobre el cuerpo. Este
trabajo es tambin igual al trabajo realizado por la 3. Se denomina energa potencial gravitatoria a :
fuerza resultante que acta sobre el cuerpo.
EP = mgh

Con la cual el trabajo del peso se puede expresar


como :

TRABAJO DE LA FUERZA ELSTICA


Se denomina fuerza elstica a la que se genera en un
Si el movimiento es acelerado ( | | aumenta) : W total (+) cuerpo deformado y la cual se opone a la deformacin, es
decir su sentido est que tiende a devolverle al cuerpo su
forma o dimensiones originales.
Si el movimiento es retardado ( | | disminuye) : W total (-)

Si el movimiento es uniforme ( | | = constante) : W total=0

TEOREMA DE LA ENERGA CINTICA


El trabajo realizado por la fuerza resultante (trabajo neto
o total) que acta sobre un cuerpo durante cualquier parte
de su movimiento es igual al cambio que experimenta la
energa cintica (EK) del cuerpo durante esa parte de su
movimiento

Para el caso de un resorte ideal, el cual presenta las


siguientes caractersticas :

1. Es de masa despreciable
2. Cumple la ley de Hooke tanto al ser estirado o
comprimido. Esta ley establece que la fuerza
deformadora (FD ) es directamente proporcional a la
deformacin (x = Lf - Lo) del resorte :

-1
Constante del resorte (N . m )
La energa cintica de un cuerpo en un punto del recorrido
que realiza est dada por : La relacin entre la fuerza deformadora (FD ) y la fuerza
recuperadora elstica (FE ) es :

-62-
ASOCIACIN EDUCATIVA PITGORAS Fsica

Luego de la ley de Hooke obtenemos que FD = Kx, por OBSERVACIONES :


lo que la grfica FD - x es una recta. A partir de esta 1. En general el trabajo de una fuerza depende de la
grfica obtenemos : trayectoria recorrida, o como el cuerpo o sistema
pasa de su posicin o estado inicial a su posicin o
estado final

2. Slo para el caso de ciertas fuerzas el trabajo es


independiente de la trayectoria, como por ejemplo
una fuerza constante. A estas fuerzas se denominan
fuerzas potenciales o conservativas.

POTENCIA (P)
Magnitud escalar la cual determina la rapidez con la cual
se realiza un trabajo. En el caso particular que el trabajo
se realice de manera uniforme, es decir se realizan
trabajos iguales en tiempos iguales cualesquiera, la
potencia es constante e igual al trabajo realizado en la
unidad de tiempo :

Por lo que el trabajo de la fuerza recuperadora elstica es:

-1
Unidad : Watt (W) = J . s

PROBLEMAS PROPUESTOS
01. El bloque de la figura experimenta durante su
movimiento una fuerza de rozamiento igual a 20 N. Si
F=50 N, qu trabajo realizan cada uno de ellos en el
traslado AB=10 m?

A) 500 J B) - 500 J C) 0
D) 200 J E) -50 J

05. Calcular el trabajo que realiza la fuerza constante


A) 500 J y - 2 000 J B) 300 J y - 100 J F=50 N al trasladar la masa m de A a B a lo largo de
C) 700 J y - 200 J D) 500 J y - 200 J la trayectoria curvilnea
E) 200 J y - 50 J

02. Calcular el mnimo trabajo realizado por la fuerza P


sabiendo que la fuerza de friccin es 20 newton y el
bloque se movi 10 m

A) 120 J B) 150 J C) 175 J


D) 200 J E) 225 J

06. Se tiene un plano inclinado con coeficiente de


A) 100 J B) 200 J C) 50 J rozamiento igual a 0,75; si el trabajo resultante
D) 150 J E) F.D. efectuado sobre el bloque m=2 kg, para trasladarlo
hasta el punto ms alto del plano es 260 joules el
03. Un cajn de 10 kg reposa sobre una plataforma valor de la fuerza F es : (g=10 m/s2)
horizontal spera (k=0,5). Sobre el cajn se aplica
una fuerza horizontal de modo que el cajn acelera
2
constantemente a razn de 2 m/s . Hallar el trabajo
de la fuerza aplicada hasta el instante en que la
2
velocidad del cajn sea de 8 m/s (g=10 m/s )

A) 1 120 J B) 540 J C) 860 J


D) 980 J E) 680 J

04. Si el sistema se mueve 10 m hacia la derecha, con


velocidad constante entonces, el trabajo realizado por A) 24 N B) 50 N C) 260 N
la tensin en el bloque de 2 kg es : D) 2 N E) 30 N

-63-
ASOCIACIN EDUCATIVA PITGORAS Fsica

07. El trabajo efectuado por la fuerza constante en A) 1 500 J B) 800 J C) 900 J


magnitud pero tangente a la curva que se muestra, D) 1 200 J E) 1 400 J
para ir de Ahasta B ser:
13. Hallar la potencia entregada al motor de un ascensor
cuando levanta la cabina con un peso total de 16 kN
a la velocidad constante de 3,6 km/h, sabiendo que la
eficiencia del motor de 80%

A) 20 kW B) 12 kW C) 18 kW
D) 24 kW E) 16 kW

14. La magnitud de F es 100 N y el coeficiente de


rozamiento cintico es 0,7 entre el bloque y la pared.
Determinar el trabajo neto que se realiza sobre un
bloque de peso 180 N, para un desplazamiento de
A) RFTg60 B) RFTg30 C) RFTg37 5 m en la vertical
D) RFTg53 E) RF

08. Una gra cuyo rendimiento es del 50% est instalada


a un motor cuyo rendimiento es del 80%. Si al motor
se le proporciona una potencia de 800 watt, calcular
la velocidad constante con la cual podr subir un
2
bloque de masa 1 kg (g=10 m/s )
A) 80 m/s B) 40 m/s C) 64 m/s
D) 32 m/s E) 10 m/s

09. El bloque de 10 kg es desplazado desde A hasta B


mediante la fuerza F=100 N y a velocidad constante. A) 120 J B) 160 J C) 480 J
Hallar el trabajo realizado por la fuerza de rozamiento D) 320 J E) 240 J
2
(g=10 m/s )
15. Sobre un cuerpo se aplica una fuerza F que
depende de la posicin x como se muestra en la
figura. Determinar el trabajo en joules realizado desde
x=0 hasta x=4 m

A) Cero B) +400 J C) -400 J


D) +600 J E) -600 J

10. Hallar la potencia mecnica de la fuerza F, si el


bloque sube con una velocidad constante de 10 m/s
2
(g=10 m/s )

A) -1, 0 B) 0, 0 C) 2, 0
D) 2, 5 E) 3, 0

16. Un bloque est sometido a la accin de cuatro


fuerzas constantes, que lo obligan a desplazarse en
A) 200 W B) 20 W C) 10 W lnea recta AB=10 m. Encuentra el trabajo neto
D) 5 W E) 0 realizado sobre l:
F1=10 N; F2=15 N; F3=10 N; F4=20 N
11. La dependencia de la fuerza en newton con la
distancia en metros est dada por la siguiente
ecuacin : F=3x+1. Hallar el trabajo de la fuerza de
x=3 m hasta x=5 m el cuerpo se mueve en el eje x
horizontal y F es horizontal
A) 20 J B) 26 J C) 30 J
D) 32 J E) 13 J

12. Un hombre carga sobre sus hombros un saco de


arena de 50 kg el cual debe levantar hasta una altura
de 6 m. Si el saco presenta un orificio donde la arena
sale uniformemente de modo que al llegar a su
A) 150 J B) 100 J C) 110 J
destino no queda ningn grano en el saco, qu
D) 200 J E) 800 J
trabajo realiz el hombre durante el recorrido?
2
(g=10 m/s )

-64-
ASOCIACIN EDUCATIVA PITGORAS Fsica

17. Una fuerza F desplaza al bloque de 20 N de peso 19. Hallar el trabajo de F para trasladar a la cadena de
a lo largo del plano inclinado donde el coeficiente de peso W lentamente desde la posicin mostrada
hasta que toda la cadena se encuentre sobre la
friccin es /5. Hallar el trabajo neto realizado sobre superficie horizontal lisa
el bloque entre A y B. (F = 50 N)

A) Wa/3 B) Wa/4 C) Wa/5


D) Wa/6 E) Wa/7

A) 840 J B) 640 J C) 420 J 20. El valor de la fuerza F vara de acuerdo a la


D) 360 J E) 210 J ecuacin :
F = 20 - 4x
18. Calcular la mxima velocidad con que un auto puede x se expresa en metros, F en newtons. Determinar
viajar en una pista, sabiendo que el aire y la pista el trabajo que realiza la fuerza F sobre el bloque
ejercen una resistencia de 2 kN y que el motor tiene desde x = 2 m hasta x = 8 m
una potencia de 150 H.P. con una eficiencia del 80%
(Dato 1 H.P = 746 W)
A) 52,28 m/s B) 41,76 m/s C) 45,66 m/s
D) 44,76 m/s E) 38,46 m/s
A) Cero B) 12 J C) 18 J
D) 24 J E) 36 J

TAREA
01. Una fuerza horizontal en N tiene la siguiente ley
F = 4x + 2. Si la fuerza desplaza un cuerpo sobre 04. La fuerza de resistencia que ofrece el agua al
el eje x, determine el trabajo realizado por la deslizamiento de una canoa es proporcional a la
fuerza cuando el cuerpo desplaza desde x = 3 m velocidad de la canoa. Si un motor de 10 c.v. hace
hasta x = 8 m marchar la canoa a 12 km/h, cuntos c.v. se
necesitarn para una velocidad de 24 km/h?
A) 120 J B) 130 J C) 140 J (Nota 1 c.v = 736 W)
D) 150 J E) 160 J A) 20 c.v B) 30 c.v C) 40 c.v
D) 50 c.v E) 25 c.v

02. Indicar lo incorrecto : 05. Segn el trayecto mostrado :


A) El trabajo se puede expresar en joule o kW - h.
B) Una fuerza variable perpendicular a la velocidad
no realiza trabajo.
C) El trabajo de la fuerza de gravedad (peso)
depende del desnivel entre las posiciones inicial
y final.
D) El trabajo es una magnitud escalar.
E) El trabajo es independiente de la trayectoria
recorrida.
03. El trabajo que realiza la fuerza de mdulo
( ) En el trayecto AB el trabajo del peso es : -mgh
constante, al desplazar el bloque una cierta distancia,
( ) En el trayecto BA el trabajo del peso es : +mgh
es W. Si se duplica el ngulo realizara un trabajo
( ) El trabajo neto en el trayecto ABA es cero
igual a 1,6 W para la misma distancia, hallar
podemos afirmar :
A) VVF B) FFF C) FVF
D) VFV E) VVV

06. Indicar la proposicin verdadera :


A) La fuerza de rozamiento siempre realiza trabajo
negativo
B) La fuerza de reaccin normal nunca realiza
trabajo
C) El trabajo neto siempre es positivo
D) La potencia mecnica es una magnitud vectorial
E) En un movimiento circular la fuerza centrpeta no
A) 90 B) 45 C) 60 realiza trabajo
D) 53 E) 37

-65-
ASOCIACIN EDUCATIVA PITGORAS Fsica

07. Un caballo jala un vagn con una fuerza F=180 N 09. Desde la parte superior de un plano que forma 53
que forma un ngulo de 30 con la horizontal , con con la horizontal y mide 5 m, es soltado un bloque de
-1
una rapidez V = 9 km.h . Qu trabajo hace el 2 kg, tardando 2 s en llegar a la base del plano. Hallar
caballo en 1 min?. Cul es la potencia que el trabajo realizado por la fuerza de friccin en todo el
2
desarrolla durante ese tiempo? descenso (g = 10 m/s )
A) 10 200 J; 375 W A) -11 J B) -22 J C) -33 J
B) 14 500 J; 225 W D) -44 J E) -55 J
C) 13 500 J; 225 W
D) 12 500 J; 425 W 10. Una fuerza =(4 + 3 ) N acta sobre una partcula
E) 11 500 J; 125 W conforme la misma se mueve en la direccin x
desde el origen hasta x = 5 m. Determine el trabajo
08. Indicar verdadero (V) o falso (F) : efectuado sobre la partcula por la fuerza
( ) Slo realizan trabajo las fuerzas que pueden A) 20 J B) 25 J C) 50 J
modificar el mdulo de la velocidad . D) 125 J E) Cero
( ) Si el trabajo total realizado sobre un cuerpo es
nulo, la fuerza resultante actuante sobre l es
cero.
( ) El trabajo total siempre ser mayor que el trabajo
realizado por una cualquiera de las fuerzas
actuantes.
A) VVV B) VFV C) VFF
D) FFF E) VVF

ENERGA

Se entiende por energa la capacidad o aptitud que tiene Casos :


un cuerpo para realizar un trabajo. Debido a esto la
energa de un cuerpo se medir por el trabajo que es a. Gravitatoria
capaz de efectuar en condiciones determinadas.
Si un cuerpo realiza trabajo su contenido energtico
disminuye en una cantidad equivalente al trabajo
efectuado. Por el contrario, si sobre el cuerpo se realiza
un trabajo su energa aumenta en la misma cantidad.
El origen de esta aptitud puede ser muy diferente de un
cuerpo a otro, por lo que la energa se manifiesta de
diferentes maneras a las que se denominan formas de la
energa.
En mecnica interesa conocer la posicin y rapidez de un
cuerpo, por lo que se tienen las siguientes formas de
energa mecnica :

1. Energa Cintica (E K ) : b. Elstica

Es la aptitud que tiene un cuerpo para realizar un


trabajo en virtud de su velocidad. Se mide por el
trabajo que habra que hacer sobre el cuerpo para
que adquiera la velocidad que posee, partiendo del
reposo.

2. Energa Potencial (E P ) :

Es la aptitud que tiene un cuerpo para efectuar un 3. Energa Mecnica (EM ) :


trabajo en virtud de su posicin o de su configuracin.
Se mide por el trabajo que hay que hacer sobre el Es la suma de la energa potencial y cintica que
cuerpo para hacerlo pasar de la posicin o posee un cuerpo en un punto del recorrido que realiza
configuracin tipo (EP = 0) a aquella en que se
encuentra.

-66-
ASOCIACIN EDUCATIVA PITGORAS Fsica

TEOREMA DEL TRABAJO Y LA ENERGA CINTICA TEOREMA DEL TRABAJO Y LA ENERGA MECNICA
El trabajo neto o total sobre una partcula es igual a la La suma de trabajos de las fuerzas no conservativas que
variacin de su energa cintica. actan sobre una partcula es igual a la variacin de su
W neto = EK = - energa mecnica.

W neto = m - m

W FNC = Suma de trabajos de las fuerzas no


FUERZAS CONSERVATIVAS
conservativas.
Son aquellas fuerzas cuyo trabajo entre dos posiciones no
depende de la trayectoria seguida por el cuerpo; las NOTA:
principales fuerzas conservativas son el peso (fuerza de
gravedad), las fuerzas elsticas, las fuerzas Si W FNC = 0 4
electromagnticas.

FUERZAS NO CONSERVATIVAS
Son aquellas cuyo trabajo s depende de la trayectoria
seguida por el cuerpo.
Ejemplo: La fuerza de rozamiento

PROBLEMAS PROPUESTOS
01. Una masa es lanzada verticalmente hacia arriba con A) 1 m/s B) 2 m/s C) 3 ms
una velocidad de 20 m/s. En qu relacin se D) 4 m/s E) 5 m/s
encuentran sus energas cintica y potencial cuando
2
se ha elevado 5 m? (g=10 m/s ) 05. Una cadena de 4 m y 10 kg, se encuentra
A) 3 B) 1/3 C) 2 inicialmente en la forma como muestra la figura.
D) 1/2 E) 1 Determinar la velocidad de la cadena cuando el
ltimo eslabn abandona el plano inc linado
2
02. Un cuerpo de 20 kg se lanza verticalmente hacia (g = 10 m/s )
arriba con una velocidad de 60 m/s. Calcular a qu
altura la energa cintica del cuerpo se ha reducido al
40% de la que tena inicialmente en el lanzamiento .
2
(g=10 m/s )
A) 24 m B) 108 m C) 72 m
D) 36 m E) 82 m

03. Si la masa m parte del reposo, hallar la relacin de A) 2 m/s B) 3 m/s C) 4 m/s
velocidades en A y B (VB/VC ) D) 5 m/s E) 8 m/s

06. Un obrero que est fijando ladrillos es abastecido por


un compaero situado 3,2 m, debajo de l. Si los
ladrillos le llegan con una velocidad de 6 m/s, qu
porcentaje de energa malgasta el compaero?
A) 24% B) 36% C) 48%
D) 64% E) 49%

07. La mxima velocidad que adquiere la esfera al ser


soltada en A es de 10 m/s. Hallar la fuerza de
A) 1/4 B) 1/3 C) 1/2 reaccin en B que acta sobre la esfera, si no existe
D) 1/5 E) 1/8 rozamiento.
2
04. En la figura mostrada, el resorte de 30 cm jala desde (g = 10 m/s ; m = 5 kg)
el reposo a un collarn de 4 kg de masa. Qu
velocidad le comunica el resorte al collarn cuando
ste pasa por B? (K = 100 N/m)

A) 180 N B) 90 N C) 270 N
D) 220 N E) 140 N

-67-
ASOCIACIN EDUCATIVA PITGORAS Fsica

08. Desde la posicin mostrada en la figura se suelta un 14. El bloque mostrado de 6 kg es empujado lentamente
cuerpo de masa m. Para qu posicin o ngulo desde A hasta B por una fuerza horizontal F, y
respecto a la horizontal el cuerpo adquiere la mitad constante. Qu trabajo realiz dicha fuerza?
2
de su velocidad mxima?. Dar como respuesta el (g=10 m/s )
seno del ngulo.

A) 2/3 B) 3/4 C) 5/8


D) 4/7 E) 7/9 A) 360 J B) 240 J C) 180 J
D) 120 J E) 60 J
09. Un cuerpo de 3 kg de masa es abandonado en A. 15. Un cuerpo es dejado en libertad en A y resbala por
Hallar la reaccin de la superficie semicircular lisa en un canal, llegando a B con una velocidad de 15 m/s.
2
B (g = 10 m/s ) Si su masa es de 4 kg, qu trabajo hizo el
rozamiento sobre l en dicha trayectoria? (g=10 m/s2)

A) 90 N B) 60 N C) 30 N
D) 120 N E) 75 N
A) -350 J B) -450 J C) -800 J
10. Con qu velocidad mnima se debe impulsar la esfera D) -900 J E) -1 000 J
desde A para que pueda elevarse justo hasta C.
AB es rugoso 16. Determinar el trabajo de la fuerza del rozamiento
cintico en el tramo AB, si el cuerpo de 3 kg baja a
velocidad constante y se encuentra sujeto a una
2
fuerza F1=5 N y h = 6 m (g = 10 m/s )

A) 2gH B) C)
D) E)

11. Una paracaidista se deja caer de un helicptero


detenido a 200 m de altura y llega al suelo a 5 m/s. Si
la masa del paracaidista y su equipo es 90 kg el
trabajo ejercido por la fuerza del aire es :
2
(g=10 m/s )
A) -1125 J B) -18 000 J C) 1125 J A) -50 J B) -120 J C) -180 J
D) -16875 J E) 178 875 J D) -130 J E) -70 J

12. Una bala de 20 g con velocidad de 500 m/s penetra 17. En la figura mostrada los planos inclinados poseen
25 cm dentro de un bloque de madera hasta rozamiento. Si un cuerpo soltado en A se detiene en
detenerse. Cul es la fuerza media producida por el C y la fuerza de friccin es igual a 1/10 del peso del
bloque? cuerpo en todo el trayecto. Calcular la altura h
A) 10 000 N B) 5 000 N C) 2 500 N
D) 6 500 N E) 8 000 N

13. Un pequeo bloque se deja caer desde A, resbala por


una superficie circular lisa y luego sube por un plano
inclinado spero hasta el punto B. Hallar h

A) 3 m B) 5 m C) 6 m
D) 7 m E) 3,5 m

A) 0,8 m B) 0,9 cm C) 0,9 m


D) 0,6 m E) N.A.

-68-
ASOCIACIN EDUCATIVA PITGORAS Fsica

18. El cuerpo de 1 kg es soltado en x. La aceleracin 20. Un pequeo cuerpo es soltado en A llegando con
en el punto y ser : velocidad cero al punto D. Hallar de la superficie
horizontal (Las superficies curvas son lisas)

A) g B) g C) 3g
D) 2g E) g

19. Si la esfera soltada en A por efectos de rozamiento A) 9/2 B) 1/4 C) 1/3


se detiene en C, hallar (AB = BC) D) 1/2 E) 1

A) /3 B) 1/3 C) /2
D) 1/2 E) /6

TAREA
01. Si el cuerpo es abandonado en A, hallar su
velocidad en B. (H=10 m)

A) 2 m B) 2,5 m C) 3 m
D) 3,5 m E) N.A.

04. Desde el piso se lanza una pelota con una velocidad


A) 14 m/s B) 28 m/s C) 7 m/s de 50 m/s, describiendo una parbola. Si en la parte
D) 56 m/s E) 64 m/s ms alta su velocidad es de 30 m/s, cul es la altura
mxima que se alcanz en el movimiento?
2
02. Un bloque de masa 2 kg parte de una altura de 5 m (g = 10 m/s )
con velocidad inicial de 5 m/s y comprime un resorte
1 m. Cul es la constante del rozamiento? A) 10 m B) 20 m C) 40 m
2
(g = 10 m/s ) D) 60 m E) 80 m

05. En la figura un anillo de masa m se desplaza


libremente en la varilla doblada. Cunto vale el
ngulo , si m parte del reposo en el punto A y llega
a B con velocidad ?

A) 250 N/m B) 125 N/m C) 100 N/m


D) 50 N/m E) 300 N/m

03. Hallar h de tal manera que la reaccin en B sea 2 A) 30 B) 37 C) 45


veces su peso m es soltado en A D) 53 E) 60

-69-
ASOCIACIN EDUCATIVA PITGORAS Fsica

06. Si el cuerpo se suelta en A, calcular el trabajo 09. Una bala disparada con una velocidad de 300 m/s se
realizado por el peso en el tramo , considerando: incrusta en un gran bloque de madera hasta una
> profundidad de 30 cm. Si la bala se disparase con la
misma velocidad contra una lmina de la misma
madera de 10 cm de espesor, calcular con qu
velocidad se mover la bala al salir de la madera, si
la oposicin que ejerce es constante.

A) 100 m/s B) 100 m/s


C) 100 m/s D) 100 m/s
E) 200 m/s

10. Se lanza una esferita de masa m a travs de la


A) mgL(Cos - Cos) tubera curva mostrada; luego ser correcto afirmar
B) mgL(Cos - Cos) (la tubera est en un plano vertical).
C) mgLCos I. Entre la tubera y la esferita existe rozamiento.
D) mgLCos II. Tiene que existir una fuerza externa a favor del
E) mgL movimiento para que no pierda rapidez la esferita.
III. Es imposible que no pierda energa cintica la
07. Qu mnima rapidez debemos comunicar esferita en su recorrido de A a B
horizontalmente a la esferita que cuelga del hilo de
50 cm de largo para que pueda realizar una vuelta
2
completa alrededor del punto fijo O? (g=10 m /s )

A) 2 m/s B) 5 m/s C) 10 m/s


D) 8 m/s E) 15 m/s
A) Slo I B) Slo II C) Slo III
08. Se lanza una piedra verticalmente hacia arriba con D) I y III E) I y II
una velocidad de 7 m/s sabiendo que se pierde el
20% de energa por el rozamiento con el aire.
Calcular hasta que altura lleg
2
(g = 10 m/s )
A) 1,50 m B) 2,6 m C) 1,56 m
D) 1,96 m E) 3,1 m

CANTIDAD DE MOVIMIENTO - IMPULSO - CHOQUES

INTRODUCCIN direccin coincide con la velocidad y mide el grado de


En el captulo de dinmica hemos explicado el movimiento oposicin que presentan los cuerpos contra los agentes
de los cuerpos utilizando bsicamente la 2da Ley de externos que pretenden alterar su velocidad y/o masa
Newton y en el captulo de Energa usando el principio de
la conservacin o el teorema de las fuerzas no
conservativas. Entonces podemos decir que hasta cierto
punto ha sido sencillo tal estudio, pues el nmero de
cuerpos participantes fue muy reducido, pero cuando se
presentan interacciones entre dos o ms cuerpos debido
a colisiones, explosiones, etc. Los conocimientos
adquiridos hasta el momento quedan muy limitados
debido a la manera compleja que aparecen las fuerzas
por lo cual se hace necesario estudiar nuevas:
magnitudes y principios fsicos como son la cantidad de
movimiento, el impulso, etc, que veremos a continuacin

CANTIDAD DE MOVIMIENTO O MOMENTUM LINEAL

Es una magnitud fsica de tipo vectorial que se calcula


como el producto de la masa por la velocidad, su

-70-
ASOCIACIN EDUCATIVA PITGORAS Fsica

IMPULSO, MPETU O IMPULSIN FUERZAS INTERNAS Y EXTERNAS


Las fuerzas que actan en un sistema se pueden
Es una magnitud fsica de tipo vectorial que se calcula clasificar en internas y externas. Si un cuerpo del sistema
como el producto de la fuerza por el intervalo de tiempo ejerce una fuerza sobre otro que tambin pertenezca al
que ella acta, su direccin coincide con el de la fuerza y sistema, aquella ser una fuerza interna. Por otra parte, si
nos indica el grado de efectividad que posee una fuerza la fuerza que acta sobre un cuerpo del sistema fuese
para cambiarle la velocidad al cuerpo sobre el cual acta ejercida por un agente que no pertenece al sistema, se
tratar entonces de una fuerza externa.
Por ejemplo, suponga que hubisemos elegido un sistema
de partculas constituido por dos bolas, una blanca y otra
roja, en una mesa de billar. Al golpear con el taco la bola
blanca, sobre el sistema habr actuado una fuerza
externa. Si dicha bola choca con la roja, las fuerzas que
una ejercen sobre la otra sern internas. Si la bola blanca
hubiese chocado con otra, por ejemplo, una amarilla, la
fuerza que recibira de esta ltima sera una fuerza
externa, pues el sistema est constituido nicamente por
las bolas blanca y roja.

OBSERVACIN :

NOTA : Las fuerzas internas pueden producir variaciones en las


Por lo general la fuerza es variable en mdulo, por lo cual cantidades de movimiento de las partculas de un sistema,
en la frmula anterior va la fuerza media FM (valor medio pero no producen variacin en la cantidad de movimiento
de la fuerza), que produce el mismo impulso que la fuerza total del mismo
variable en el mismo intervalo de tiempo

TEOREMA DEL IMPULSO Y LA CANTIDAD DE


MOVIMIENTO
Fuerzas internas de accin y reaccin, producen impulsos
El impulso que ejerce la fuerza resultante sobre un cuerpo de igual magnitud, pero de sentidos contrarios
durante cierto intervalo de tiempo, se invierte en cambiarle
su cantidad de movimiento CONSERVACIN DE LA CANTIDAD DE MOVIMIENTO
Si la resultante de todas las fuerzas externas que actan
Si: F : Fuerza resultante externa sobre un cuerpo o sistema de cuerpos es nula, entonces
la cantidad de movimiento total de este cuerpo o sistema
Y se conservar, o sea, se mantendr constante
Y Sabemos :

. t = m -m
Si : FR = 0 Y

OBSERVACIN :
En general :
Originalmente, Newton enunci su segunda ley en
trminos de la cantidad de movimiento y fue l quien le
dio dicho nombre, el enunciado es el siguiente : La
rapidez de cambio de la cantidad de movimiento de un
cuerpo es proporcional a la fuerza resultante aplicada a OBSERVACIN :
l Debemos indicar que las condiciones para la
conservacin de la cantidad de movimiento son muchas
mas amplias que las condiciones para la conservacin de
la energa mecnica. sta no vara si slo actan fuerzas
conservativas. La cantidad de movimiento por otra parte,
se conservar an cuando estn actuando fuerzas
disipativas, como la friccin, pues estas fuerzas son
internas al sistema

-71-
ASOCIACIN EDUCATIVA PITGORAS Fsica

FUERZAS IMPULSIVAS OBSERVACIN :


Cuando estalla una bomba o cuando dos automviles En los choques inelsticos y plsticos parte de la energa
chocan, as como en algunos otros casos semejantes, cintica perdida se utiliza para producir deformaciones o
aparecen entre los cuerpos fuerzas muy intensas, pero bien se transforma en calor.
que actan durante un intervalo de tiempo muy breve. Por
ejemplo, cuando un jugador de ftbol patea un baln, la CENTRO DE MASA
fuerza de interaccin entre ste y el pie del jugador es del Cuando tratamos el caso de un cuerpo finito, o sea, un
4
orden de 10 N y dura casi 0,01 s. Estas fuerzas se cuerpo que tiene tamao, hemos supuesto que se puede
denominan fuerzas impulsivas. Debemos observar que representar en forma aproximada por una partcula
estas fuerzas, en general, producen enormes puntual, o que slo tiene movimiento de traslacin. Sin
aceleraciones en los objetos que actan, es decir, al ser embargo, los cuerpos reales finitos pueden experimentar
aplicadas en intervalos de tiempo muy breves, producen movimiento rotatorio o de otros tipos tambin. Por ejemplo
variaciones considerables en la velocidad de dichos el movimiento de traslacin y rotacin.
cuerpos. Las observaciones del movimiento de los cuerpos indican
de cuando gira un cuerpo, o cuando hay varios cuerpos
CHOQUES que se muevan en relacin uno con otro, hay un punto
Un choque es aquel fenmeno que se produce cuando que se mueve en la misma trayectoria que seguira una
dos cuerpos interactan por contacto durante un tiempo partcula si se sujetara a la misma fuerza neta. A este
muy breve y se originan fuerzas impulsivas o impulsoras punto se le llama centro de masa (cm). El movimiento
variables y muy intensas. general de un cuerpo finito, o sistema de cuerpos, se
En todo choque la cantidad de movimiento permanece puede definir como la suma del movimiento de traslacin
constante del centro de masa y los movimientos rotatorio, vibratorio
y de otros tipos con respecto al centro de masa.
COEFICIENTE DE RESTITUCIN (e) El movimiento de una clavadista es traslacin pura en (a),
pero es de traslacin y rotacin en (b).
(a) (b)

Imaginemos a un sistema formado por n partculas de


masas m1, m2, m3 ... mn
(x1; y1) coordenadas de m1
(x2; y2) coordenadas de m2
(xn; yn) coordenadas de mn

Cada velocidad con signo

se cumple : 0 # e # 1

OBSERVACIN :
Para un choque oblicuo las velocidades deben tener la
direccin perpendicular a la tangente comn en el punto
de contacto, es decir, las velocidades se descomponen en
las direcciones de las rectas tangente a los cuerpos y
perpendicular a la tangente.

CHOQUE ELSTICO (e = 1).- Se conserva la energa


cintica del sistema de cuerpos que impactan. Adems
los cuerpos despus del choque no quedan deformados

CHOQUE INELSTICO (0 < e < 1).- No se conserva la


energa cintica del sistema y los cuerpos despus del
choque quedan deformados.
Donde ( ) son las coordenadas del centro de
CHOQUE COMPLETAMENTE INELSTICO O
PLSTICO (e = 0).- Tampoco se conserva la energa masas del sistema de partculas.
cintica y los cuerpos despus del choque quedan
pegados y se mueven juntos con la misma velocidad OBSERVACIN :
Para cuerpos no muy grandes el centro de gravedad y el
centro de masa coinciden.

-72-
ASOCIACIN EDUCATIVA PITGORAS Fsica

CENTRO DE MASA Y MOVIMIENTO


DE TRASLACIN
La cantidad de movimiento total de un sistema de
partculas es igual al producto de la masa total (M) por la
velocidad del centro de masa del sistema.

PROBLEMAS PROPUESTOS
01. Un rifle de 5 kg dispara una bola de 100 g con una lanza verticalmente respecto a tierra un proyectil de
velocidad de 200 m/s. Hallar la velocidad de retroceso masa m, hallar la velocidad final del carro.
del rifle Despreciar todo tipo de rozamiento
A) 1 m/s B) 2 m/s C) 3 m/s
D) 4 m/s E) 5 m/s
02. Una persona de 80 kg se desplaza con una velocidad
de 2 m/s al encuentro de una plataforma de 120 kg
que se desplaza con una velocidad de 6 m/s. Si la
persona salta sobre la plataforma. Hallar la velocidad
del conjunto despus del salto
A) 56 m/s B) 5,6 m/s C) 2,8 m/s A) V B) V C) V
D) 28 m/s E) 14 m/s
D) V E) V
03. Se dispara una bala como se indica, si la bala queda
incrustada hallar la velocidad inicial de la bala para
que el conjunto suba 10 cm 07. En la figura mostrada m1 < m2; hallar el
desplazamiento x de la balsa de masa m3 cuando
m1 y m2 lleguen a los extremos de la balsa.

A) 11 m/s B) 12 m/s C) 13 m/s


D) 14 m/s E) 15 m/s

04. Un vehculo de 1 000 kg acelera del reposo a


72 km/h en 5 s
v Qu impulso se aplica al vehculo durante los 5 A) B)
s?
v Si se supone que la fuerza tangencial es
constante, cul es la magnitud de la fuerza? C) D)
A) 2 kN.s y 4 kN B) 4 kN.s y 2 kN
C) 20 kN.s y 4 kN D) 40 kN.s y 2 kN E) No se sabe
E) 30 kN.s y 2 kN
08. En la figura mostrada m1 = 2 kg y m2 = 6 kg. Si
05. Un hombre de masa m se encuentra parado sobre despus del choque la esferita avanza con
una plataforma de masa M=4m que se mueve con
velocidad constante V. Si sbitamente la persona U2=10 m/s, calcular la velocidad de la esferita
corre con velocidad V con respecto a la plataforma.
Indique con qu velocidad se mueve la plataforma en
este caso.

A) V B) 2V C) V/2
D) V/5 E) 4V/5
A) 10 m/s; a la derecha B) 10 m/s; a la izquierda
06. La figura muestra un can asociado a un auto C) 5 m/s; a la derecha D) 5 m/s; a la izquierda
movindose horizontalmente con una velocidad V E) Cero
por inercia. La masa total del sistema es M. Si se

-73-
ASOCIACIN EDUCATIVA PITGORAS Fsica

09. Si las esferitas y , luego del choque quedan


pegados y tienen una velocidad de 2 m/s tal como
muestra la figura, determinar la masa m2"
Dato m1 = 3m

A) 60 m B) 50 m C) 40 m
D) 30 m E) 70 m

14. 2 esferas A y B de igual masa realizan un choque


inelstico (e = 0,8) tal como muestra la figura
determinar la velocidad de cada esfera despus del
choque
A) 3 m B) 0,75 m C) m
D) 5 m E) 6 m

10. Una esferita de cera A se mueve con VA = 4,5 m/s


luego de impactar plasticamente con B, qu
cantidad de calor se disipa?
2
(mA = 2 kg; mB = 1 kg; g = 10 m/s ) A) 1 m/s; 9 m/s B) 8 m/s; 2 m/s C) 2 m/s; 8 m/s
D) 9 m/s; 1 m/s E) 3 m/s; 7 m/s

15. En la figura las masas (M = 200 g y m=100 g),


despus de producirse el choque completamente
plstico. Qu altura alcanzan respecto a su posicin
2
original? (g = 10 m/s )

A) 3,75 joule B) 6,75 joule C) 13,5 joule


D) 20,25 joule E) 10,75 joule

11. Una esfera es soltada desde una altura de 10 m.


Hallar la mxima altura que alcanzar luego de su
primer rebote e = 0,5

A) 2,5 m B) 5 m C) 10 m
D) 12,5 m E) 20 m
A) 1,22 cm B) 2,22 cm C) 1,92 cm
12. Una bola de masa m y velocidad V pega contra
D) 1,52 cm E) 1,53 cm
una pared perpendicular y rebota con la misma
velocidad . Si el tiempo que dura el choque es t,
16. Se suelta la esferita desde la posicin mostrada,
cul es la fuerza ejercida por la bola sobre la pared?
luego del choque determinar hasta qu ngulo como
mximo logra desviarse respecto de la vertical
Considere e = 0,2

A) B) mV Sent C)

D) mV Sen E)
A) 30 B) 37 C) 53
D) 45 E) 60
13. Dada la figura encontrar la distancia que los separa
cuando la bolita de mayor masa toque la pared. El
choque es inelstico (e=0,25); desprecie el radio de
las esferas

-74-
ASOCIACIN EDUCATIVA PITGORAS Fsica

17. En el billar, una bola impacta sobre una banda tal 19. Si la masa M fue soltada desde una altura H y
como muestra la figura. Calcular la pendiente del realiz un choque elstico con m, determinar la
ngulo altura mxima alcanzada por m
(M = )

A) H/25 B) H/16 C) 4H/9


D) 4H/25 E) 3H/25

20. Dos cuerpos se deslizan libremente por una varilla


horizontal tal como se indica. Determinar el mdulo
de la fuerza media de interaccin entre los cuerpos si
la duracin del choque es 0,005 s
A) 1 B) 2/3 C) 3/2 mA = 8 kg
D) 3/4 E) 4/3 mB = 2 kg
El coeficiente de restitucin es 0,5
18. La esfera de 1 kg parte del reposo desde 20 m de
altura respecto de la tabla de 5 kg. Si luego del
choque rebota hasta 5 m, calcular el coeficiente de
restitucin.
(g = 10 m/s2)

A) 480 N B) 2 400 N C) 4 800 N


D) Cero E) 48 N

A) 0,1 B) 0,2 C) 0,5


D) 0,8 E) 1

TAREA

01. Una explosin rompe un objeto en 3 partes, una de


ellas de 2 kg de masa sale disparada con una
velocidad de 20 m/s formando un ngulo recto con la
parte de 3 kg con una velocidad de 10 m/s
Cul es la velocidad del tercer pedazo cuya masa es
1 kg?

A) 10 m/s B) 20 m/s C) 50 m/s


D) 70 m/s E) 80 m/s

02. El proyectil de 10 kg y V = 20 m/s se incrusta en el


bloque de M = 90 kg. Calcular la velocidad del
sistema despus del choque

A) 1 m/s B) 2 m/s C) 5 m/s


D) 4 m/s E) 8 m/s

-75-
ASOCIACIN EDUCATIVA PITGORAS Fsica

03. Un bloque de m = 0,02 kg se desliza por una 07. Si la energa se conserva, hallar las velocidades
despus del choque en (m/s)
superficie lisa y con velocidad = (-6, 0) m/s, desde
el instante t = 0 s, experimenta la accin de una
fuerza variable, tal como se indica en el grfico (F - t).
Calcular la velocidad del cuerpo para t = 0,12 s

A) Cero; 10 B) 5; 5 C) Cero; 5
D) 4; 10 E) 10; 10

08. Una esfera es lanzada horizontalmente contra una


superficie inclinada, si logra rebotar verticalmente
segn se muestra, calcular
A) V = 30 m/s B) V = 36 m/s C) V = 42 m/s = ye=
D) V = 24 m/s E) V = 6 m/s

04. Una esferita de 1 kg desliza sobre una superficie

horizontal con una velocidad = (6,8) m/s incidiendo


sobre una pared vertical con un ngulo de incidencia
de 37. Si luego del impacto que dur 0,1 s pierde el
25% de su rapidez inicial y adems su velocidad de
rebote es perpendicular a su velocidad inicial, qu
fuerza media aplica la pared vertical a la esferita?
A) 100 N B) 125 N C) 200 N
D) 300 N E) 500 N A) 30 B) 37 C) 53
D) 8 E) 16
05. Una raqueta logra golpear una pelota de tenis de
200 g desviando su direccin segn como se indica. 09. Calcular la distancia que se aparta la tabla de la
Si V1 =35 m/s y V2 = 75 m/s, qu fuerza media pared cuando la persona desde la posicin mostrada
experiment la pelota, si la duracin del contacto fue camina a velocidad constante hasta el otro extremo.
0,050? Masa de la tabla 5m
Masa de la persona m

A) 400 N B) 300 N C) 500 N A) L/6 B) L/5 C) 2L/3


D) 700 N E) 900 N D) 2L/4 E) L/4

06. Una bola de caucho impacta sobre un piso horizontal 10. Se muestra una grfica del comportamiento de una
segn se muestra en la figura. Calcular el coeficiente fuerza que ejerce la pared sobre la esfera durante el
de restitucin choque. Determinar la velocidad inmediatamente
= 0,5 despus del choque

A) 0,55 B) 0,6 C) 0,8


D) 0,9 E) 0,1 A) 16 m/s B) 8 m/s C) 5 m/s
D) 4 m/s E) 32 m/s

-76-
ASOCIACIN EDUCATIVA PITGORAS Fsica

GRAVITACIN UNIVERSAL
GRAVITACIN UNIVERSAL Newton estableci que dos cuerpos cualesquiera
experimentan una atraccin mutua y la fuerza que define
Muchas veces nos hemos hecho la pregunta, por qu los
cuerpos soltados desde cierta altura se precipitan a tierra? dicha atraccin se denomina: Fuerza gravitatoria ( ).
o por qu la Luna se mantiene en rbita alrededor de la
Tierra? o por qu los planetas se mueven alrededor del
Sol?. Todas estas preguntas tienen hoy en da una
respuesta satisfactoria gracias a la Ley Universal de la
Gravitacin. El valor de la fuerza gravitacional es directamente
Desde hace muchos siglos atrs (400 a.C.) que el ser proporcional al producto de las masas de los cuerpos e
humano se interes por entender el movimiento de los inversamente proporcional al cuadrado de la distancia que
cuerpos celestes; fueron los griegos los primeros en este los separa.
campo.
Si bien los instrumentos pticos de precisin tardaron
todava 2 000 aos en aparecer la simple observacin del
cielo nocturno haba proporcionado los datos suficientes
respecto al movimiento de los cuerpos celestes, para G: Constante de gravitacin
establecer diversas teoras al respecto. G:
Las primeras basadas en la hiptesis hechas por
Aristteles, consideraban a la Tierra como el centro del
Universo (Sistema geocntrico), lo cual haca que Para el caso de la atraccin entre los cuerpos y la Tierra
describir el movimiento de los cuerpos celestes fuese la fuerza gravitacional, tambin se denomina fuerza de
sumamente complicado. Esta teora con el tiempo sufri
ciertas modificaciones, pero an tena serias limitaciones gravedad ( ).
para explicar con sencillez una serie de fenmenos que
se observaban.

Fue alrededor del ao 1 600 d.C. cuando un joven


estudiante polaco Nicols Coprnico tuvo la audacia de
establecer una teora en la cual se considera al Sol como
el centro de nuestro sistema, y los dems planetas
incluido la Tierra orbitando alrededor de l. La teora de
Coprnico no era del todo satisfactoria, ya que l slo
aceptaba como trayectoria para los planetas una
circunferencia y eso hacia que su teora tenga muchas
limitaciones, pero estableci las bases para que luego
Johannes Kepler y posteriormente Isaac Newton dieran la g = Valor de la aceleracin de la gravedad a una altura
gran respuesta a: Cmo funciona el Universo? h respecto de la superficie terrestre.
24
MT : Masa de la Tierra MT = 6.10 kg
6
RT : Radio terrestre RT = 6,4.10 m
NEWTON Y LA LEY DE LA GRAVITACIN
OBSERVACIN :
Muchas veces pensamos que fue Newton quien En las inmediaciones de la superficie terrestre: h << R
estableci el concepto de gravedad, pero antes de
Newton ya se conoca el concepto de gravedad.
El gran acierto de Newton fue establecer que los cuerpos
Cabe resaltar que antes que Newton enunciara la Ley
que se precipitaban a tierra, la Luna orbitando alrededor
Universal de la Gravitacin, un astrnomo alemn
de la Tierra, los planetas movindose alrededor del Sol,
Johannes Kepler ya haba establecido tres leyes
son todas manifestaciones de una atraccin universal que
relacionadas con el movimiento de los planetas,
experimentan los cuerpos.
basndose en las observaciones, hechas por su maestro
Tycho Brahe, dichas leyes son reconocidas actualmente
como las leyes de Kepler, pero tener presente que estas
leyes se pueden deducir de la ley establecida por Newton.
Extendi el concepto de gravedad a todo el Universo!
Veamos a continuacin las leyes de Kepler.
Primera Ley (Ley de las rbitas)
Todos los planetas se trasladan alrededor del Sol,
describiendo trayectorias elpticas, donde el Sol ocupa
uno de los focos de la elipse.

-77-
ASOCIACIN EDUCATIVA PITGORAS Fsica

Segunda Ley (Ley de las reas)

Durante el movimiento que desarrolla un planeta, el radio


vector que une el Sol con el planeta barre reas iguales,
cuando los intervalos de tiempo son iguales.
Es decir el rea barrida por el radio vector es proporcional
al tiempo empleado.

Para el caso de dos planetas A y B.

Notar que si tAB = tCD ; entonces A1 = A2

Tercera Ley (Ley de los periodos) La importancia del trabajo de Newton radica en que
El periodo (T) de un planeta (tiempo que emplea en dar proporcion una explicacin sencilla a diversos
una vuelta alrededor del Sol) elevado al cuadrado es fenmenos como por ejemplo las mareas ocenicas, y
proporcional al cubo del semieje mayor de la rbita adems permiti predecir el descubrimiento de algunos
elptica que describe. planetas de nuestro Sistema Solar. (Neptuno y Plutn)

PROBLEMAS PROPUESTOS
01. Para que se d cuenta de la pequeez de la fuerza 06. Un cuerpo pesa en la Tierra 180 N. Determine Qu
de atraccin entre dos objetos comunes, calcule la peso tiene en un planeta cuya masa es el doble de la
fuerza con la que dos personas se atraen terrestre y cuyo radio es el triple del radio de la
(gravitacionalmente). Para simplificar los clculos, Tierra?
suponga que las masas de las personas son A) 20 N B) 40 N C) 180 N
m1=m2=100 kg, que la distancia entre ellas es d=1 m D) 45 N E) 80 N
y considere que G=10 -10 N.m2/kg2
-3 -6 -9
A) 10 N B) 10 N C) 10 N 07. Cul es la aceleracin de la gravedad en la
6 3
D) 10 N E) 10 N superficie del Sol cuyo radio equivale a 100 radios
terrestres y cuya densidad media es 1/4 de la
02. Como los cuerpos celestes tienen masas densidad terrestre?
2 2 2
enormes, la fuerza gravitacional entre ellos es muy A) 125 m/s B) 375 m/s C) 245 m/s
2 2
grande (aun cuando la distancia que los separa D) 300 m/s E) 250 m/s
es, tambin enorme). A fin de comprobar lo
anterior, calcule el valor aproximado de la fuerza 08. Si consideramos que la separacin entre la Tierra y la
de atraccin entre la Tierra y la Luna, Luna es d y que adems la masa de la Tierra es
considerando que G =10-10 N.m2/kg2, masa de la igual a 81 veces la masa de la Luna, decir en qu
25 23
Tierra MT =10 kg, masa de la Luna ML = 10 kg y lugar del espacio entre estos dos cuerpos se puede
8
distancia de la Tierra a la Luna d= 10 m. colocar una pequea masa para que se encuentre en
22 20 10
A) 10 N B) 10 N C) 10 N equilibrio. Dar respuesta como la distancia de aquel
15 25
D) 10 N E) 10 N punto a la Luna
A) 0,9 d B) 0,6 d C) 0,1 d
03. Si en la superficie terrestre la aceleracin de la D) 0,2 d E) 0,5 d
2
gravedad es 9,8 m/s , determinar el valor de dicha
aceleracin en un punto situado a una altura igual a
3R/4, de la superficie terrestre (R = radio de la 09. Dos cuerpos se atraen con una fuerza de 800 N. Si
Tierra) uno de ellos duplica su masa y el otro la triplica y la
A) 6,85 m/s2 B) 3,42 m/s2 C) 3,2 m/s2 distancia entre ellos se cuadriplica, cul es la nueva
2 2
D) 4,8 m/s E) 1,6 m/s fuerza de atraccin entre ellos?
A) 100 N B) 200 N C) 300 N
04. En qu relacin estn la gravedad de dos puntos, D) 400 N E) 600 N
situados uno a una profundidad igual a la tercera
parte del radio terrestre y el otro a una altura doble 10. Determinar la fuerza resultante que acta sobre la
del radio terrestre ambos respecto de la superficie de masa m
la Tierra
A) 1; 2 B) 9; 2 C) 1: 3 (Considere : )
D) 3; 2 E) 6; 1

05. La masa de la Luna es 1/80 de la masa de la Tierra y


su radio 1/4 de sta. Cul es la aceleracin de la
gravedad en la superficie de la Luna? A) B) C)
(g = gravedad en la superficie terrestre)
A) g/20 B) g/4 C) g/5 D) E)
D) g/10 E) g/16

-78-
ASOCIACIN EDUCATIVA PITGORAS Fsica

11. Si dos cuerpos de masa m cada uno, separados por 17. En el sistema planetario mostrado, el planeta demora
una distancia L se atraen con una fuerza F. Hallar 6 meses en ir del apogeo al punto P y al perigeo 15
la fuerza resultante que soporta la masa m del meses. Si el rea total de la elipse es S, hallar el
sistema mostrado rea sombreada

A) S/4 B) S/3 C) S/6


A) 3 F B) 2 F C) 5 F D) S/5 E) S/2
D) 7 F E) 8 F
18. En la figura se observa el movimiento de un planeta
12. Un planeta de masa M tiene un satlite de masa m en torno a una estrella E . Si se sabe que desde la
que gira circulantemente alrededor del planeta, en posicin P tarda, el quntuple en llegar a A, que de
una rbita de radio R. A qu distancia del planeta C hasta P, encontrar qu fraccin de la superficie
entre m y M, la aceleracin de la gravedad es cero? elptica es la porcin sombreada
2
(M = K m)
A) B) C)

D) E)

13. Sabiendo que un satlite gira alrededor de su rbita


y adems se halla a una altura h sobre la superficie A) 1/6 B) 2/3 C) 4/5
terrestre, donde la aceleracin de la gravedad es la D) 3/2 E) N.A.
16 ava. parte de la gravedad terrestre. Calcular con
qu velocidad gira el satlite alrededor de su rbita 19. Calcular la densidad de un planeta de forma
esfrica, si un satlite gira a su alrededor en una
A) B) C) rbita circular con periodo T y a una distancia de
la superficie del planeta igual a la mitad de su
radio R (G = cte universal de gravitacin)
D) E) A) 3/8GT
2
B) 27/8GT
2
C) 18/8GT
2
2 2
D) 9/2GT E) 81/8GT
14. Dos satlites de masas M1 y M2 giran alrededor de un
planeta, en rbitas circulares de radios R1 y R2 20. Un cuerpo de masa m es lanzado verticalmente
respectivamente. Si el periodo del satlite M1 es de hacia arriba con una velocidad V0. Desde la
120 das, hallar el periodo del satlite M2 superficie de la tierra, donde su masa es M y su
(R2 = 4R1) radio R. Calcular su velocidad, cuando ha recorrido
A) 960 das B) 720 das C) 680 das una altura igual a R
D) 480 das E) 240 das

15. Un planeta M tiene 2 satlites A y B los que giran


a su alrededor, describiendo rbitas A) B) C)
aproximadamente circulares. Si el periodo de B es
160 das y el radio de la rbita de giro de A es la
cuarta parte del radio de la rbita de B, hallar el D) E) V0
periodo de A
A) 10 das B) 15 das C) 20 das
D) 25 das E) 30 das

16. Dos satlites de la Tierra cada una de masa m, se


mueven en rbitas circulares concntricas con la
Tierra. Si sus posiciones son 2R y 4R respecto del
centro terrestre; en que relacin estn sus energas
cinticas
A) 1/6 B) 1/2 C) 3
D) 1/4 E) 4

-79-
ASOCIACIN EDUCATIVA PITGORAS Fsica

TAREA
01. Imagine que la masa del Sol se volviese 06. Suponga que Jpiter posee un satlite cuya rbita
repentinamente 4 veces ms grande. Para que la tiene un radio igual al de la rbita de la Luna
fuerza de atraccin del Sol sobre la Tierra no sufriese alrededor de la Tierra. El periodo del movimiento de
alteraciones, la distancia entre la Tierra y el Sol la Luna alrededor de la Tierra, como usted ya debe
tendra que volverse. saber, es de casi 27 das. El periodo de este
A) 4 veces mayor B) 4 veces menor supuesto satlite de Jpiter, sera mayor, menor o
C) 2 veces mayor D) 2 veces menor igual a 27 das
E) 8 veces mayor
A) Menor B) Mayor C) Igual
02. Sea F la fuerza de atraccin del Sol sobre un D) F.D. E) Mayor igual
planeta. Si la masa del Sol se volviese 3 veces ms
grande; la del planeta, 5 veces mayor, y la distancia 07. La masa de Jpiter es casi 300 veces mayor que la
entre ellos se redujera a la mitad, la fuerza de de la Tierra. Si el radio de aquel planeta fuera igual al
atraccin entre el Sol y el planeta sera : radio terrestre, cuntas veces mayor que en la
A) 3F B) 15F C) 7,5F Tierra sera la aceleracin gravitatoria en Jpiter?
D) (15/4)F E) 60F A) 150 veces menor
B) 300 veces mayor
03. Un satlite es colocado en rbita a 36 000 km C) 500 veces menor
de altura (misma altura del Intelsat), de modo que el E) 300 veces menor
plano de su rbita pasa por los polos de la Tierra. Un E) 500 veces mayor
observador situado en el polo sur, ve pasar el satlite
sobre su cabeza a las 8 h de la maana de un da 08. Un cuerpo de masa m se abandona desde una
determinado. La prxima vez que el satlite pase altura igual al radio de la Tierra (h = R), respecto de
sobre este observador ser : la superficie terrestre de masa M. Hallar la velocidad
A) A las 12 h del mismo da del cuerpo m cuando choca con la superficie.
B) A las 20 h del mismo da G = constante de gravitacin universal
C) A las 24 h del mismo da
D) A las 8 h del da siguiente
E) A las 12 h del da siguiente A) B) C)

04. Se muestra la rbita de un satlite alrededor del Sol.


D) E)
Si demora 100 das en ir del punto A al punto B,
determinar el periodo de dicho satlite
A1=A2=2A3=4A4 09. Un cuerpo pesa al nivel del mar 75 N. A qu altura
debe elevarse para que su nuevo peso sea 3 N?
R = radio terrestre
A) R B) 2R C) 3R
D) 4R E) 5R

10. Suponga que un satlite se encuentra en rbita sobre


el ecuador de la Tierra, a la misma altura que el
satlite estacionario, pero girando en sentido
contrario a la rotacin de la Tierra
A. El tiempo que este satlite tarda en dar una vuelta
completa en su rbita, tambin sera de 24 h?
B. Dicho satlite sera estacionario?
A) 375 das B) 250 das C) 275 das
C. Si un observador en la Tierra viese pasar este
D) Ninguna E) 200 das
satlite sobre su cabeza en un instante dado,
despus de qu tiempo volvera a suceder esto?
05. El radio de Jpiter es casi 10 veces mayor que el de
la Tierra. Si la masa de dicho planeta exterior (o sea,
A) V; F; 12 horas
de los que estn fuera de la rbita terrestre) fuera
B) F; V; 10 horas
igual a la de la Tierra, cuntas veces menor que en
C) V; V; 20 horas
sta sera la aceleracin de la gravedad en Jpiter?
D) F; F; 10 horas
A) 100 veces menor
E) F; V; 25 horas
B) 50 veces menor
C) 50 veces mayor
D) 100 veces mayor
E) 150 veces menor

-80-
ASOCIACIN EDUCATIVA PITGORAS Fsica

MOVIMIENTO ARMNICO SIMPLE


CONCEPTOS PREVIOS CINEMTICA DEL M.A.S.

Movimiento Peridico.- Es aquel movimiento que se Si una partcula realiza un movimiento circular uniforme
repite en tiempos iguales llamado periodo. (MCU) su proyeccin en cualquier dimetro realiza un
M.A.S.
Movimiento Oscilatorio.- Tambin se le denomina
movimiento vibratorio. Es aquel movimiento donde el
mvil va y regresa sobre la misma trayectoria en torno a
una posicin fija de equilibrio.

MOVIMIENTO ARMNICO SIMPLE

Es aquel movimiento rectilneo realizado por un mvil que


es oscilatorio y peridico donde su aceleracin siempre
seala hacia la posicin de equilibrio y su magnitud es
directamente proporcional a la distancia del mvil a la
posicin de equilibrio (elongacin).

Suponiendo que el mvil parte de B, =ngulo de fase


inicial (partida), + t = ngulo de fase en un tiempo t.
Luego:
P, Q: Extremos
P.E = Posicin de equilibrio o punto medio de PQ

Oscilacin Completa.- Movimiento de ida de P a Q y de


regreso de Q a P.

Periodo (T).- Tiempo empleado en dar cada oscilacin = Frecuencia angular del M.A.S. = Constante
completa.

Frecuencia (f).- Nmero de oscilaciones completas que


realiza el mvil en cada unidad de tiempo.
CASOS

01. = rad (parte del extremo de arriba)

x = ASen !
Unidad(S.I): 1 hertz(Hz) =

02. =0 (parte de la P.E. y hacia arriba)


NOTA: La frecuencia es la inversa del periodo |

Elongacin.- Desplazamiento del mvil desde la posicin


de equilibrio. Su valor nos indica la distancia del mvil a
la posicin de equilibrio. VELOCIDAD (V)

Amplitud (A).- Elongacin mxima cuando el mvil est


en los extremos.

PROPIEDAD

Adems el mdulo de la velocidad es:

-81-
ASOCIACIN EDUCATIVA PITGORAS Fsica

ACELERACIN( ) PERIODO (T)

FRECUENCIA (f)

|
t

Para recordar:
Para recordar: La magnitud de la aceleracin es
, T y f slo dependen de la masa del cuerpo y la
directamente proporcional a la elongacin. constante elstica del resorte.
OBSERVACIONES CONSERVACIN DE LA ENERGA MECNICA DEL
M.A.S.
01. En la P.E. x=0

En los extremos x=A

02. En los extremos x=A

En la P.E. x=0

Dinmica del M.A.S.- La fuerza resultante ( ) que acta


sobre el cuerpo que realiza el M.A.S se llama fuerza ASOCIACIN DE RESORTES
recuperadora. Seala hacia la P.E y su magnitud es
directamente proporcional a la elongacin.
En serie

Por la 2da. Ley de Newton:

En la P.E | FR =0

Sistema Masa-Resorte.- El resorte es de masa En paralelo


despreciable y es elstico. Efecta el sistema un M.A.S si
el rozamiento es nulo.

-82-
ASOCIACIN EDUCATIVA PITGORAS Fsica

PNDULO SIMPLE Si es pequeo (#10) el movimiento se considera un


M.A.S.
Sistema fsico formado por un cuerpo de masa puntual
2
suspendido por una cuerda ligera e inextensible. Cuando FR = m x
2
se separa hacia un lado de su posicin de equilibrio y se mgSen = m x
le suelta el pndulo oscila en un plano vertical por la 2
influencia de la gravedad. mg = m x | =

Luego:
= | T=

T = Periodo

El periodo del pndulo no depende de la masa de la


partcula ni del ngulo . El periodo depende de la
longitud de la cuerda y de la aceleracin de la gravedad
del lugar donde se realiza el M.A.S.

PROBLEMAS PROPUESTOS

01. La ecuacin del movimiento de un punto tiene la 04. Un cuerpo describe un M.A.S. siendo su ecuacin :
forma:
x = 10 cm donde t se expresa

en segundos. Hallar la velocidad del cuerpo cuando


hallar: se encuentra a 6 cm de la posicin de equilibrio
I. El periodo de vibracin
A) 3 cm/s B) 2; cm/s C) cm/s
II. La velocidad mxima del punto
III. La aceleracin mxima D) 8 cm/s E) 4; cm/s
si t est en segundos
2 05. Despus de qu intervalo de tiempo de empezado
A) 2 s; 2 m/s; -/2 m/s el M.A.S. de una partcula su elongacin equivale a
2 2
B) 8 s; - m/s; - /3 m/s los 4/5 de su amplitud por primera vez?. El periodo es
2 2
C) 4 s; m/s; - /2 m/s de 36 s para t = 0 y la partcula se encuentra en un
2 2
D) 6 s; + 2 m/s; - /3 m/s extremo
4 2
E) 1 s; m/s; - m/s A) 3,7 s B) 6,2 s C) 5,3 s
D) 9,2 s E) 12 s
02. Una partcula describe un M.A.S. cuya velocidad est
determinada por la expresin V = 8Cos
06. Cunto tiempo transcurrir desde el comienzo del
Colocar verdadero (V) o falso (F) en los siguientes movimiento armnico simple hasta que el punto
enunciados : vibrante tenga una elongacin igual a la mitad de la
( ) En t = 0 la partcula est en la posicin de amplitud?. El periodo del movimiento es 24 s y el
equilibrio movimiento se inicia de la posicin de equilibrio
( ) La amplitud del M.A.S. es de 2 m
( ) El mnimo tiempo entre los instantes en que la A) 1 s B) 2 s C) 6 s
magnitud de la aceleracin es mxima y luego D) 4 s E) 3 s
mnima en /4 segundos
07. Una bolita sujeta a un resorte se desplaza la distancia
A) FFF B) FVV C) FVF de 1 cm de su posicin de equilibrio y se suelta. Qu
D) VVV E) VFV distancia recorrer la bolita en 2 s si la frecuencia de
sus oscilaciones es de 5 Hz?
03. Escribir la ecuacin de un movimiento vibratorio
armnico de amplitud igual a 10 centmetros, A) 10 cm B) Cero C) 40 cm
sabiendo que en un minuto realiza 90 vibraciones y D) 50 cm E) 60 cm
que la fase inicial de estas vibraciones es igual a 60
08. Una partcula describe un M.A.S.; a 3 m de su
posicin de equilibrio su velocidad es de 2 m/s y a
A) cm B) cm 2 m de la posicin de equilibrio 3 m/s. Cul es el
periodo de oscilacin?
C) cm D) cm
A) s B) 1,5 s C) 3 s
D) 2 s E) 4 s
E) cm

-83-
ASOCIACIN EDUCATIVA PITGORAS Fsica

09. Un cuerpo de masa 100 g pende de un resorte.


A) B) C)
Cuando se tira de el 10 cm por debajo de su P.E. y se
suelta oscila con un periodo de 2 s. Si se mueve
hacia arriba, qu tiempo tarda en ir de un punto D) E)
situado 5 cm por debajo de su posicin de equilibrio
(P.E.) a otro situado 5 cm por encima de ella?
15. Una caja de masa M est sobre una masa
A) 1/3 s B) 2/3 s C) 1 s horizontal. De la caja por medio de un resorte de
D) 1/2 s E) 3/2 s rigidez K, est suspendido un bloque de masa m.
Con qu amplitud de las oscilaciones del bloque, la
10. La amplitud del M.A.S. que efecta una partcula es caja empezar a saltar sobre la mesa?
A = 2 cm y su energa total de las vibraciones :
E = 3.10-7 J. Cul ser la elongacin de la partcula
cuando la fuerza resultante que acta sobre l sea :
F = 2,25.10 -5 N?
-2 -2 -2
A) 2.10 m B) 1,5.10 m C) 3.10 m
-2 -2
D) 4.10 m E) 2,5.10 m

11. A qu es igual la relacin entre la energa cintica


de un punto que vibra con M.A.S. y su energa
potencial, en el momento que la elongacin:
x= ; siendo A la amplitud?
A) B) C)
A) 3 B) 9 C) 12
D) 15 E) 3/4 D) E)

12. Para los sistemas mostrados, qu podemos afirmar


respecto a los periodos? 16. Una masa m sujeta a un muelle elstico realiza un
M.A.S con un periodo de y una amplitud
de 20 cm. Hallar su nueva amplitud cuando sobre m
caiga una masa igual de barro al pasar por la posicin
de equilibrio.

A) T2 > T1 B) T1 > T2 C) T1 = T2
D) T1 = 2T2 E) T2 = 3T1

13. El periodo de vibracin del sistema es de s y si se


retira el bloque A, el perodo resulta s. Calcular la A) 5 B) 10 C) 6
masa del bloque C, las masas de los bloques A y B D) 8 E) 12
son de 400 g.
17. Si el periodo de un pndulo es de 3 s, cul ser su
periodo si su longitud disminuye en 75%?

A) 2 s B) 1 s C) 1,5 s
D) 2,5 s E) 0,5 s

18. Se suelta una esferilla unida a un hilo inelstico de


longitud 4 m. Sabiendo que el tiempo que emplea en
ir y volver a un extremo es de 3,5 segundos, hallar H
(altura del obstculo).
A) 200 g B) 400 g C) 600 g
D) 800 g E) 1 kg

14. Determinar la mxima amplitud de las oscilaciones


del sistema mostrado si se sabe que el piso horizontal
es liso, de tal manera que el bloque de masa m no
llegue a resbalar. La masa del carrito es M.

A) 1 m B) 1,5 m C) 1,75 m
D) 2,5 m E) 3 m

-84-
ASOCIACIN EDUCATIVA PITGORAS Fsica

19. Suponiendo que se ha logrado cavar un tnel que 20. Hallar la longitud del hilo de un pndulo simple de tal
atraviesa diametralmente a la Tierra. Suponer que manera que si dicha longitud aumentase en 3 m su
cualquier cuerpo que se dejara caer en el tnel periodo se duplica
efectuara un M.A.S. si la densidad de la Tierra se A) 3 m B) 1 m C) 6 m
considera constante. Hallar el periodo de oscilacin. D) 2 m E) 9 m
R = radio de la Tierra

A) 2 B) C) 2

D) E)

TAREA
01. El M.A.S. de una partcula est descrito por la A) A1 = 2A2 B) A1 = 5A2 C) A2 =2A1
D) A2 = 2,5A1 E) A1 = 4A2
ecuacin x = 4 m donde t est en
08. Encuentre la constante equivalente del sistema de
segundos. El periodo del movimiento y su velocidad resortes mostrado
mxima son :
A) 1 s; 3 m/s B) 2 s; 4 m/s C) 3 s; 3 m/s
D) 1,5 s; 2 m/s E) 2,5 s; 4 m/s

02. Se observa que el tiempo que tarda un oscilador


armnico en pasar de su posicin de equilibrio a la de
desplazamiento mximo con relacin a esta, es 2 s.
Cul es su periodo?
A) 2 s B) 4 s C) 6 s
D) 8 s E) 10 s

03. Una partcula realiza en M.A.S. con una amplitud de


20 cm y un periodo de medio segundo. Determinar la
mxima velocidad adquirida en su trayectoria
A) 8 cm/s B) 80 cm/s C) 40 cm/s
D) 120 cm/s E) 10 cm/s A) 4K B) 5K C) K/4
D) K/5 E) K/6
04. Un cuerpo de 2 kg que realiza un M.A.S. est sujeto
a un resorte de constante 8 N/m. La magnitud de la 09. En qu relacin se encuentran los periodos de
aceleracin cuando est a 0,5 m de su posicin de oscilacin de los sistemas A y B?
equilibrio es:
2 2 2
A) 0,1 m/s B) 4 m/s C) 0,5 m/s
2 2
D) 2 m/s E) 8 m/s

05. Una partcula empieza su M.A.S. desde su posicin


de equilibrio. Despus de qu fraccin de periodo (T)
su velocidad ser igual a la mitad de su velocidad
mxima
A) T/2 B) T/3 C) T/6
D) T/12 E) T/10 A) 1 B) 1/3 C) 1/2
D) 3 E) 2
06. Un bloque realiza un M.A.S. en el instante en que la
elongacin es la mitad de la amplitud. Qu fraccin 10. Qu valor debe tener la constante K para que el
de la energa total del sistema es cintica? periodo de oscilacin del bloque de 10 kg sea igual a
2 s, si los dems resortes tienen una constante de
A) 1/2 B) 1/5 C) 3/4 1 N/m?
D) 4/9 E) 2/7

07. En el sistema mostrado los bloques tienen igual masa


y oscilan verticalmente. Si tienen la misma velocidad
mxima y K2=0,25K1, la relacin entre sus amplitudes
es:

A) 7 N/m B) 2 N/m C) 3 N/m


D) 4 N/m E) 8 N/m

-85-
ASOCIACIN EDUCATIVA PITGORAS Fsica

HIDROSTTICA
ESTTICA DE FLUIDOS Las unidades de presin atmosfrica ms usadas son :
Es la parte de la mecnica de fluidos que estudia el
comportamiento y los efectos que originan los fluidos en 1 atm = 1,01 . 10 5 Pa
reposo. = 76 cmHg
5
A su vez, la esttica de fluidos se divide en : 1 Bar = 10 Pa
I. Hidrosttica : Estudia a los lquidos en reposo
II. Neumosttica : Estudia a los gases en reposo
PRESIN DE UN LQUIDO EN REPOSO (PRESIN
A qu llamanos fluido? HIDROSTTICA)
Es toda sustancia capaz de fluir, en particular, un lquido
o un gas cualesquiera. Una de las propiedades ms Consideremos un recipiente que contiene un lquido de
importantes es la de ejercer y transmitir presin en toda densidad L
direccin.

Qu es la presin?
Para explicarlo, consideremos la siguiente situacin: Si
ponemos un libro sobre una mesa, no importa como lo
coloquemos - ya sea en posicin horizontal o vertical - la
fuerza que ejerce el libro sobre la mesa es la misma.

Observamos que la columna del lquido ejerce una


presin sobre la superficie de rea A debido a su peso,
esto es :
; pero : F N = mg

Ahora pongamos el libro sobre la palma de nuestra mano ! PH = =


de las dos maneras indicadas anteriormente. A pesar de
que la fuerza es la misma, observaremos que el libro
presiona la palma de la mano con mayor intensidad en el
caso B que en A.

A qu se debe esto?
L : Densidad del lquido (kg/m3)
Para aclarar ideas, supongamos que el libro pesa 20 N
h : Profundidad (m)

Otra manera til de expresar la ecuacin anterior es la


siguiente :

Como : = ! = g

!
3
L : Peso especfico del lquido (N/m )

PRINCIPIO DE PASCAL

En la figura adjunta se muestra un lquido dentro de un


recipiente provisto de un pistn al cual podemos aplicar
Notemos que en el caso B la fuerza de 20 N se cualquier presin externa.
distribuye sobre una menor superficie.
Para caracterizar la accin de una fuerza normal sobre
una superficie se utiliza una magnitud fsica denominada
presin (P)

FN : Fuerza normal a la superficie


A : rea de la superficie
Unidad en el S.I = = Pascal (Pa)
Si ahora aplicamos sobre el mbolo una fuerza de 2 N

-86-
ASOCIACIN EDUCATIVA PITGORAS Fsica

observamos que : Pero, de acuerdo al principio de Pascal.


P1 = P2

De donde :

De acuerdo a este resultado, si A2 > A1, entonces


F2 > F1. Luego, en este caso, la prensa hidrulica
La presin ejercida por la fuerza de 2 N sobre el lquido multiplica la fuerza.
es:
PRINCIPIO DE ARQUMEDES
P= = 2 Pa
Si colocamos un bloque de madera sobre un recipiente
lleno de agua, observaremos que ste flota. Cmo se
puede explicar esto?
Que justamente es igual a la variacin de la presin en las
Consideremos un cuerpo en forma de paraleleppedo
dos lecturas :
sumergido dentro de un lquido de densidad L tal como
( - P1 y - P2 )
se muestra.

El principio de Pascal establece que :

El fluido (gas o lquido) transmite la presin que se


les ejerce en todas las direcciones y con igual valor.

NOTA :

Para prensa hidrulica

Las fuerzas que actan en las caras laterales son iguales


y se equilibran, es decir, F3 = F4. Por el efecto de estas
Aplicacin : Prensa hidrulica fuerzas el cuerpo slo se comprime.

En la vertical, como P2 > P1 entonces F2 > F1 por esta


razn el cuerpo es empujado por una fuerza resultante
F2 - F1 a la cual se denomina empuje hidrosttico (E).

E = F 2 - F1
= P2A - P1A = (P2 - P1)A
= (Lgh2 - Lgh1)A
= Lg(h2 - h1)A

Generalizando este resultado


Al ejercer sobre el pistn de rea A1 una fuerza F1 ste
transmite al lquido una presin P1 dada por :

P1 =

Luego, el lquido le transmite al pistn de rea A2 una


presin P2 dada por :
P2 =

-87-
ASOCIACIN EDUCATIVA PITGORAS Fsica

ATodo cuerpo sumergido total o parcialmente en un fluido Podemos expresar esta ecuacin de una forma diferente
experimenta una fuerza resultante vertical y dirigida hacia considerando la densidad del cuerpo como C y volumen
arriba denominada empuje y acta en el centro de V tenemos que :
gravedad de la parte sumergida (M). Esto es lo que Preal = mg = C Vg
establece el Principio de Arqumedes.
adems :
Si el cuerpo est totalmente sumergido : E = LgV

Vsumergido - Vcuerpo Paparente=C Vg - LgV


= C Vg(1 - )
2

Generalizando este resultado para sistemas acelerados,


observamos que la magnitud del empuje depende de la
Gravedad efectiva que afecta al fluido.

OBSERVACIONES

1. Manmetro : Es aquel instrumento que se utiliza para


medir la presin de un gas encerrado en l.
donde :

: Aceleracin del sistema.


Casos particulares

1. Si el sistema acelera hacia arriba verticalmente con

aceleracin a ! = g + a. Por lo tanto :

2. Si el sistema acelera hacia la derecha


Como todos los puntos pertenecientes a una isbara
horizontalmente con aceleracin a!
estn sometidos a la misma presin, entonces :
Por lo tanto :
P1 = P2

Pgas = PATM + Lgh

Se define el peso aparente de un cuerpo como la


2. El principio de Arqumedes tambin es vlido cuando
diferencia entre el peso real y el empuje que experimenta
un cuerpo est sumergido en forma parcial o total en
dicho cuerpo cuando se sumerge en un lquido de
un GAS. En este caso :
densidad L. Es decir:
Paparente = Preal - E E = gas . g . Vsum

PROBLEMAS PROPUESTOS
01. En el tanque mostrado, hallar la presin del aire 02. En el tubo mostrado, determinar la relacin (a/b), si
3 3 5
encerrado. (agua = 10 kg/m . Patm =10 Pa) las densidades de los lquidos son :
3 3 3
1 = 200 kg/m ; 2 = 500 kg/m ; 3 = 300 kg/m

A) 100 kPa B) 110 kPa C) 140 kPa A) 2/3 B) 1/3 C) 1/2


D) 120 kPa E) 130 kPa D) 2/5 E) 3/5

-88-
ASOCIACIN EDUCATIVA PITGORAS Fsica

03. En el siguiente tubo, determinar x A) 1 cm B) 2 cm C) 3 cm


3 D) 4 cm E) 5 cm
(aceite = 0,8 g/cm ; )

09 El cubo de 2 m de arista y 30 kN de peso se


encuentra en equilibrio mediante un globo de 90 kN
de peso y con la mitad de su volumen sumergido en
agua. Hallar el volumen del globo

A) 2 cm B) 4 cm C) 6 cm
D) 8 cm E) 10 cm

04. Determinar la presin en el gas, si la densidad del


3
lquido es 3 g/cm 3 3 3
A) 2 m B) 4 m C) 8 m
3 3
D) 12 m E) 16 m

10. Un cuerpo de densidad igual a 0,95 g/cm3 flota entre


aceite y agua tal como se muestra en la figura. Hallar
3
la relacin V1/V2. La densidad del aceite es 0,8 g/cm .

A) 24 kPa B) 124 kPa C) 62 kPa


D) 76 kPa E) 94 kPa

05. Un recipiente cilndrico de 5 m de altura y 2 m de A) 1/3 B) 2/3 C) 3/4


dimetro en la base, se encuentra completamente D) 4/3 E) 5/2
3
lleno de un lquido cuya densidad es 800 kg/m .
Determine el valor de la fuerza que soporta el fondo 11. Determinar la densidad del cuerpo que se encuentra
del recipiente debido a la presin hidrosttica. en equilibrio con el 20% de su volumen sumergido en
2
(g = 10 m/s ) el lquido 1
A) 10 000 N B) 20 000 N C) 30 000 N 3
D) 40 000 N E) 50 000 N 1 = 2 000 kg/m Densidades de los lquidos (1) y (2)
3
2 = 5 000 kg/m respectivamente
06. Se tiene una prensa hidrulica cuyos mbolos poseen
radios de 10 cm y 40 cm. Determinar la aceleracin
con la cual subir un cuerpo de 10 kg que se
encuentra sobre el mbolo mayor, cuando se aplica
una fuerza de 10 N sobre el mbolo menor.
(g = 10 m/s2)
2 2 2
A) 4 m/s B) 8 m/s C) 12 m/s
2 2
D) 16 m/s E) 20 m/s

07. En una prensa hidrulica de ramas cilndricas cuyos


radios estn en la relacin de 1/8, se aplica una 3 3
fuerza de 30 N en el mbolo menor. Qu peso A) 2 200 kg/m B) 3 300 kg/m
3 3
podr ser levantado por el otro mbolo? C) 4 000 kg/m D) 4 400 kg/m
3
A) 2 000 N B) 1 820 N C) 1 450 N E) 3 500 kg/m
D) 1 600 N E) 1 920 N 3
12. El bloque de 1 kg de masa y volumen 200 cm se
08. En la figura mostrada se tiene un bloque de 60 N que encuentra en equilibrio. Hallar la tensin en la cuerda
soporta un empuje de 40 N. El bloque se encuentra que lo sostiene
en equilibrio sujeto a un resorte de K = 20 N/cm.
Hallar la variacin de la deformacin en el resorte
cuando el sistema acelera verticalmente hacia arriba
2
con a = 10 m/s , si el bloque al final no se mueve
2
respecto al lquido. (g = 10 m/s )

A) 12 N B) 10 N C) 8 N
D) 6 N E) 4 N

-89-
ASOCIACIN EDUCATIVA PITGORAS Fsica

13. Determine el valor de la aceleracin del sistema, se


sabe que : mA = 1 kg; mB = 2 kg; B = 4 000 kg/m3 y
2
g = 10 m/s

A) 1,27 m/s2 B) 1,37 m/s2 C) 1,47 m/s2


2 2
D) 1,57 m/s E) 1,67 m/s
17. Desde 2 m de altura sobre la superficie libre de un
3
14. Qu tiempo emplear un cuerpo de 4 kg y densidad lquido de peso especfico 6 000 N/m , se deja en
3
800 kg/m en llegar a la superficie libre del agua libertad un cuerpo. Hallar el peso especfico del
despus de ser dejado en libertad a 20 m de cuerpo, sabiendo que en el lquido recorre una
2
profundidad en el agua? distancia mxima de 6 m. (g = 10 m/s )
2 3 3
(g = 10 m/s ) A) 1 500 N/m B) 3 000 N/m
C) 4 500 N/m3
A) 1 s B) 2 s C) 3 s D) 5 000 N/m3 E) 5 500 N/m3
D) 4 s E) 5 s
18. Un cuerpo de 240 N de peso es sumergido
15. Dos esferas de igual volumen estn unidas por una completamente en el agua y se observa que posee
cuerda y sumergidas en dos lquidos de densidades un peso aparente de 160 N. Si al estar sumergido
3 3
1=1,2 g/cm ; 2=0,8 g/cm , como se muestra. Si la totalmente en cierto lquido tiene un peso aparente de
tensin de la cuerda es 2 N y la densidad de la esfera 120 N, determine la densidad del lquido
3
inferior es 1,4 g/cm , hallar la densidad de la esfera
3 3
superior A) 600 kg/m B) 800 kg/m
3 3
C) 1 200 kg/m D) 1 300 kg/m
3
E) 1 500 kg/m

19. Una esfera hueca de radio exterior R y radio interior


r, flota en un lquido de densidad encontrndose
completamente sumergida. Si: R = 2r, determine la
densidad del material del cual est hecha la esfera
3 3 3
A) 1,2 g/cm B) 0,8 g/cm C) 1,6 g/cm
3 3
D) 0,6 g/cm E) 0,5 g/cm

16. Un cilindro de aluminio est suspendido de un


dinammetro y colocado en el interior de un vaso
inicialmente vaco se comienza a agregar agua al
vaso poco a poco (de manera que el cilindro de
aluminio se mantiene en equilibrio). Diga cul de los
grficos representa mejor la fuerza F que marca el
dinammetro con la altura h del nivel del agua en el
vaso A) 3/2 B) 4/3 C) 6/5
D) 7/6 E) 8/7
3
20. Si la esfera (d= 2g/cm ) es lanzada con una velocidad
inicial de 20 m/s, cunto tiempo permanecer en
2
movimiento? (g=10 m/s )

A) 1 s B) 2 s C) 3 s
D) 4 s E) 5 s

-90-
ASOCIACIN EDUCATIVA PITGORAS Fsica

TAREA
01. Determinar el valor de la presin absoluta que ejerce A) 7/4 B) 16/7 C) 4/7
el gas A en el manmetro que se muestra en la D) 8/7 E) 7/16
figura. Considerar que :
5 2
1 atm=10 Pa y g = 10 m/s 07. A qu profundidad x como mnimo habr que
3
introducir la pelota de 0,25 g/cm para que pase por
el rea?

A) 136 kPa B) 186 kPa C) 236 kPa


D) 286 kPa E) 336 kPa

02. Si al recipiente mostrado se le coloca un mbolo de


2
800 g de masa y rea 0,2 m en el cilindro de igual
rea, en cunto se incrementa la presin en el fondo A) H/4 B) 2H/3 C) H/3
del recipiente? (g=10 m/s2) D) H/2 E) H/5
3
08. Un objeto de 3,5 g/cm de densidad se encuentra
dentro de un recipiente que contiene agua
3 3
A =1 g/cm y mercurio (Hg=13,5 g/cm ) flotando
entre los dos lquidos. Calcular la relacin entre los
volmenes de la parte que se encuentra en el agua y
de la parte que se encuentra en el mercurio
A) 80 Pa B) 40 Pa C) 10 Pa
D) 8 Pa E) 4 Pa
3
03. Un tubo en U contiene mercurio (=13,6 g/cm ).
Qu altura de agua se debe verter en una rama para
que el mercurio se eleve 25 mm, en la otra rama?
A) 0,68 cm B) 6,8 cm C) 68 cm
D) 680 cm E) 6 800 cm
04. Calcular la lectura del dinammetro mostrado si el
cubo flota con el 50% de su volumen, la densidad del
3 2 A) 2/3 B) 3/2 C) 1/4
cubo de 10 cm de lado es 0,3 g/cm (g=10 m/s )
D) 4/1 E) 2/1

09. Una esfera se sumerge primero en agua y luego en


3
aceite (aceite=0,8 g/cm ). Cul es el volumen de la
esfera si su peso aparente en el aceite excede
en 20 N al peso aparente en el agua?
3 3 3
A) 0,1 m B) 0,01 m C) 0,2 m
3 3
D) 0,02 m E) 0,5 m
A) 5 N B) 1 N C) 8 N
D) 2 N E) 6 N 10. Qu porcentaje de su volumen emerge el globo si su
volumen es cuatro veces el volumen del bloque
05. En un vaso con agua flota en posicin vertical un
sumergido?
trozo de madera. Cmo variara el nivel del agua en 3 3
el vaso si sel trozo de madera toma la posicin (globo=1/5 g/cm ; bloque=3 g/cm ; )
horizontal?
A) El nivel aumenta
B) El nivel disminuye
C) El nivel no vara
D) Faltan datos para decir
E) Depende del tamao de la madera
06. La barra uniforme de longitud L y densidad x flota
con la cuarta parte de su longitud sumergida en un
lquido de densidad y. Hallar x/y

A) 40% B) 50% C) 60%


D) 30% E) 70%

-91-
ASOCIACIN EDUCATIVA PITGORAS Fsica

CALOR
DILATACIN TRMICA
Modificacin de las dimensiones de un cuerpo debido a la Coeficiente de Dilatacin
=
modificacin de su temperatura. Cbica

Temperatura: Toda una serie de observaciones nos lleva VF = Vo (1 + T)


a la conviccin de que en toda sustancia existe un
movimiento interno continuo. Este movimiento interno CONSIDERACIONES
representa de por s el movimiento de las molculas, que
componen la sustancia en cuestin, el cual es (1) Escalas de temperatura:
desordenado y no se interrumpe nunca, denominado
movimiento trmico.
El carcter del movimiento trmico de las molculas
depende del carcter de las interacciones de stas y
cambia cuando la sustancia cambia de estado fsico
(fase).
La intensidad del movimiento trmico molecular depende
del grado de calentamiento del cuerpo, que se caracteriza
por su temperatura absoluta ( ) . Tericamente se ha TC = (TF - 32) = TK - 273
demostrado que la energa cintica promedio (Ek) de las
molculas es proporcional a la temperatura absoluta.
TC = (5/9) TF = TK
Dilatacin de Slidos: Dependiendo del nmero de
dimensiones que se consideren, se distinguen los (2) Los diferentes coeficientes de dilatacin se expresan
-1 -1 -1
siguientes tipos de dilatacin: en la inversa de grado de temperatura: C ; F ; K
(3) Para un mismo slido si <<< 1 : 2 y 3
1. Dilatacin Lineal: (4) Los lquidos slo experimentan dilatacin cbica
Reglas Empricas 3 L (D.P.) T y L (D.P.) Lo
CALORIMETRA
Estudia los procedimientos adecuados para determinar
las cantidades de calor y los sistemas de unidades
correspondiente.
Calor: Energa que intercambian dos cuerpos en contacto
a diferentes temperaturas y durante el cual no se realiza
trabajo. El flujo de energa se da siempre del cuerpo a
mayor temperatura hacia el cuerpo a menor temperatura,
cesando cuando alcanzan el equilibrio trmico. En el
Coeficiente de Dilatacin
equilibrio trmico los cuerpos poseen una temperatura
Lineal
comn (TE : Temperatura de Equilibrio) siendo nulo el
intercambio neto de energa.
LF = Lo (1 + T) (T = TF - To) Siendo el calor una forma de energa, la cantidad de calor
se debera expresar en joule, pero se emplean otras
2. Dilatacin Superficial: unidades como:
Reglas Empricas 3 S (D.P.) T y S (D.P.) So

Kilocalora (kcal) = 1 000 cal


B.T.U. = 252 cal

Capacidad Calorfica o Trmica (C): Magnitud que


caracteriza las propiedades calorimtricas de un cuerpo
o sistema. Se obtiene mediante la relacin entre la
Coeficiente de Dilatacin cantidad de calor (Q) absorbida o cedida por el cuerpo o
Superficial sistema y el correspondiente cambio de temperatura
(T = TF - To)
SF = So (1 + T)

3. Dilatacin Cbica:
Reglas Empricas 3 V (D.P.) T V (D.P.) Vo

Calor Especfico (Ce): Para una sustancia dada se


cumple que la capacidad calorfica es directamente
proporcional a la masa de la sustancia considerada. Se
define el calor especfico de una sustancia como la
capacidad calorfica por unidad de masa.

-92-
ASOCIACIN EDUCATIVA PITGORAS Fsica

El calor especfico es una caracterstica de cada A. Cambios Progresivos: En este caso la sustancia
sustancia. La cantidad de calor absorbida o cedida por un absorbe calor (Q(+)), tenindose los siguientes casos:
cuerpo o sistema se puede obtener por las expresiones: Fusin: Paso del slido al lquido
Vaporizacin: Paso del lquido al vapor

B. Cambios Regresivos: En este caso la sustancia


cede calor (Q(-)), tenindose los siguientes casos:
Solidificacin: Paso del lquido al slido
Condensacin: Paso del vapor al lquido
Unidades:
Los diferentes cambios de fase presentan las siguientes
caractersticas:
(1) Es un proceso isotrmico e isobrico
(2) La temperatura de cambio de fase depende de la
Q = C (TF - To) Si TF > To 3 Q(+) (calor ganado) presin que soporta la sustancia considerada
(3) Todo cambio de fase implica una modificacin de las
Q = Cem (TF - To) Si TF < To 3 Q(-) (calor perdido) propiedades fsicas de la sustancia considerada

Equivalente en Agua: Se denomina as a la masa de Calor Latente (L): Caracteriza la cantidad de calor
agua que posee la misma capacidad calorfica que el necesaria para que una sustancia, a la conveniente
cuerpo o sistema considerado. Si dos sistemas diferentes temperatura y presin, experimente el cambio de fase
poseen la misma capacidad calorfica se dice que son correspondiente.
trmicamente equivalentes. El calor latente determina la cantidad de calor que se le
debe entregar o sustraer a la unidad de masa para que el
cambio se produzca.

3 Q = Lm

Unidades:
Principio Fundamental de la Calorimetra: Si se tienen cal/g ; kcal/kg ; joule/kg
dos o ms cuerpos en contacto a diferentes temperaturas,
y aislados trmicamente (es decir slo intercambian Para una sustancia que experimente los cambios
energa entre ellos), entonces el calor cedido por unos es mencionados se cumple:
numricamente igual al calor ganando por los otros.
LSOLD = -LFUS LCOND = -LVAP.
Calor Ganado = |Calor Perdidol Cabe mencionar que el calor latente, al igual que la
temperatura de cambio de fase, depende de la presin
3Q = 0 (Suma algebraica) que soporta la sustancia considerada.

CAMBIOS DE FASE

Modificacin del orden molecular de una sustancia, que


se encuentra a la conveniente temperatura y presin,
debido a una entrega o sustraccin de calor.
En la naturaleza las sustancias se encuentran en tres
fases o estados fsicos, clasificndose los pasos entre
ellos en:

PROBLEMAS PROPUESTOS
01. Un cuerpo tiene una capacidad calorfica de 6 cal/C 04. En un calormetro de equivalente en agua igual a
y su masa es 300 g. Si su temperatura pasa de 16 C 20 g, se tiene 280 g de agua a la temperatura de
a 26 C, qu cantidad de calor habr absorbido? 15 C. Si se introduce un bloque metlico de 400 g
A) 50 cal B) 60 cal C) 70 cal a 100 C se logra una temperatura de equilibrio de
D) 120 cal E) 80 cal 25 C. Hallar el C e del metal en cal/gC

02. Si la cantidad de calor necesario para aumentar A) 0,9 B) 0,8 C) 0,6


en 100 C la temperatura de 10 kg de un metal es D) 1,2 E) 0,1
100 kcal, qu porcentaje de calor se disipa al medio
exterior? (Ce = 0,085 cal/gC) 05. Se tiene en un recipiente 100 g de agua a la
A) 5% B) 10% C) 15% temperatura de 20 C. Si se introduce un trozo de
D) 20% E) 25% metal de 400 g y a la temperatura de 100 C,
determinar la temperatura final de equilibrio, si el
03. En un recipiente vaciamos 200 g de agua a 20 C, calor especfico del metal es 0,11 cal/g C
40 g de agua a 40 C y 60 g de agua a 80 C.
Calcular la temperatura de equilibrio A) 20 C B) 32,2 C C) 12,6 C
A) 34,66 C B) 35 C C) 38 C D) 44,4 C E) 52,2 C
D) 50 C E) 70 C

-93-
ASOCIACIN EDUCATIVA PITGORAS Fsica

06. Se mezclan masas iguales de tres lquidos A, B y C A) 3 y 8 B) 10 y 15 C) 8 y 15


cuyas temperaturas son de 20; 40 y 60 C D) 6 y 15 E) 7 y 10
respectivamente. Si:
13. Una bala de plomo de masa 5 g se mueve con una
Ce(A) = Ce(B) = Ce(C), hallar la temperatura final de
energa cintica de 12,6 J, choca contra un blanco y
la mezcla queda en reposo. Cul es el incremento en la
A) 40 C B) 46 C C) 20 C temperatura de la bala si no hay flujo de calor hacia
D) 23 C E) 57 C el medio ambiente?
(CePb = 0,03 cal/gC)
07. Se tiene 30 g de agua a 60 C. Determinar la cantidad A) 40,2 C B) 20,1 C C) 30,6 C
de calor que se requiere para tener 30 g de vapor de D) 50 C E) 15,3 C
agua a 120 C 14. Un clavo de hierro de 20 g y calor especfico
A) 15,2 kcal B) 17,7 kcal C) 18,6 kcal 481 J/kgC est siendo golpeado por un martillo de
D) 19,0 kcal E) 20,0 kcal 2 kg de masa. La velocidad de impacto del martillo
es m/s. Si la mitad de la energa cintica es
08. Se tiene 360 g de agua a 20 C. Qu cantidad de
convertida en energa trmica del clavo, cuntas
calor se debe extraer para convertirla en hielo a 0 C?
veces hay que golpear el clavo para elevar su
A) 6 kcal B) 12 kcal C) 18 kcal
temperatura en 25 C?
D) 24 kcal E) 36 kcal
A) 17 golpes B) 20 golpes C) 27 golpes
D) 30 golpes E) 37 golpes
09. Si mezclamos 20 g de hielo a -60 C con M gramos
de vapor de agua a 100 C se obtiene una 15. En un calormetro de equivalente en agua 20 g,
temperatura de equilibrio de 40 C. Entonces el valor se tiene 180 g de agua a 15 C, un bloque metlico
de M es : de 500 g y Ce=0,03 cal/gC ingresa a 80 C en el
A) 12 B) 5 C) 8 calormetro. Cul ser la temperatura de equilibrio?
D) 15 E) 10 A) 23,5 C B) 30,5 C C) 19,5 C
D) 47,5 C E) 42,3 C
10. En un calormetro de equivalente en agua igual
a 20 g, se tiene 180 g de agua en equilibrio trmico 16. En un calormetro de 500 g y calor especfico
con 100 g de hielo. Si se inyecta 20 g de vapor de 0,03 cal/gC se tiene 50 g de hielo a -10 C, se vierte
agua a 100 C, cul es la temperatura de equilibrio? en el calormetro 70 g de agua a 40 C. Encuentre
A) 10 C B) 0 C C) 20 C usted las condiciones finales del sistema
D) 15 C E) 23 C A) Agua 100 g; hielo 20 g a 0 C
B) Agua 80 g; hielo 40 g a 0 C
11. El grfico representa la temperatura T en funcin del C) Agua 45 g; hielo 75 g a 0 C
calor absorbido por 20 gramos de cierto lquido. D) Agua 120 g; a 2 C
Cunto vale el calor latente de evaporacin del E) Agua 120 g; a 23 C
-1
lquido si : Tg = 10 ?
17. Se mezclan igual cantidad de masa de hielo a 0 C y
vapor de agua a 100 C, en un recipiente de
capacidad calorfica despreciable. Cul es la
temperatura final de equilibrio?
A) -10 C B) 0 C C) 15 C
D) 100 C E) 141 C

18. Se tiene en un recipiente 100 g de agua a la


temperatura de 20C. Si se introduce un trozo de
metal de 400 g y a la temperatura de 100 C,
determinar la temperatura final de equilibrio, si el
A) 150 cal/g B) 200 cal/g C) 250 cal/g calor especfico del metal es 0,11 cal/gC
D) 300 cal/g E) 350 cal/g A) 20 C B) 32,2 C C) 12,6 C
D) 44,4 C E) 52,2 C
12. Una muestra de mineral de 10 g de masa recibe calor
de modo que su temperatura tiene un 19. En un calormetro de capacidad calorfica
comportamiento como el mostrado en la figura. despreciable se tiene 45 g de hielo a -24 C. Si se
Determinar los calores latentes especficos de fusin hace ingresar 26 g de vapor de agua a 100 C, hallar
y vaporizacin en cal/g la temperatura final de equilibrio
A) 100 C B) 0 C C) 36 C
D) 56 C E) 13 C

20. Si en un calormetro ideal, se introducen hielo a


-10 C con agua lquida a 85 C en iguales
cantidades, entonces podemos afirmar que en el
equilibrio habr:
A) Agua lquida a temperatura sobre 0 C
B) Hielo a temperatura bajo 0 C
C) Solamente hielo a 0 C
D) Solamente agua lquida a 0 C
E) Agua y hielo a 0 C

-94-
ASOCIACIN EDUCATIVA PITGORAS Fsica

TAREA
01 Calcular la temperatura de equilibrio al mezclar 40 g 06. Un proyectil de 50 g que se desplaza a 200 m/s, se
de agua a 10 C con 60 g de agua a 30 C y incrusta sobre un bloque de hielo que se encuentra
con 120 g de agua a 60 C a 0 C. Suponiendo que el 50% de la energa cintica
A) 36,65 C B) 59,14 C C) 42,72 C se convierte en calor que es absorbido por el hielo,
D) 53,5 C E) 24 C hallar la masa de hielo que se derrite

02. En un recipiente de capacidad calorfica despreciable A) 2,5 g B) 2 g C) 1,5 g


se tiene X gramos de hielo a 0 C, en contacto con D) 1 g E) 0,5 g
Y gramos de vapor de agua a 100 C. Determinar la
relacin entre X e Y, para lograr que todo el contenido 07. En un calormetro de equivalente en agua igual a
logre su equilibrio trmico, obteniendo slo lquido a 10 g contiene 150 g de agua a 0 C. Se introduce un
100 C bloque metlico de 200 g a 200 C. Hallar la
(LF = 80 cal/g; LV = 540 cal/g) temperatura de equilibrio.
A) X = 3Y B) Y = 3X C) X = Y
D) X = 4Y E) Y = 4X
A) 2 C B) 2,01 C C) 3 C
03. Una masa m de cierto metal experimenta una D) 4,87 C E) 5 C
variacin de temperatura de acuerdo a la siguiente
grfica al entregarle calor. Cul(es) de las siguientes 08. En un calormetro de calor especfico despreciable
afirmaciones es(son) verdadera(s) si la presin es se tienen 1 000 g de agua a cierta temperatura.
constante? Si un cuerpo metlico se introduce a 65 C, entonces
la temperatura de equilibrio es de 50 C, pero si el
cuerpo metlico se introduce a 30 C, entonces la
temperatura de equilibrio es 25 C. Determine la
temperatura inicial del agua.
A) 5 C B) 7,5 C C) 10 C
D) 12,5 C E) 15 C

09. Desde una altura de 58,8 m, se deja en libertad una


esfera de 10 kg, la cual desciende, impacta en el piso
I. En el tramo BC existe un cambio de fase y luego slo se eleva hasta una altura de 49 m.
II. El calor especifico de la sustancia lquida es Determine cunto de hielo a 0 C se puede fundir
(Q3 + Q1) / m(T2 - T1) debido al calor que se disipa en el impacto
2
III. El calor latente de fusin del material es (Considerar: 1 joule = 0,24 cal y g = 9,8 m/s )
(Q2-Q1) /m
A) Slo I B) I y II C) I y III A) 1,88 g B) 2,88 g C) 3,88 g
D) II y III E) Slo II D) 4,88 g E) 5,88 g

04. En un recipiente se tiene agua a 0 C. Si se introduce 10. En un calormetro de capacidad calorfica


800 g de hielo a -10 C, qu cantidad de agua se despreciable se tiene 45 g de hielo a -24 C. Si se
solidificar? hace ingresar 26 g de vapor de agua a 100 C, hallar
A) 20 g B) 30 g C) 40 g la temperatura final de equilibrio
D) 50 g E) 80 g A) 100 C B) 0 C C) 36 C
D) 56 C E) 13 C
05. Un bloque de plata (Ce = 0,06 cal/g C) de 200 g de
masa se encuentra a 21 C. Qu cantidad de calor
debe suministrrsele para derretirlo, si la
temperatura de fusin es 961 C y su calor latente
de fusin es 21 cal/g?
A) 10 000 cal B) 14 400 cal C) 15 000 cal
D) 15 480 cal E) 16 724 cal

TERMODINMICA
CONCEPTO DE TERMODINMICA postular el principio de la conservacin de la energa. La
La termodinmica trata acerca de la transformacin de segunda ley impone restricciones sobre el uso eficiente de
energa trmica en energa mecnica y el proceso la energa disponible.
inverso, la conversin de trabajo en calor. Puesto que casi
toda la energa disponible de la materia prima se libera en Sistema Termodinmico
forma de calor, resulta fcil advertir por qu la
termodinmica juega un papel tan importante en la ciencia Es aquella porcin de materia que puede considerarse
y la tecnologa. limitada por una superficie cerrada real o imaginaria. La
En este captulo se estudiarn dos leyes bsicas que regin no incluida en el sistema constituye el exterior o
deben obedecerse cuando se utiliza energa trmica para alrededores o ambiente.
realizar trabajo. La primera ley es simplemente volver a

-95-
ASOCIACIN EDUCATIVA PITGORAS Fsica

Estado de un sistema
Es una situacin determinada del sistema definida por los
valores de sus variables termodinmicas (presin,
volumen, temperatura, etc), en el diagrama PV se
representa por un punto,

T = Temperatura absoluta (K)


U = U2 - U1
LA VARIACIN DE ENERGA INTERNA (U) NO
DEPENDE DE LA TRAYECTORIA
Estado A
P1 V1 T1
Trabajo realizado por un gas (W)
Proceso termodinmico Para que un gas efecte trabajo necesariamente debe
Es una sucesin continua de estados que el sistema cambiar su volumen ya sea expandindose o
experimenta cuando es estimulado externamente, en el comprimindose, que se realice mayor o menor cantidad
diagrama PV se representa por una curva continua. de trabajo depende del proceso que se siga al cambiar de
volumen, en un diagrama PV el trabajo est representado
Proceso termodinmico AB por el rea que est entre la grfica y el eje horizontal.

Ciclo termodinmico Si el volumen aumenta W(+)


Es una sucesin de estados o procesos de tal forma que Si el volumen disminuye W(-)
el sistema al final vuelve a su estado inicial.
CAPACIDAD CALORFICA MOLAR
Ciclo ABCA
Debido a que todo gas puede ser calentado o enfriado
manteniendo la presin o volumen constante, entonces
existir 2 tipos de capacidad calorfica. Uno a presin
constante y el otro a volumen constante siendo el primero
mayor que el segundo y su diferencia nos determina la
constante universal de los gases (R).

Cp = Capacidad calorfica molar a presin constante


Cv = Capacidad calorfica molar a volumen constante
Cp > Cv y Cp - Cv = R
Energa interna de un gas ideal (U)
R = 8,31 =2
Es la suma de las energas cinticas de traslacin,
vibracin y rotacin de todas las molculas que componen
determinada masa de gas ideal, esta magnitud depende Para gases monoatmicos:
de la temperatura absoluta (T) y de la cantidad de gas (n) He, Ne, Ar, Kr, Xe:

Para un gas monoatmico formado por n moles la


energa interna es :
Para gases diatmicos: H, N, O, CO:

Para un gas diatmico formado por n moles la energa


interna es :

k : constante adiabtica
Donde:
R = Constante universal de los gases = 8,31

-96-
ASOCIACIN EDUCATIVA PITGORAS Fsica

PRIMERA LEY DE LA TERMODINMICA rea = A = P(Vf - Vi)


rea = A = PV
En todo proceso termodinmico el calor que entra o sale rea = A = W = Trabajo
de un sistema ser igual al trabajo realizado por el
sistema o sobre l, ms la variacin de la energa interna. II. PROCESO ISCORO ( V = Cte )
Es aquel proceso termodinmico, en el cual una
sustancia evoluciona desde un estado inicial hasta
otro final manteniendo su volumen constante.
(5) Diagrama P - V

(1) W = 0 Y Q = U
(2) Q = n CVt
(3) U = nCVt
Q = W + U (4) Ley de Gay - Lussac :

Q = Calor que entra o sale.


W = Trabajo realizado por o sobre el sistema.
U = Variacin de la energa interna.
III. PROCESO ISOTRMICO (T = Cte )
En este proceso, se hace evolucionar a la sustancia
desde un estado inicial hasta otro final, manteniendo
su temperatura constante.

(1) U = 0 ! Q = W

(2) Q = 2,3 Pi Vi =PiViLn

(3) W = 2,3PiVi =PiViLn

(4) Ley de Boyle - Mariotte : PiVi = PfVf

PROCESOS TERMODINMICOS : IV. PROCESO ADIABTICO ( Q = 0 )


Es aquel proceso termodinmico, en el cual se hace
Proceso termodinmico, es la secuencia de estados por evolucionar a la sustancia desde un estado inicial
los cuales se obliga a pasar a la sustancia de trabajo para hasta otro final sin adicin ni sustraccin de calor.
que se permita la conversin de calor en trabajo.

I. PROCESO ISOBRICO :
En este proceso se hace evolucionar a un sistema
desde un estado inicial hasta otro final manteniendo
en todo instante la presin constante.
(1) W = PV
(2) Q = nCPt
(3) U = nCVt
(1) Q = 0 ! W = - U
(4) ( Ley de Charles ) (2) U = nCvt
(3) ;
(5) Diagrama P - vs - V
(4)

NOTA: La pendiente de la curva adiabtica es mayor que


la pendiente de la curva isotrmica.

-97-
ASOCIACIN EDUCATIVA PITGORAS Fsica

SEGUNDA LEY DE LA TERMODINMICA CICLO DE CARNOT


A) Ningn cuerpo es capaz de entregar calor en forma
espontnea a otro cuerpo de mayor temperatura, Es aquel ciclo con el cual una mquina trmica tendra la
existiendo la posibilidad de forzarlo a ello si es que mxima eficiencia, est constituido por dos procesos
previamente en l se invierte trabajo. isotrmicos y dos procesos adiabticos, su eficiencia slo
B) No existe mquina trmica que sea capaz de depende de las temperaturas absolutas de los focos entre
convertir en forma continua todo el calor en trabajo. los cuales opera.
C) No existe ninguna mquina trmica cuya eficiencia
sea del 100%.

MQUINA TRMICA
Es aquel dispositivo que transforma parte del calor que
recibe en trabajo mecnico, est constituido por una
fuente caliente (caldera u horno), que entrega calor (Q1)
a la mquina y otra fuente fra (condensador o sumidero
de calor), donde se expulsa el calor residual (Q2). El
trabajo til que se obtiene de la mquina trmica es
W=Q1-Q2

Representacin esquemtica y clculo de la eficiencia de


una mquina trmica. I. Expansin Isotrmica (A - B)

II. Expansin Adiabtica (B - C)

III. Compresin Isotrmica (C - D)

IV. Compresin Adiabtica (D - A)

Relaciones :

PROBLEMAS PROPUESTOS
01. Si se realiza sobre un sistema un trabajo igual 03. Se comprime un gas a presin constante de 160 kPa
3 3
a 418 J mientras que se suministra a la vez 200 cal desde 8 dm hasta 4 dm . Si el gas disipa 100 J de
de calor a dicho sistema, el cambio de energa interna calor, qu variacin experiment la energa interna
del sistema en caloras es: (1 cal = 4,18 J) del gas?

A) 300 B) 100 C) 319 A) 450 J B) 540 J C) 640 J


D) 218 E) 200 D) 700 J E) 900 J

02. Hallar el trabajo realizado por el gas ideal en este 04. Un gas experimenta un proceso termodinmico tal
ciclo : como se indica en la grfica. Si su energa interna
aument en 25 kJ, qu cantidad de calor gan o
cedi al medio exterior?

A) Gan 10 kJ B) Perdi 10 kJ
A) -10 kJ B) -20 kJ C) -30 kJ C) Gan 15 kJ D) Perdi 15 kJ
D) -40 kJ E) -60 kJ E) No gan ni perdi

-98-
ASOCIACIN EDUCATIVA PITGORAS Fsica

05. Un ventilador suministra 1,5 kW a un sistema durante 12. El grfico representa el ciclo de Carnot sealar
60 s aumentando su volumen de 0,03 m3 a 0,09 m 3, verdadero (V) o falso (F) segn corresponda:
mientras la presin se mantiene constante en
500 kPa. Qu variacin de energa interna
experimenta el sistema, si ste disipa 12 kJ de calor
durante el mencionado periodo?

A) 48 kJ B) 118 kJ C) 72 kJ
D) -48 kJ E) -72 kJ

06. Un gas se expande isotrmicamente. Si ha recibido


una cantidad de calor de una kilocalora, calcular el
trabajo que el gas realiza en la expansin. ( ) Es el ciclo ms eficiente que se conoce.
(1 cal = 4,186 J) ( ) (1-2) y (3-4) son procesos isotrmicos.
( ) (2-3) y (4-1) son procesos adiabticos.
A) 4 186 J B) 1 178 J C) 1 427 J ( ) En (3-4) la mquina recibe calor.
D) 0,24 J E) 0 J
A) VVVV B) VVVF C) VFFV
07. Cuando un sistema pasa del estado A al B a lo largo D) VFFF E) FVVV
de la trayectoria ACB, recibe 20 000 cal y realiza
7 500 cal de trabajo. Cunto calor recibe el sistema 13. Se tiene el siguiente diagrama de un ciclo de Carnot
a lo largo de la trayectoria ADB, si el trabajo es Cul sera su diagrama calor (Q) versus temperatura
2 500 cal? (T)?

A) 12 000 cal B) 18 000 cal C) 15 000 cal


D) 19 000 cal E) 22 000 cal

08. Un gas ideal encerrado en un cilindro que tiene pistn


deslizante, mediante un proceso isotrmico recibe
48 cal de calor y logra desplazar al pistn
horizontalmente Qu cambio de energa interna
experiment el gas?. Qu trabajo realiz el
gas al mover al pistn?. Dar como respuesta la
diferencia del mayor menos el menor (en joules)
(1 joule = 0,24 cal)

A) Cero B) 100 C) 200


D) -100 E) -200

09. Una mquina trmica sigue el ciclo de Carnot


trabajando entre las temperaturas de 200 K y 800 K.
Halle el trabajo, sabiendo que expulsa 120 cal

A) 270 cal B) 300 cal C) 360 cal


D) 540 cal E) 240 cal

10. Un motor que funciona con el ciclo de Carnot tiene su


foco caliente a 127 C, toma 100 cal a esta
temperatura en cada ciclo, y cede 80 cal al foco fro.
Calcular la temperatura de este depsito
14. Decir si es verdadero (V) o falso (F):
A) 50 C B) 197 K C) 320 K ( ) En una transformacin a volumen constante no
D) 102 C E) 273 K se realiza trabajo.
( ) En una transformacin isotrmica no hay
11. La temperatura del foco caliente de una mquina intercambio de calor, entre el sistema y el medio
trmica que sigue el ciclo de Carnot es de 300 K y su ambiente.
eficiencia es del 50%. En qu porcentaje disminuira ( ) La cantidad de calor necesaria para calentar un
su eficiencia, si la temperatura del foco caliente gas a presin constante es menor que la
disminuye en 50 C? necesaria para calentarlo a volumen constante.

A) 5% B) 8% C) 10% A) VFV B) FFF C) VFF


D) 12% E) 15% D) VVV E) FVF

-99-
ASOCIACIN EDUCATIVA PITGORAS Fsica

15. La eficiencia de una mquina trmica que sigue el


ciclo de Carnot es n1 = 0,4 y la eficiencia de otra A) B)
semejante es n2 = 0,6. Hallar la eficiencia de una
mquina trmica compuesta por las dos anteriores, si C) D)
se sabe que el calor que libera la primera mquina es
absorbida por la segunda para realizar su ciclo.
E)
A) 0,24 B) 0,33 C) 0,76
D) 0,40 E) 0,50
18. Un mol de un gas monoatmico se expande
16. La grfica es una adiabtica si V1 y V2 son las adiabticamente variando su temperatura desde
energas internas de los puntos (1) y (2) 290 C hasta 250 C y su volumen de 2 a 4 litros.
respectivamente. Indicar qu relacin se cumple : Calcular el trabajo realizado en este proceso

A) 498,6 J B) 226,8 J C) 333,8 J


D) 518,6 J E) 281,6 J

19. Determinar la variacin de energa interna que tiene


lugar en la fusin de 91 kg de hielo a la presin de
5 3
10 Pa. La densidad del hielo es 910 kg/m ;
joule = 0,24 cal
A) 7280,216 kcal
B) 9280,216 kcal
A) V1 $ V2 B) V1 # V2 C) V1 > V2 C) 6280,216 kcal
D) V1 < V2 E) Falta informacin para decidir D) 8280,216 kcal
E) 1680,216 kcal
17. En un cilindro vertical de seccin S debajo del
mbolo, cuya masa es m, hay aire, sobre el mbolo 20. Qu cantidad de calor se debe suministrar a un gas
se encuentra una pesa. Si se quita dicha pesa, el ideal, para que al expandirse isobricamente realice
volumen que ocupa el aire que hay debajo del mbolo un trabajo de 4 J?
se duplica y la temperatura de dicho aire se hace la = k = 7/5
mitad. Determinar la masa M de la pesa, la presin
= k = exponente adiabtico del gas
atmosfrica es igual a P o
A) 8 J B) 10 J C) 12 J
(g = gravedad)
D) 14 J E) 16 J

TAREA
01. Un gas ideal realiza el ciclo1231. El trabajo realizado 03. Se tiene un sistema termodinmico el cual realiza el
por el gas en este ciclo es : ciclo mostrado. El trabajo realizado durante el ciclo
es:

A) 1/2 [(P1 + P2)(V2 - V3)]


B) 1/2 [(P1 + P3)(V2 + V3 )]
C) 1/2 [(P1 - P2)(V2 + V3)]
D) 1/2 [(P1 - P3)(V2 - V3)]
E) Cero A) 20 J B) 40 J C) -60 J
D) 80 J E) 100 J
02. En la figura se muestra el diagrama Presin versus
Volumen correspondiente al ciclo termodinmico 04. Cuando un gas pasa del estado A al estado C
efectuado por un sistema. Determine el trabajo neto siguiendo la trayectoria ABC, recibe 20 000 cal y
efectuado durante el ciclo efecta un trabajo de 7 500 cal. Calcular el calor
recibido a lo largo de la trayectoria ADC, si el trabajo
efectuado es de 2 500 cal

A) (P1 - P2)(V2 - V1) B) (P2 - P1)(V1 - V2)


C) (P2 - P1)(V2 - V1) D) P1V1 + P2V2 A) 5 000 cal B) 10 000 cal C) 15 000 cal
E) P1V1 - P2V2 D) 20 000 cal E) 25 00 cal

-100-
ASOCIACIN EDUCATIVA PITGORAS Fsica

05. Conociendo que en el proceso AB el gas ideal recibi 07. El plano P-V muestra el proceso que sigue un gas
100 cal, encuentre la variacin de la energa interna ideal, la energa interna en A es 60 J y en B es de
en el mencionado proceso 75 J. Halle el calor suministrado en el proceso AB

A) 80 J B) 95 J C) 103 J
A) 300 J B) -300 J C) 100 J D) 108 J E) 121 J
D) 48 J E) 18 J
08. Una mquina trmica ideal trabaja entre las
06. En el proceso termodinmico mostrado, determine el temperatura de 500 K y 200 K. Si recibe 1 600 cal de
trabajo efectuado por el sistema. las calderas halle el trabajo neto (1 cal = 4,18 J)
5
(considere : 1 atm = 10 Pa) A) 2012,8 J B) 3012,8 C) 4012,8
D) 5012,8 E) 6012,8

09. Una mquina trmica produce 480 J por cada


10 ciclos con un rendimiento de 30%. Halle el calor
que la mquina trmica cede al foco fro en cada ciclo
A) 84 J B) 54 J C) 28 J
D) 140 J E) 112 J

10. En una mquina de Carnot el calor rechazado es la


A) 100 J B) 120 J C) 150 J cuarta parte del calor que absorbe la mquina. Halle
D) 180 J E) 240 J la temperatura del foco caliente si el foco fro est a
7 C
A) 347 C B) 112 C C) 847 C
D) 947 C E) 1 000 C

ELECTROSTTICA : CAMPO ELCTRICO - FUERZA ELCTRICA


ELECTROSTTICA Si q(-) ! El cuerpo gana electrones
CONCEPTO :
-
Es una parte de la teora de la electricidad que tiene por Si q = 0 ! El cuerpo no gana ni pierde electrones e
finalidad analizar los diferentes fenmenos fsicos
originados por las cargas elctricas o cuerpos cargados
en estado de reposo. LEYES DE LA ELECTRICIDAD

CARGA ELCTRICA 1. LEY CUALITATIVA O DE LAS ATRACCIONES Y


Que es? es una propiedad que se asigna a las partculas REPULSIONES
elementales del tomo con la finalidad de establecer
interacciones Electromagnticas.
Qu cantidad de electricidad o carga elctrica poseen
las partculas elementales?
v Electrn : 1e- = -1,6.10-19 coulomb
+ -19
v Protn : 1p = +1,6.10 coulomb
v Neutrn : 1n = 0 coulomb

Cmo se mide la cantidad de carga elctrica que est en


un cuerpo conductor?
Siendo los electrones libres los que se desplazan, la
carga presente en un conductor se define:

-
q = N |e | (Cuantizacin de la carga)

N = Nmero entero
N = Nmero de electrones transferidos 2. LEY CUANTITATIVA O DE COULOMB

Si q(+) ! El cuerpo pierde electrones

-101-
ASOCIACIN EDUCATIVA PITGORAS Fsica

Es la caracterstica vectorial de un campo elctrico que se


mide en base a la fuerza que por unidad de carga se
manifiesta sobre una partcula expuesta a la accin del
campo elctrico.

Donde :
Q1, Q2 : Cargas elctricas
d : distancia

K = Constante fsica de las propiedades dielctricas del


medio que rodea a las cargas
Se define :
Experimentalmente :

a = o
LNEA DE FUERZA

ga : Permisividad elctrica absoluta Son denominadas tambin lnea de campo elctrico,


g : Constante dielctrica sirven para descubrir en forma grfica la accin de un
-12
campo elctrico. Estas lneas fueron ideadas por Faraday
go : 8,85.10 quien convencionalmente estableci que toda lnea de
fuerza sale de una carga positiva e ingresa a otra carga
negativa
g : Permisividad elctrica del aire o vaco

Unidades en el SI

F ! Newton (N)
q1 y q 2 ! Coulomb (C)
d ! Metros (M)

9
K = 9 .10

CAMPO ELCTRICO

CONCEPTO : Es una forma de existencia de la materia


que permite la transmisin de interacciones elctricas
entre dos cuerpos cargados REPULSIN
En la naturaleza toda partcula o cuerpo necesariamente
posee carga y campo elctrico

INTENSIDAD DEL CAMPO ELCTRICO ATRACCIN :

-102-
ASOCIACIN EDUCATIVA PITGORAS Fsica

3. Todo campo elctrico uniforme es decir que pose el


mismo mdulo y propiedades fsicas viene a ser ...
por un conjunto de lneas de fuerzas paralelas entre
s

PROPIEDADES DE LAS LNEAS DE FUERZA

1. A mayor nmero de lneas de fuerza el cuerpo


conductor tendr mayor cantidad de carga elctrica,
y as mismo su campo elctrico ser mas intenso

N lneas Cantidad de

de fuerza carga elctrica

2. La intensidad de campo elctrico resultante siempre 4. Toda lnea de fuerza siempre es perpendicular a toda
es tangente o colineal a toda lnea de fuerza y por superficie conductora esto es debido a que para cada
ese motivo las lneas de fuerza nunca llegan a carga positiva existe su correspondiente carga
cortarse entre s negativa

PROBLEMAS PROPUESTOS

01. Dos cargas elctricas se repelen con 10 N. Si la


distancia que los separa se reduce a la mitad y cada
una de las cargas se duplica entonces, la nueva
fuerza de repulsin ser de :

A) 16 N B) 160 N C) 80 N
D) 40 N E) 45 N

02. Dos cargas puntuales separadas 60 cm en el vaco


se repelen con una fuerza de 10 N. Calcular el valor
de las cargas si una de ellas es cuatro veces mayor
que la otra. A) 16 N; 32 N B) 2,4 N; 4,8 N C) 24 N; 48 N
D) 1,6 N; 3,2 N E) 0,16 N; 0,32 N
-5 -5 -4 -4
A) 10 C; 4.10 C B) 10 C; 4.10 C
-2 -2
C) 10 C; 4.10 C
-3 -3
D) 10 C; 4.10 C 05. La figura muestra dos esferas idnticas de peso 10 N
-6 -6
E) 10 C; 4.10 C y carga q=20 C cada uno. Hallar la tensin en la
cuerda (1) y (2)
03. Tres cuerpos igualmente cargados estn localizados
como lo indica la figura.

La fuerza elctrica ejercida por A sobre B es de 20 N.


Cul es la fuerza elctrica resultante sobre B?
A) 100 N B) 45 N C) 25 N
D) 15 N E) 30 N

04. Dos esferas de igual peso e igual magnitud de carga A) 20 N; 50 N B) 30 N; 40 N C) 50 N; 60 N


-6
q = 4.10 C se encuentran en equilibrio en la forma D) 35 N; 30 N E) 30 N; 60 N
mostrada. Calcular el peso de las esferas y la tensin
de la cuerda.
06. En el sistema de cargas mostrado, cul es la fuerza

-103-
ASOCIACIN EDUCATIVA PITGORAS Fsica

elctrica resultante que acta sobre la carga -q?


(L= lado del cuadrado)

2
A) 2K(q/L) 2 2 A) -8 C B) -6 C C) -4 C
2 B) 4K(q/L)2 C) 2 K(q/L)
D) 3K(q/L) E) 2K(q/L) D) -2 C E) - C
07. Hallar la fuerza resultante sobre la carga Q1" 12. Si el hexgono regular que se muestra en la figura es
(Q1 = 1.10-3C; Q2=3.10-4C; Q3=16.10-4C) de lado a, determine una expresin para la intensidad
del campo elctrico resultante en el centro del
hexgono (K = constante de la ley de Coulomb)

A) 500 N B) 300 N C) 400 N


D) 600 N E) 700 N

08. Hallar el valor de la reaccin normal de la pared


vertical sobre la esfera de carga Q1 y pesa 40 N; se
sabe que el sistema se encuentra en equilibrio y que
todas las superficies son lisas. Considere que : 2 2 2
Q2=2Q1 = 40 C A) 2KQ/a B) 3KQ/a C) 4KQ/a
2 2
D) 5KQ/a E) 6KQ/a

13. Sabiendo que existe equilibrio determinar la


deformacin del resorte (de material aislante) cuya
K = 15 N/cm
5
Datos : m=4 kg; q=+60 C y E = 5.10 N/C

A) 20 N B) 30 N C) 40 N
D) 50 N E) 60 N

09. Hallar el valor de la deformacin del resorte de


3
constante K=10 N/m sabiendo que el sistema se
encuentra en reposo. Datos :
q2=4q1=4.10-5 C, m1=2 kg y g=10 m/s2
A) 5 cm B) 4 cm C) 3 cm
D) 2 cm E) 1 cm

14. Se muestra el reposo de una carga de -10 C, cuyo


peso es de 20 N, en el interior de un campo elctrico
uniforme. Halle E

A) 5 mm B) 10 mm C) 15 mm
D) 20 mm E) 25 mm
-6 -6
10. Se tienen dos cargas de 5.10 C y 8.10 C separadas
por 6m. Hallar el campo elctrico resultante en el
punto medio de la lnea que las une
A) 5.103 N/C B) 3.103 N/C C) 8.103 N/C
2 3
D) 5.10 N/C E) 2.10 N/C

11. Tres cargas elctricas puntuales son colocadas en los


vrtices de un cuadrado, tal como se indica en la A) 0,8 N/C B) 1,0 N/C C) 1,2 N/C
figura. Determine el valor de la carga Q, si se sabe D) 1,4 N/C E) 1,6 N/C
que la intensidad del campo elctrico resultante en el
vrtice libre tiene direccin vertical

-104-
ASOCIACIN EDUCATIVA PITGORAS Fsica

15. En el interior de un ascensor que sube con


aceleracin a viaja un pndulo de masa m y carga
q, habiendo un campo elctrico uniforme E
horizontal. Halle Tg

g = Aceleracin de la gravedad

A) Sen = (ma - Eq)/mg


B) Sen = (Eq - ma)/mg
C) Tg = (ma - Eq)/mg
D) Tg = (Eq - ma)/mg
E) Cos = (Eq + ma)/mg

18. En los vrtices de un tetraedro regular de arista a, se


A) B) C) colocan cargas elctricas puntuales de valor Q cada
una. Determine una expresin para la intensidad del
campo elctrico resultante en el centro de una de
las caras del tetraedro. (K = constante de la ley de
D) E) Coulomb)
2 2 2
A) KQ/2a B) 3KQ/2a C) 5KQ/2a
16. Una esferita de masa 160 g y carga elctrica D) 7KQ/2a2 E) 9KQ/2a2
q=-200 C, es lanzada verticalmente hacia arriba
dentro del campo elctrico E=500 N/C. Determinar la 19. En los extremos de un dimetro de 12 cm de longitud
velocidad inicial de lanzamiento, para que la esferita que pertenece a la base de un cono de 8 cm de altura
alcance una altura mxima de 2 m (despreciar el se han colocado cargas elctricas puntuales de 5 nC
campo gravitatorio) cada una. Determine la intensidad del campo
elctrico resultante en el vrtice del cono

A) 1 800 N/C B) 3 600 N/C C) 4 800 N/C


D) 7 200 N/C E) 8 000 N/C

20. Suponga que un electrn de masa m y carga elctrica


e, se lanza con velocidad V segn el eje de dos
placas paralelas horizontales de longitud L. Si el
campo elctrico uniforme de intensidad E que existe
entre las placas, es vertical y hacia abajo, determine
A) 0,5 m/s B) 0,4 m/s C) 0,25 m/s el desplazamiento vertical efectuado por el electrn,
D) 0,2 m/s E) 0,1 m/s al salir de las placas
2 2 2 2 2 2
17. El coche mostrado se mueve con una aceleracin A) EeL /mV B) 2EeL /mV C) EeL /4mV
2 2 2 2
constante a y se sabe que entre las paredes D) EeL /2mV E) 4EeL /mV
verticales existe un campo elctrico uniforme de
intensidad . Determine el ngulo , sabiendo que la
esfera pendular es de masa m y carga q

TAREA

01. Un cuerpo A rechaza a un grupo de sustancias otro C) Aumenta al disminuir la distancia de separacin
cuerpo B rechaza a otro grupo de sustancias; pero D) Disminuye al disminuir la distancia de separacin
las sustancias de ambos grupos se atraen entre s, E) Ninguna anterior es correcta
entonces:
03. Hallar +q2 para que toda carga q colocada en A
A) A y B estn cargados positivamente quede siempre en equilibrio sabiendo que q1 =+18 C
B) A y B estn cargados negativamente
C) A est cargado positivamente y B negativamente
o viceversa
D) A est neutro y B est cargado positivamente o
negativamente
E) A y B estn polarizados o descargados
A) 72 C B) 42 C C) 36 C
02. La fuerza de atraccin entre dos cargas elctricas D) 28 C E) 56 C
puntuales:
A) Aumenta al disminuir el valor de las cargas
B) Disminuye al aumentar el valor de las cargas

-105-
ASOCIACIN EDUCATIVA PITGORAS Fsica

-3
04. En qu relacin se hallan las magnitudes de las 09. Una carga negativa de -10 C cuya masa es de
cargas -q2 y +q1 (q2/q1), para que la carga +q3 0,08 kg reposa sobre una superficie no conductora, el
permanezca en reposo? campo elctrico uniforme es de 1 000 N/C. Halle la
2
fuerza normal entre el plano y la carga. (g = 10 m/s )

A) 1/2 B) 2/3 C) 2/9


D) 1/4 E) 1/9

05. Se tiene cuatro cargas de igual valor Q, cada una


colocada en los vrtices de un cuadrado de lado L
y adems una carga q en el centro. Determinar la A) 0,2 N B) 0,8 N C) 1,8 N
fuerza de interaccin sobre q debido a las cargas D) 1,4 N E) 0
ubicadas en los vrtices
2 2 2 2 2 2
10. El sistema que se indica se encuentra en equilibrio,
A) KQ /L B) 4KQ /L C) 5KQ /L se sabe que el cuerpo pendular pesa 8 N, posee
2 2
D) Cero E) 2KQ /L una carga elctrica de -20 C y adems que el
campo elctrico uniforme tiene una intensidad de
06. Hallar el valor de la carga Q, si se sabe que se 4.105 N/C. Determine el valor de
encuentra en equilibrio y pesa 120 N

A) 16 B) 30 C) 37
-1 -2 -3 D) 45 E) 60
A) 10 C B) 10 C C) 10 C
-4 -5
D) 10 C E) 10 C
-6 -6
07. Se tienen dos cargas de 4.10 C y 9.10 C
separadas una distancia de 3 m. A qu distancia de
la primera carga el campo elctrico total es nulo?

A) 1,8 m B) 2 m C) 1 m
D) 1,2 m E) 1,5 m

08. Hallar el campo elctrico uniforme donde se


-3
encuentra en equilibrio la esferita de peso 4.10 N y
carga elctrica q = +10 C unida a un hilo de seda

A) 100 N/C B) 200 N/C C) 300 N/C


D) 400 N/C E) 500 N/C

-106-

You might also like